1700 Files Explained by Dr. Khalid Saifullah (Question 1 To Question 1707)
1700 Files Explained by Dr. Khalid Saifullah (Question 1 To Question 1707)
[The patient is
smoker and probably developed squamous cell
Saifullah (Question 1 to question
lung cancer which is working as a tumour
1707) producing ectopic ACTH causing pigmentation.
Resulting raised cortisole is leading to diabetes
8.A 53yo female presents with an acute 1. Active treatment [i) radical prostatectomy ii)
painful hot knee joint. She is a known case radical radiotherapy iii) hormone therapy iv)
of RA. On examination, the knee is red, brachytherapy v) pelvic radiotherapy vi)
orchidectomy
tender and swollen. The hamstring
muscles are in spasm. Her temp is 38.5C 2. Active surveillance
and BP is 120/80mmHg. 3. Watchful waiting
What is the SINGLE best next inv? 4. Palliative care (Source: NICE)].
A. Joint aspiration for cytology and
culture and sensitivity 12. An 18yo female has peri-orbital
B. Joint aspiration for positive blisters. Some of them are crusted, others
birefrengent crystals secreting pinkish fluid. What is the most
C. Joint aspiration for negative likely dx?
birefrengent crystals a. Shingles
D. Blood culture b. Chicken pox
E. Serum uric acid c. Varicella
d. Rubella
Ans. The likely key is A. Joint aspiration for
e. Measles
cytology and culture and sensitivity. [Any
chronically arthritic joint is predisposed to Ans. The key is A. Shingles. [Here ophthalmic
infection. Moreover chronic use of steroid in Rh. division of trigeminal nerve is involved. Typically
arthritis is one of the important predisposing shingles are unilateral].
factor. In this age group likely organism is
Staphylococcus. In younger age group Neisseria
gonorrhea is more common].
13. A 29yo lady who is a bank manager is
referred by the GP to the medical OPC due
to a long hx of tiredness and pain in the
9.An 80yo man presented with pain in his
joints. An autoimmune screen result
lower back and hip. He also complains of
showed smooth muscle antibodies
waking up in the night to go to the
positive. What is the most appropriate
washroom and has urgency as well as
next inv?
dribbling. What is the most likely dx?
a. ECG
a. BPH
b. TFT
b. Prostatitis
c. LFT
c. UTI
d. Serum glucose
d. Prostate carcinoma
e. Jejunal biopsy
e. Bladder carcinoma
Ans. The key is C. LFT. [A case of autoimmune
hepatitis. Autoimmune hepatitis is an uncommon
cause of chronic hepatitis which if untreated can Ans. The key is C. Small cells with round nucleus
lead to cirrhosis. However with treatment outlook and scant indistinct cytoplasm (Lobular
is very good. Smooth muscle antibody is positive carcinoma)
in autoimmune hepatitis. Definitive investigation
is liver biopsy. Treated with steroid [start with
high dose prednisolone]. Azathioprine is 16. A 22yo man has a reduced conscious
commonly added with steroid to reduce its dose level and a fixed dilated pupil after being
as steroid has more side effects than
involved in a MVC(motor vehicle crash) .
azathioprine].
Choose the single most appropriate
option?
a. Facial nerve
14. A 5yo with recurrent chest pain, finger b. Oculomotor nerve
clubbing with offensive stool. Choose the c. Olfactory nerve
single most likely investigation? d. Optic nerve
a. Endomyseal/Alpha glidin antibody e. Trigeminal nerve
b. Sweat test Ans. The key is B. Oculomotor nerve. [3rd nerve
c. Barium meal damage can cause fixed dilated pupil].
d. ECG
e. Glucose tolerance test
15. A clinical picture of breast cancer Ans. The key is D. Universal prevention
application protocol.
originated from the mammary duct.
Biopsy was done and there were
neoplastic cells found. Choose the 18. A 7yo child is brought to the ED with a
histological picture of the cancer? 1 day hx of being listless. On examination,
a. Neoplastic cells are arranged in small the child is drowsy with an extensive non-
clusters occupying a space between blanching rash. What advice would you
collagen bundles (Seirrhous carcinoma) give the parents?
b. Spindle cell neoplasms with margins,
which infiltrate adjacent structure, fat a. All family members need antibiotic
invaded (Breast sarcoma) therapy
c. Small cells with round nucleus and scant b. Only the mother should be given
indistinct cytoplasm (Lobular carcinoma) rifampicin prophylaxis
c. All family members need isolation
d. All family members should be given precursors of adenocarcinomas and follow a
rifampicin prophylaxis predictable cancerous temporal course unless
interrupted by treatment. They can be either
Ans. The key is D. All family members should be pedunculated or sessile. Polyps are generally
given rifampicin prophylaxis. [Meningococcal asymptomatic but may occasionally ulcerate and
disease. Diagnosis is done with blood or CSF PCR. bleed; uncommonly, they may result in
Initial prehospital management: Benzyl penicillin obstruction if very large.
or cefotaxime].
Adenomas are divided into 3 subtypes based on
histologic criteria, as follows: (1) tubular, (2)
tubulovillous, and (3) villous. According to World
19. A 47yo man has a temp of 39C and is
Health Organization (WHO) criteria, villous
delirious. He has developed blisters mainly adenomas are composed of greater than 80%
on his trunk, which appeared a few hours villous architecture. Tubular adenomas are
ago. He is well and not on any encountered most frequently (80-86%).
medications. He last travelled 5 months Tubulovillous adenomas are encountered less
ago to Italy. Which of the following is the frequently (8-16%), and villous adenomas are
most likely dx? encountered least frequently (5%).
a. Shingles
Villous adenomas are associated more often with
b. Chicken pox
larger adenomas and more severe degrees of
c. Pemphigoid
dysplasia. These adenomas occur more
d. Bullous pemphigus frequently in the rectum and rectosigmoid,
although they may occur anywhere in the colon.
Ans. The key is B. Chicken pox. [Adults more
They generally are sessile structures that appear
commonly develop a more generalized brain
as velvety or cauliflowerlike projections. See the
inflammation ("encephalitis") whose symptoms
images below.
may include delirium and seizures. Incubation
period of chicken-pox is 10-21 days. So this travel Note that the vast majority of patients are
history is not significant]. asymptomatic and have unremarkable laboratory
findings
Ans. The key is A. NSAIDs. [Oral NSAIDs 24. A 40yo woman notices increasing
commenced immediately and continue for 1 – 2 lower abdominal distention with little/no
weeks; Colchicine can be effective alternative but pain. On examination, a lobulated cystic
is slower to work than NSAIDs. Intra articular mass is felt and it seems to be arising
corticosteroids are highly effective in acute gouty from the pelvis. What is the most
monoarthritis.
appropriate inv?
22. A pt was lying down on the operating
a. CA 125
table in a position with his arms hanging b. CA 153
down for 3 hours. Soon after he woke up, c. CA 199
he complains of numbness and weakness d. CEA
in that hand and has limited wrist e. AFP
movement/wrist drop and sensory loss
Ans. The key is A. CA 125. [Ovarian ca is the likely
over dorsum of that hand, weakness of diagnosis for which tumour marker is CA 125].
extension of the fingers and loss of
sensation at the web of the thumb. What
structure is likely to be damaged? 25. A resident of a nursing home
a. Radial nerve presented with rashes in his finger webs
b. Median nerve and also on his abdomen, with complaints
c. Ulnar nerve of itching which is severe at night. He was
d. Axillary nerve dx with scabies. What the best tx for his
e. Suprascapular nerve condition?
Ans. The key is A. Radial nerve. [Here arm hanging a. 0.5% permethrin
down compressing the radial nerve at the spiral b. Doxycycline
groove is the cause of given scenario]. c. 5% permethrin
d. Reassure
e. Acyclovir
23. A pt who was previously on 120mg
Ans. The key is C. 5% permethrin. [Scabies
slow release oral morphine has had his
outbreaks in nursing homes and cases of crusted
dose increased to 200mg. He is still in scabies may require combination therapy
significant pain. He complains of consisting of topical application of permethrin
drowsiness and constipation. What is the and 2 oral doses of ivermectin at 200 mcg/kg
next step in the management? (administered 1 wk apart)].
41.A 24yo biker has been rescued after Ans. The key is B. Cardiac tamponade. [chest is
being trapped under rocks for almost 12h. clear, so there is no pneumothorax or pleural
effusion. Muffled heart sound is due to fluid in
He complains of reddish brown urine. His
pericardial space, low BP from reduced chamber
creatinine is 350umol/L and his urea is
expansion due to pericardial fluid’s pressure and
15mmol/L. What is the most important restricted right heart expansion causes raised
step in the management of this patient? JVP].
a. Dialysis
b. IV NS 44. A 50yo pt is admitted for elective
c. IV dextrose herniorraphy. Which of the following
d. IV KCl options will lead to a postponement of the
e. Pain relief operation?
Ans. Key is B. IV NS. [It is a case of rhabdomyolysis a. SBP 110mmHg
which is initially treated with IV NS]. b. MI 2 months ago
c. Hgb 12g/dl
d. Pain around hernia
42. A 74yo man who has been a smoker e. Abdominal distention
since he was 20 has recently been dx with
Ans. The key is B. MI 2 months ago. [After MI
SCLC. What serum electrolyte picture will
elective surgery should not be done before 6
confirm the presence of SIADH? months post MI, as operation in earlier than this
time has significant increase in mortality].
a. High serum Na, low serum osmolarity,
high urine osmolarity
45. A 32yo woman of 39wks gestation
b. Low serum Na, low serum osmolarity,
attends the antenatal day unit feeling very
high urine osmolarity
unwell with sudden onset of epigastric
c. Low serum Na, high serum osmolarity,
pain associated with nausea and
high urine osmolarity
vomiting. Her temp is 36.7C. Exam: she is
d. High serum Na, low serum osmolarity,
low urine osmolarity
found to have RUQ tenderness. Her blood
e. High serum Na, high serum osmolarity,
results show mild anemia, low platelets,
low urine osmolarity elevated liver enzymes and hemolysis.
What is the most likely dx?
Ans. . The key is B. Low serum Na, low serum a. Acute fatty liver of pregnancy
osmolarity, high urine osmolarity. b. Acute pyelonephritis
c. Cholecystitis
43. A man brought into the ED after being d. HELLP syndrome
stabbed in the chest. Chest is bilaterally e. Acute hepatitis
clear with muffled heart sounds. BP is
60/nil. Pulse is 120bpm. JVP raised. What Ans. The key is D. HELLP syndrome. [The main
treatment is to deliver the baby as soon as
is the most likely dx?
possible [as early as after 34 weeks if multisystem Ans. 1. The key is D. Subdural hematoma. [In
disease is present]. elderly head injury usually leads to subdural
hematoma even if head injury is minor or trivial
and extradural hematoma in elderly is extremely
46. A woman comes with an ulcerated
uncommon even in more severe head injury.
lesion 3 cm in the labia majorum. What is Management: 1st line: Evacuation by barr hole
the lymphatic drainage of this area? craniostomy. 2nd line: Craniotomy if the clot is
organized].
a. External iliac
b. Superficial inguinal LN
c. Para-aortic
49. A 25yo female complains of
d. Iliac
e. Aortic intermittent pain in her fingers. She
describes episodes of numbness and
Ans. Key is B. Superficial inguinal LN. burning of the fingers. She wears gloves
47. A man post-cholecystectomy whenever she leaves the house. What is
presented with jaundice, fever and dark the most probable dx?
urine. What is the most diagnostic inv? a. Kawasaki disease
a. ERCP b. Takayasu arteritis
b. USG Abdomen c. Buerger’s disease
c. CT Scan d. Embolism
d. MRCP e. Raynaud’s phenomenon
e. MRI
Ans. The key is A. ERCP [Post operative US will not Ans. The key is E. Raynaud’s phenomenon.
give good results. We shall not go for ercp first as [Intermittent nature points towards some triggers
it has complications like pancreatitis. Acceptable and wearing of gloves during going out indicates
options are CT, MRI and MRCP among which most cold weather. Also female sex makes the
easiest and less time consuming but with very diagnosis of Raynaud’s phenomenon more likely].
good test result is CT scan. So CT is most
appropriate! But as the Question wants most
diagnostic it is ERCP (though not practical)!!! The 50. A 22yo lady has been unwell for some
diagnosis here is choledocolithiasis with
time. She came to the hospital with
cholangitis].
complaints of fever and painful vesicles in
48. A 79yo stumbled and sustained a her left ear. What is the most probable
minor head injury 2 weeks ago. He has dx?
become increasingly confused, drowsy
and unsteady. He has a GCS of 13. He a. Acne
takes warfarin for Afib. What is the most b. Herpes zoster
likely dx? c. Chicken pox
a. Extradural hemorrhage d. Insect bite
b. Cerebellar hemorrhage e. Cellulitis
c. Epidural hemorrhage
Ans. The key is B. Herpes Zoster. [This is a case of
d. Subdural hemorrhage
Herpes zoster oticus and if facial nerve is also
e. Subarachnoid hemorrhage involved then it is called Ramsay Hunt syndrome].
51. A 5yo girl had earache and some e. Pyridoxine
yellowish foul smelling discharge, Ans. The key is A. Folic acid. [Frequently
perforation at the attic and conductive associated with neural tube defect].
hearing loss. She has no past hx of any ear
infections. What is the most appropriate 53. A 23yo woman has been having pain
at the base of her thumb, the pain is
dx?
reproduced when lifting her 3 month old
a. Acute OM baby or changing diapers and also with
b. OM with effusion forceful abduction of the thumb against
c. Acquired cholesteatoma resistance. What is the likely cause?
d. Congenital cholesteatoma a. Avascular necrosis of scaphoid
e. Otitis externa b. Trigger finger.
c. De Quervain’s tenosynovitis
Ans. 2. King's College Hospital criteria for liver Ans. The key is D. 1 alpha hydroxylation of Vit D.
transplantation in paracetamol-induced acute
liver failure. 73. Pt with puffiness of face and rash
arterial pH <7.3 or arterial lactate >3.0 mmol/L showing cotton wool spots on fundoscopy.
after adequate fluid resuscitation, OR What’s the dx?
if all three of the following occur in a 24-hour
a. Macular degeneration
period:
Creatinine >300 μmol/L. b. Hypertensive retinopathy
PT >100 seconds (INR >6.5). c. Diabetic background
Grade III/IV encephalopathy. d. Proliferative diabetic retinopathy
e. SLE
Iron-deficiency anaemia.
78. A 67yo man has deteriorating vision in
Folate deficiency or vitamin B12
his left eye. He has longstanding COPD deficiency.
and is on multiple drug therapy. What
single medication is likely to cause this Bleeding, resulting from low vitamin K.
visual deterioration? Oedema, which occurs in protein/calorie
a. B2 agonist malnutrition.
b. Corticosteroid
c. Diuretic 80. A 35yo male is bitterly annoyed with
d. Theophylline people around him. He thinks that people
are putting ideas into his head. What is
the single most likely dx?
Q. 1. What is the key? a. Thought block
Q. 2. What is the cause of deteriorating vision? b. Thought insertion
c. Thought broadcasting
d. Thought withdrawal
Ans. 1. The key is B. Corticosteroid.
e. Reference
Ans. 2. Prolonged corticostiroids [also topical i.e.
eye drop] can cause cataract. Q. 1. What is the key?
Q. 2. In which disease you will find this feature?
Ans. 1. The key is B. Thought insertion.
Ans. 1. The key is B. IV fluids calculated from the Ans. 1. The key is B. Penicillin.
time of burn.
Ans. 2. cellulitis is usually caused by
Ans. 2. Resuscitation fluids required in the first 24 staphylococcus and streptococcus. To cover both
hours from the time of injury. Flucloxacillin (for staphylococcus) and Penicillin
For adults: 3 ml (in partial thickness burn) (to cover streptococci) should be prescribed.
of Hartmann’s solution/kg body weight/% total
Ans. 4. Important risk factors: dermatitis, IV 90. An 11yo boy is being checked by the
injections, renal failure, organ transplantation, diabetic specialist nurse. His HbA1c was
DM, post operative wond. Risk factors for
high and he has been skipping meals 92. A man has reducible bulge below the
recently. He has been unhappy at school. pubic tubercle, and on occlusion of the
Which single member of the clinical team deep inguinal ring, cough impulse is
would you refer him to next? present. What is the most likely dx?
a. GP a. Direct inguinal
b. Pediatrician b. Indirect inguinal
c. Dietician c. Femoral
d. Clinical psychologist d. Spigelian
e. Lumbar
Cognitive behavioral therapy (CBT) usually lasts Ans. 1. The key is D. Achalasia.
for between 5 and 20 sessions, with each session
lasting 30 to 60 minutes, CBT works by changing Ans. 2. Points in favour: Aspiration pneumonia
people's attitudes and their behavior by focusing due to retained food and fluid in oesophagus. In
on the thoughts, images, beliefs and attitudes. achalasia usually there is no acid reflux.
Dysphagia for both food and drink. Air-fluid level
behind heart.
Why it is not hiatus hernia? Ans. Differentiating Q. 1. What is the key?
point:-i) In hiatus hernia usually you will get Q. 2. What is the type of the given case?
associated GORD ii) Also in hiatus hernia there Q. 3. What are the points in favour of mentioned
may be nausea or vomiting. type?
Ans. 2. Pregnancy itself is a risk factor for DIC. Q. 1. What is the key?
Placental abruption is a more common cause of Q. 2. What is the diagnosis?
DIC. Q. 3. What are the points in favour of your
diagnosis?
Other causes of pregnancy related DIC are:
eclampsia, retention of a dead fetus, amniotic Ans. 1. The key is D. Mature lymphocytes.
fluid embolism, retained placenta or bacterial
Ans. 2. The diagnosis is CLL.
sepsis.
Ans. 3. It is CLL because of his age (73 yrs),
103. A 28yo woman has delivered with cervical lymphadenpathy, recurrent infections
rotational forceps after an 8h labor and (mature but functionally defective lymphocytes),
3h second stage. Choose the single most and pale conjunctiva (anaemia).
likely predisposing factor for PPH for this
pt? 106. A 45yo lady has 10m hx of SOB. She
a. Atonic uterus is found to have irregularly irregular pulse
b. Cervical/vaginal trauma and loud P2 with fixed splitting and
c. Retained product ejection systolic murmur in left 2nd ICS.
d. Preterm labor What is the probable dx?
e. Uterine infection a. TOF
b. ASD 109. A 33yo male involved in a street fight
c. VSD presents with bruises and deformity in the
d. PDA upper part of his leg. XR shows fx of the
e. CoA neck of fibula. What is the single most
associated nerve injury?
Q. 1. What is the key?
a. Sciatic nerve
Q. 2. What is the diagnosis?
Ans. 1. The key is B. Atrial septal defect. b. Gluteal nerve
c. Musculocutaneous nerve
Ans. 2. Diagnosis is ASD with atrial fibrillation. [i) d. Lateral peroneal nerve
atrial fibrillation = irregularly irregular pulse. ii) e. Tibial nerve
ASD = SOB, fixed splitting with loud P2, ESM in
f. Femoral nerve
pulmonary area]. This picture is typical. One
should not misdiagnose SOB, ESM in pulmonary Ans. The key is D. Lateral peroneal nerve. [Lateral
area and loud P2 as pulmonary hypertension peroneal nerve is other name of superficial
(though in elderly this can develop with ASD). peroneal nerve].
107. A 5m baby present with recurrent 110. A 35yo man presents with hx of
vomiting. Mother noticed some of the dyspepsia. H.Pylori antibodies are
vomitus is blood stained. Choose the negative. No improvement is seen after
single most likely inv? 1m of tx. What is the next step?
a. Upper GI endoscopy a. Urea breath test
b. Barium meal b. Gastroscopy
c. US c. CT
d. Colonoscopy d. MRI
e. CT abdomen
Ans. The key A. TOF. [TOF usually does not Q. 1. What is the key?
become symptomatic at birth or early infancy and Q. 2. What are the points in favour of your
given features (central cyanosis and clubbing with diagnosis?
murmur of right ventricular outflow obstruction Ans. Given key is E. Osteomyelitis which is a
i.e. ejection systolic murmur in 2nd left ICS) are wrong key. The correct answer is B. Septic
well known features of TOF]. [Tetralogy of fallot = arthritis.
1. VSD + 2. Overriding of the aorta + Right Ans. Points in favour of diagnosis: i) Pain in joints
ventricular outflow tract obstruction + Right (knee and hip). In osteeomyelitis there is no joint
ventricular hypertrophy]. pain but pain in other parts of bone like shaft. ii)
Fever iii) Painful restricted movement of joint.
121. An 8yo child who is tall for his age NB This controversial question was debated and
ultimately settled as septic arthritis by old
and has a refractory error for which he
plabbers.
wears glasses has presented with severe
crushing chest pain. What is the most 123. A man with anterior resection and
likely dx? end to end anastomosis done complains
a. Fragile X syndrome of severe pain in the chest and abdominal
b. Prader-willi syndrome distension. What is the most appropriate
c. DiGeorge syndrome inv likely to review the cause this
d. Marfans syndrome deterioration?
a. XR abdomen
Q. 1. What is the key?
b. Exploratory laparoscopy
Q. 2. What is the cause of this severe crushing
chest pain? c. CT
Q. 3. What are the most common cardiac d. US
abnormalitis found in this disease? e. Laparotomy
Ans. 1. The key is D. Marfans syndrome. Ans. The key is E. Laparotomy. It is a wrong key!
Right key is C. CT. [This is likely an anastomotic
Ans. 2. Cause of severe crushing chest pain may leak].
be aortic dissection.
145. A 53yo woman presented with pain Chronic myelogenous leukemia (CML), also
known as chronic myeloid leukemia, is a cancer of
in the eye, blurry vision and clumsiness for
the white blood cells. It is a form of leukemia
3 months. She has a hx of difficulty in
characterized by the increased and unregulated
swallowing and weakness in her right growth of myeloid cells in the bone marrow and
upper limb 2y ago. What is the the accumulation of these cells in the blood. CML
investigation of choice? is a clonal bone marrow stem cell disorder in
a. CSF analysis which a proliferation of mature granulocytes
b. EEG (neutrophils, eosinophils and basophils) and their
c. EMG precursors is found. It is a type of
d. MRI brain myeloproliferative neoplasm associated with a
e. Visual evoked response test characteristic chromosomal translocation called
the Philadelphia chromosome.
Q. What is the key?
e. Uterine artery embolization
147. A 6yo pt comes with easy bruising in Ans. The key is E. Uterine artery embolization.
different places when she falls. CBC: [Done by interventional radiologist expert in
WBC=25, Hgb=10.9, Plt=45. Her Paul arterial embolization technique. Particles are
Bunnel test +ve. What is the most likely placed in uterine artery to block circulation to
dx? uterine body. No operation or GA is required].
a. Glandular fever
149. A girl with hx of allergies visited a
b. ITP
friend’s farm. She got stridor, wheeze and
c. Trauma
erythematous rash. What is the most
d. NAI
appropriate tx?
e. Septicemia
a. 0.25ml IM adrenaline
Q. 1. What is the key? b. 0.25ml PO adrenaline
Q. 2. What are the lab. Values that suggests the c. 0.25ml IV adrenaline
diagnosis here? d. IV chlorphearamine
Ans. 1. The key is A. Glandular fever = infectious Ans. The key is A. 0.25 ml IM adrenaline
mononucleosis [Presence of stridor and wheeze are suggestive of
anaphilaxis and treatment option is IM
Ans. 2. Suggestive lab. Values: WBC=25 adrenaline].
(leucocytosis), Hgb=10.9 (usually patient is not
anaemic), Plt=45 (thrombocytopenia-leading to 150. A 5yo boy is referred to the hospital
easy bruising), Positive paul bunnel test. … and seen with his father who is worried
The Paul Bunnell test is used to screen for that he has been listless. He is not sure
infectious mononucleosis (IM), which is seen why his GP suggested he should come to
most commonly in adolescents and young adults the ED and is keen to get some tablets
…. mononuclear spot test or monospot test, a and go home. Exam: tired and irritable,
form of the heterophile antibody test,is a rapid swelling around eyes. Renal biopsy:
test for infectious mononucleosis due to Epstein– remarkable for podocyte fusion on EM.
Barr virus (EBV). It is an improvement on the What is the most probable dx?
Paul–Bunnell test. a. NAI
b. Myelodysplastic disease
148. A 41yo woman who has completed
c. HSP
her family, has suffered from extremely
d. Membranous GN
heavy periods for many years. No medical
e. Minimal change GN
tx has worked. She admits that she would
rather avoid open surgery. After Ans. The key is E. Minimal change
discussion, you collectively decide on a glomerulonephritis. [Podocyte fusion on electron
procedure that wouldn’t require open microscopy]
surgery or GA.
151. A 6yo boy is brought to the hospital
Select the most appropriate management
for a 3rd episode of sore throat in 1
for this case.
a. Endometrial ablation
month. He is found bleeding from gums
b. Hysterectomy
and nose and has pale conjunctiva.
c. Fibroid resection What’s the single cell type?
a. Clumped platelets
d. Myomectomy
b. Microcytes
c. Granulocyte without blast cells tension pneumothorax is more in stab wond and
d. Blast cells no tracheal deviation is controversial. This is
e. Mature lymphocytes probability of bad recall!!
156. A 12yo pt presents with copious 158. An otherwise healthy 13yo boy
diarrhea. Exam: urine output=low, presents with recurrent episodes of facial
mucous membrane=dry, skin turgor =low. and tongue swelling and abdominal pain.
What is the most appropriate initial His father has had similar episodes.
management? What is the most likely dx?
a. Antibiotic a. C1 esterase deficiency
b. Antimotility b. HIV
c. Anti-emetic c. Mumps
d. Fluid replacement d. Sarcoidosis
e. Reassurance e. Sjogren’s syndrome
[http://www.urmc.rochester.edu/encyclopedia/
165. A 20yo man complains of recent
content.aspx?
onset of itching which followed a viral
contenttypeid=22&contentid=flailchest]
infection. There are numerous wheals of
all sizes on his skin particularly after he
has scratched it. These can last up to an
163. A 28yo man with complains of
hour. What is the most probable dx?
headache and nose bleeds also has pain in
a. Uremia
the lower limbs on exertion. O/E: radio-
b. Urticaria
femoral delay, cold legs with weak pulse c. Psychogenic itching
and mild systolic murmur with normal d. Atopic eczema
S1S2. What is the most probable dx? e. Primary biliary cirrhosis
a. TOF
b. ASD
c. VSD
Ans. The key is B. Urticaria.
d. PDA
e. CoA
169. A 40yo man collapsed at home and 171. A 50yo chronic smoker came to OPD
died. The GPs report says he suffered from with complaint of chronic productive
cough, SOB and wheeze. Labs: Ans. The key is C. Minimal change disease. [Points
CBC=increase in PCV. CXR >6ribs seen in favour: i) Age 15 yrs ii) Generalized oedema iii)
above the diaphragm in midclavicular line. Protein in urine +++ vi) Normal eGFR of 110
(Normal range- 90 to 120 mL/min)].
ABG=pO2 decreased. What is the most
likely dx? 174. A 72yo man is receiving
a. Interstitial lung disease chemotherapy for SCLC. He has his 4th tx
b. Wegener’s granulomatosis 8 days ago. He has a cough with some
c. Ca bronchi green sputum but feels well. Temp=37.6C.
d. COPD
Chest exam = few coarse crepitations in
e. Amyloidosis
the right base. HR=92bpm. CBC:
Q. 1. What is the key? Hgb=12.5g/dL, WBC=1.1, Neutrophils=0.6,
Q. 2. What are the points in favour? Plt=89. Sputum, urine and blood culture
Ans. 1. The key is D. COPD. sent to microbiology.
Ans. 2. Points in favour: i) Age 50 yrs ii) Chronic What is the most appropriate
smoker iii) Chronic productive cough, SOB and management?
Wheeze iv) Raised PCV (hematocrit)secondary to a. Broad spectrum antibiotics IV
chronic hypoxaemia v) Low set diaphragm and b. Broad spectrum antibiotics PO
widened horizontal ribs vi) Hypoxaemia on ABG. c. GCSF
d. Postpone tx until bacteriology results
172. A 44yo pt has sudden onset of available
breathlessness and stridor few minutes e. Reassure and send home
after extubation for thyroidectomy. The
pat had longstanding goiter for which he Q. 1. What is the key?
had the surgery. What is the most likely Q. 2. What is the Diagnosis?
Q. 3. What is the treatment of low WBC count?
dx? Ans. 1. The key is A. Broad spectrum antibiotics IV
a. Thyroid storm
b. Hematoma Ans. 2. The diagnosis is lower respiratory tract
c. Unilateral recurrent laryngeal nerve infection.
injury Ans. 3. GCSF(granulocyte-colony stimulating
d. External laryngeal nerve injury factor) subcutaneously. [it is the treatment of
e. Tracheomalacia chemotherapy induced leucopenia]
Ans. The key is tracheomalacia. [Prolonged 175. A 25yo woman with T1DM has
pressure over trachea by goiter is a cause of delivered a baby weighing 4.5kg. Her
tracheomalacia following thryroidectomy]. uterus is well contracted. Choose the
173. A 15yo boy presents with generalized single most likely predisposing factor for
edema. His urinalysis reveals protein +++, PPH from the options?
a. Atonic uterus
eGFR =110. What is the most likely dx?
a. IgA nephropathy b. Cervical/vaginal trauma
b. Membranous nephropathy c. Retained POC
c. Minimal change disease d. Large placental site
d. PSGN e. Rupture uterus
e. Lupus nephritis
Q. 1. What is the key?
Q. 2. Reason for your answer.
Ans. 1. The key is B. Cervical/vaginal trauma c. Proguanil
d. Quinine
Ans. 2. The baby is a big baby. If patient’s uterus
e. Artesonate
was not well contracted we would fear of atonic
uterus! But as uterus is well contracted it is not Q. 1. What is the key?
atonic uterus. Rather most likely cause is trauma Q. 2. What does Shuffner’s dot in RBC indicate?
dring delivery of this big baby.
Ans. 1. The key is B. Mefloquine. Probably wrong
176. A 23yo lady presents with headache. key as mefloquine can not eradicate hepatic
Exam: photophobia and generalized rash cycle! Primaquine is the drug that can eradicate
that doesn’t blanch on pressure. What hepatic cycle.
must be done immediately?
a. IV benzylpenicillin Ans. 2. Shuffners dot indicates, it is plasmodium
b. Isolate pt ovale or plasmodium vivex infestation.
c. Gown and mask 179. A 35yo woman had an uneventful lap
d. Blood culture chole 18h ago(uneventful laparoscopic
cholecystectomy). She has a pulse=108bpm,
temp 37.8C. There are signs of reduced air
Ans. The key is A. IV benzylpenicillin. entry at the right base but the CXR doesn’t
show an obvious abnormality. What is the
most appropriate management strategy?
177. A 4yo baby has generalized tonic- a. Cefuroxime PO
clonic seizure and fever of 39C. his mother b. Ceftriaxone IV
informs you that this has happened 3-4x c. Chlorpheniramine PO
before. What is the most likely dx? d. Chest physiotherapy
a. Febrile convulsion e. Reassure
b. Absence seizures
c. Epilepsy Q. 1. What is the key?
d. Partial complex seizure Q. 2. What is the diagnosis?
Ans: The key is C. Epilepsy. It is wrong key. Ans. 1. The key is D. Chest physiotherapy.
Correct key should be A. Febrile convulsion. Ans. 2. Atelactasis. [Atelectasis can be seen in
[Points in favour: seizers in presence of high chest x-ray but sometimes it becomes
fever. Same previous illness indicate seizers undetectable in chest x-ray as we are suspecting
during fever which goes against epilepsy. Though in this case].
usually febrile convulsion dosen’t tend to recur
but it can recur as well. Epilepsy has no relation 180. A 20yo pop star singer complains of
to fever but is due to pathology in brain]. inability to raise the pitch of her voice. She
178. A middle aged Asian presents with attributes this to the thyroid surgery she
underwent a few months back.
episodes of fever with rigors and chills for
last 1y. Blood film: ring form plasmodium What is the most likely dx?
a. Thyroid storm
with schaffners dots in RBCs.
b. Bilateral recurrent laryngeal nerve injury
What is the drug to eradicate this
c. Unilateral recurrent laryngeal nerve
infection? injury
a. Doxycycline
d. External laryngeal nerve injury
b. Mefloquine
e. Thyroid cyst
Ans. The key is D. External laryngeal nerve injury. facies and CXR shows absent thymic
[External laryngeal nerve functions to tense the shadow. What is the most likely dx?
vocal cords by activating the cricothyroid muscle, a. Downs syndrome
increasing pitch]. b. Fragile X syndrome
181. A 28yo woman at 39wk gestation is c. DiGeorge syndrome
d. Marfans syndrome
in labor. She develops abdominal pain and
HR=125bpm, BP=100/42mmHg, Q. 1. What is the key?
temp=37.2C and saturation=99%. Exam: Q. 2. What are the points in favour?
lower abdomen is exquisitely tender. Ans. 1. The key is C. DiGeorge syndrome.
CTG=prv normal, now showing reduced Ans. 2. Points in favour: i) Early age of onset ii)
variability and late deceleration develops abnormal facies iii) absent thymic shadow on
with slow recovery. She has had 1 prv LSCS Chest X-ray iii) history of recurrent infection
for a breech baby. Choose the most [Infections are common in children due to
appropriate CS complication for this lady? problems with the immune system's T-cell-
a. Endometritis mediated response that in some patients is due
b. UTI to an absent or hypoplastic thymus]. [in
c. Urinary tract injury newborne can be recognized by convulsions from
d. Pleurisy hypocalcaemia due to malfunctioning parathyroid
e. Uterine rupture glands and low level of parathyroid hormones].
Ans. The key is E. Uterine rupture. [Features of 184. A 30yo man presents with deep
shock, exquisitely tender lower abdomen with penetrating knife wound. He said he had
abdominal pain, previously normal CTG now TT when he left school. What will you do
becoming non-reassuring and previous history of for him now?
LSCS makes the diagnosis of uterine rupture]. a. Human Ig only
b. Human Ig and TT
182. An 8m infant presented with c. Full course of tetanus vaccine only
FTT(Failure to thrive) and constipation. d. Human Ig and full course of tetanus
Exam: large tongue and fam hx of vaccine
prolonged neonatal jaundice. What is the e. Antibiotic
most likely dx?
a. Downs syndrome Ans. The key is B. Human Ig and TT. It is a wrong
b. Fragile X syndrome key!! According to UK greenbook correct key is A.
c. Praderwilli syndrome Human Ig only.??????
d. DiGeorge syndrome
185. A 32yo previously healthy woman
e. Congenital hypothyroidism
has developed pain and swelling of both
Q. 1. What is the key? knees and ankles with nodular rash over
Q. 2. What are the points in favour? her shins. As part of the inv a CXR has
Ans.1. The key is E. Congenital hypothyroidism. been performed. What is the single most
Ans. 2. Points in fevour:i) FTT (failure to thrive) ii) likely CXR appearance?
constipation iii) macroglossia iv) prolonged a. Apical granuloma
neonatal jaundice. b. Bilateral hilar lymphadenopathy
c. Lobar consolidation
183. A 3m infant has presented with d. Pleural effusion
recurrent infections. He has abnormal e. Reticular shadowing in the bases
Q. 1. What is the key? c. Hydrocele
Q. 2. What is the name of this condition? What d. Teratoma
are the points in favour? e. Testicular torsion
Ans. 1. The key is B. bilateral hilar Ans. The key is A. Epididymal cyst. [the location of
lymphadenopathy. upper pole of the posterior part of testis is the
Ans. 2. The name is Lofgren’s syndrome. It is the common site for epididymal pathology].
triad of i) erythema nodosum ii) bilateral hilar
lymphadenopathy and iii) arthralgia. An epididymal cyst is a fluid-filled cyst found in
Lofgren's syndrome is an acute form of the epididymis, a coiled tube in the back of the
sarcoidosis characterized by erythema nodosum, testicle that stores and transports sperm. This
bilateral hilar lymphadenopathy (BHL), and creates a benign lump in the testicle and can be
polyarthralgia or polyarthritis. This syndrome is
common among Caucasians . very common with increasing age. An epididymal
cyst doesn't require treatment unless it causes
pain or discomfort, may need surgery.
186. A neonate’s CXR shows double
bubble sign. Exam: low set ears, flat
occiput. What is the most likely dx?
a. Downs syndrome
189. A young footballer has collapsed
b. Fragile X syndrome during a game. During initial evaluation:
c. Turner’s syndrome RR=14/min, pulse=88bpm,
d. DiGeorge syndrome BP=110/70mmHg. He seems to be
sweating and muttering some
Q. 1. What is the key? incomprehensible words. What is the most
Q. 2. What double bubble sign indicate? imp next step?
Ans. 1. The key is A. Down’s syndrome. a. CT
Ans. 2. Double bubble sign indicate duodenal b. MRI
atresia. c. Blood sugar
d. Body temp
187. A 19yo boy complains of itching on e. IV fluids
the site of insect bite. What is the single
most appropriate management? Ans. The key is C. Blood sugar. [Normal vital signs,
a. Penicillin oral sweating and collapse during exercise gives
b. Doxycycline oral suspicion of hypoglycemia which is very easy to
c. Oral antihistamine treat but fatal if left unrecognized. So most
d. Oral ciprofloxacin important next step is to check the blood sugar].
e. Reassurance 190. A 45yo waitress complains of pelvic
Ans. The key is C. Oral antihistamine. pain which worsens pre-menstrually and
on standing and walking. She also
188. A man presents with scrotal swelling, complains of post-coital ache. Select the
the swelling is cystic and is non-tender. It most likely cause leading to her
is located in the upper pole of the symptoms?
posterior part of the testis. What is the a. PID
most likely dx? b. Endometritiosis
a. Epididymal cyst c. Pelvic congestion syndrome
b. Testicular ca d. Adenomyosis
e. Premature ovarian failure c. Vocabulary consists of only 2 meaningless
words
Ans. The key is C. Pelvic congestion syndrome. d. She can’t make a sentence
[Women with pelvic congestion syndrome
e. None
experience a constant pain that may be dull and
aching, but is occasionally more acute. The pain is Q. 1. What is the key?
worse at the end of the day and after long Q. 2. How many words is told clearly by a 15
periods of standing, and sufferers get relief when month old child?
they lie down. The pain is worse during or after Ans. 1. The key is C. Vocabulary consists of only 2
sexual intercourse, and can be worse just before meaningless words.
the onset of the menstrual period. Cause is
Ans. 2. At 15 months the child can clearly say 5
attributed to female hormone estrogen induced
words and his first meaningful clear word he says
vascular (venous) changes. So it is usually seen in
at 12 months.
females].
Pelvic congestion syndrome (PCS) is a chronic 193. A 35yo lady who has been using IUCD
condition that occurs in women when varicose for one year now complains of pelvic pain
veins form below the abdomen within the pelvic and heavy painful periods. Select the most
region. Varicose veins are veins that become likely cause leading to her symptoms?
swollen, twisted, and lengthened as a result of a. PID
poor vein function, Pelvic congestion syndrome b. Endometriosis
occurs when varicose veins develop around the c. Adenomyosis
ovaries, similar to varicose veins that occur in the d. Fibroids
legs … Possible treatment options include: e. Asherman syndrome
Gonadotropin-releasing hormone drugs, which
block ovarian function and may relieve pain. Q. 1. What is the key?
Progestin hormone drugs, which may relieve Q. 2. What points favour your diagnosis?
pain. Procedures to shut off damaged veins Ans. 1. The key is A. PID.
(sclerotherapy, embolization) Ans. 2. The given picture may have D/D of PID or
191. A 37yo female had a fall with fibroid. As IUCD is a risk factor for PID, it is the
most likely diagnosis of given picture.
outstretched hand, presented with dinner
Menorrhagia for fibroids are usually painless].
fork deformity and tenderness over the
right arm. What is the single most
associated nerve injury?
a. Axillary nerve
b. Radial nerve
c. Musculocutaneous nerve
d. Median nerve 194. The dx cells of Hodgkin disease are:
e. Ulnar nerve
a. T-cells
Ans. The key is D. Median nerve. [Median nerve is b. R-S cells
the nerve injured in Colle’s fracture]. c. B-cells
d. Macrophages
192. A mother comes with her 15m child. e. Auer rods
Which of the following will bother you?
Ans. The key is B. R-S cells. [Diagnostic cell in
a. Shies away from strangers Hodgkins disease is Reed-Sternberg cells].
b. Can walk but not run
195. A 16yo girl is admitted after taking a
paracetamol OD 4 h ago. She has
consumed large amounts of alcohol. Her
plasma paracetamol conc is just below the
conc that would suggest tx. What should
be the tx option for her? 197. A young girl complains of episodic
a. Refer to psychiatry ward headaches preceded by fortification
b. Refer to medical ward spectra. Each episode last for 2-3 days.
c. N-acetylcystine During headache pt prefers quiet, dark
d. Serum plasma paracetamol room.
e. No further investigation What is the tx of choice for acute stage?
a. Paracetamol
Ans. The key is A. Refer to psychiatry ward. [Short
b. Aspirin
term acute alcohol consumption causes enzyme
c. Sumatriptan
inhibition as in present case and even then
paracetamol level is under tx level. So the d. Gabapentin
patients drug level is in safe side but e. Cafergot
simultaneous drug overdose and alcohol
Ans. The key is B. Aspirin. [OHCM, 9th Eition, page-
consumption needs psychiatric evaluation and
462 where NSAIDS like ketoprophen or
hence the option here is A].
dispersible aspirin 900 mg/6 hr are recommended
196. A 64yo woman has been on HRT for as treatment in acute stage. Dx is migraine with
9yrs. She had regular withdrawal bleeds aura].
until 3 yrs ago and since then has been
taking a no bleed prep. Recently she
noticed a brown vaginal discharge. 198. A 60yo pt recovering from a surgery
Choose the single most appropriate initial for toxic goiter is found to be hypotensive,
inv? cyanosed in the the RR. Exam: tense neck.
a. Cervical smear There is blood oozing from the drain.
b. High vaginal swab What is the most likely dx?
c. TFT a. Thyroid storm
d. Transvaginal US b. Reactionary hemorrhage
c. Secondary hemorrhage
Q. 1. What is the key? d. Primary hemorrhage
Q. 2. Why this test will be done? e. Tracheomalacia
Ans. 1. The key is D. Transvaginal US.
206. A 23 yo girl presented with perioral 207. A 25 yo woman has been feeling
paresthesia and carpopedal spasm 20 anxious and nervous for the last few
mins after a huge argument with her months. She also complains of
boyfriend. What is the next step for this palpitations and tremors. Her symptoms
pt? last for a few minutes and are very hard
a. SSRI to control. She tells you that taking
b. Diazepam alcohol initially helped her relieve her
c. Rebreath into a paper bag symptoms but now this effect is wearing
d. Propranolol off and she has her symptoms even after
e. Alprazolam drinking alcohol. What is the dx?
a. Panic disorder
Q. 1. What is the key? b. Depression
Q. 2. What is the likely diagnosis? c. OCD
Ans. 1. The key is C. Rebreathin in paper bag. d. Alcohol addiction
[hyperventilation causes CO2 washout and
e. GAD 210. A 2 month child with diarrhea and
vomiting for 6 days is brought in looking
Ans. The key is A. Panic disorder.
lethargic. What is the appropriate initial
208. A 2yo child is very naughty. His inv?
teacher complains that he is easily a. BUE
distracted. His parents say that he can’t b. Random blood sugar
do a particular task for a long time. He c. CBC
sometimes hurts himself and breaks many d. CXR
things. This causes many troubles at e. AXR
home. What is the dx? Ans. The key is A. BUE. [Electrolyte imbalance
a. ASD
should be checked in this baby presenting with
b. Dyslexia diarrhea and vomiting for considerable time].
c. ADHD
d. Antisocial personality disorder 211. A 72 yo man fell while shopping and
e. Oppositional defiant hurt his knee. His vitals are fine. He speaks
in a low voice and is very slow to give
Ans. The key is C. ADHD (Attention deficit answers. What is the most probable dx?
hyperreactive disorder). Age??? a. Alzheimers
b. Vascular demetia
c. TIA
d. Pseudo-dementia
209. A 79 yo lady who is otherwise well e. Picks dementia
recently started abdominal pain. She is
Q. 1. What is the key?
afebrile and complains that she passed air Q. 2. What are the points in favour?
bubbles during urination. A urethral
catheter showed fecal leakage in the Ans. 1. The key is A. Alzheimers.
urinary bag. What is the likely pathology? Ans. 2. Points in favour: i) age 72 yrs ii) fall iii) loss
a. Diuretics
or slowness of speech.
b. CD
c. Rectosigmoid tumor why not vascular? in vascular: i) confusion ii)
d. Large bowel perforation disorientation iii)loss of vision
e. UC why not pseudodementia? in pseudo i) onset is
short and abrupt ii associated depression
Ans. The key is A. MRI cerebellum. [Balance Ans. The key is C. Exploratory surgery. [To exclude
difficulties, vomitin and nausea suggests torsion].
cerebellar lesion. In posterior fossa lesion MRI is
preferred].
221. A 47 yo ex-soldier suffers from low
219. A 2y pt presents with colicky pain mood and anxiety. He can’t forget the
which radiates from loin to groin. He images he faces before and has always
complains of similar episodes in the past. had flashbacks. He is not able to watch
Inv has been done and 7mm stone was the news because there are usually some
found in the ureter. reports about war. What is he suffering
What is the most appropriate from?
management? a. Depression
a. Percutaneous nephrolithiotomy b. PTSD
b. Open surgery c. Panic attack
c. Ureteroscopy or laser d. Agoraphobia
d. Conservative tx e. GAD
e. ESWL
242. A 31yo single man lives with his 245. A teacher brings in a child who says
mother. He usually drives to work. He she fell down after hitting a table. On
always thinks when the traffic lights probing further, you decide that it was
change, his mother is calling him, so he most probably an absence seizure.
drives back home. What led you to this dx?
a. The child had not eaten since morning
What is the dx?
b. The child suddenly went blank and there
a. OCD
was up-rolling of eyes
b. GAD
c. The child started moving his fingers
c. Schizophrenia
uncontrollably before he fell
d. Bipolar
d. The child’s body became rigid and then
e. Cyclothymia
started to jerk
243. A 56yo woman is known case of 246. A man has discharge from his left ear
pernicious anemia. She refuses to take after a fight. Where is the discharge
hydroxycobalamin IM as she is needle shy. coming from?
She asks for oral medication. a. CSF
Why will oral meds be not effective? b. Inner ear
a. Intrinsic factor def c. Outer ear
b. Malabsorption d. Brain
c. Irritated gastric mucosa
d. Lack of gastric acidity
Ans. The key is A. CSF. [probable fracture base of
Ans. The key is A. Intrinsic factor def. [Vitamin skull].
B12 cannot be absorbed without intrinsic factor].
247. A 40 yo manic depressive is noted to takes longer than before to finish tasks as
have high serum levels of lithium and she must constantly wash her hands.
profound What is the most appropriate
hypokalemia. His GP had started him on management?
anti-HTNs. Choose the single most likely a. CBT
cause? b. SSRI
a. Verapamil c. ECT
b. Amiodarone d. Antipsychotics
c. Ranitidine e. Desensitization
d. Lithium
Q. 1. What is the key?
e. Thiazide
Q. 2. What is the diagnosis?
Ans. The key is E. Thiazide. [Thiazide was Ans. 1. The key is A. CBT.
prescribed for Hpt and when lithium was Ans. 2. The diagnosis is OCD.
prescribed its level increased due to thiazide and
thiazide also caused hypokalemia resulting the
given picture].
250. A 61yo man underwent a surgery in
which ileal resection had been done. He
complains of fatigue, headache, and heart
248. A 74yo man presents with weakness
racing. Labs: MCV=108fL, Hgb=8.9g/dL.
in his arm and leg from which he
What is the most likely dx?
recovered within a few days and short a. Vit B12 def
term memory loss. He has an extensor b. Iron def
plantar response. He has similar episodes c. Folate def
2 years ago and became unable to identify d. Hemolytic anemia
objects and to make proper judgment. e. Anemia of chronic disease
What is the most appropriate dx?
a. Alcoholic dementia Q. 1. What is the key?
b. Pick’s dementia Q. 2. What are the points in favour?
c. Huntington’s disease
d. Alzheimer’s disease Ans. 1. The key is Vit. B12 deficiency.
e. Vascular dementia
Ans. 2. Vit B12 is absorbed mostly in ileum. [As
ileal resection is done B12 is not absorbed leading
to megaloblastic anaemia].
Ans. The key is E. Vascular dementia.
[hemiparesis, memory impairment, extensor
planter reflex, inability to identify objects, poor
judgment are features of vascular dementia].
251. A 7yo is brought by his mother who
says that he was well at birth but has
been suffering from repeated chest and GI
249. A nurse comes to you saying that she infections since then. She also says that he
has recently developed the habit of is not growing well for this age.
washing her hands after every 15-20 mins. What is the likely condition of this child?
a. CF
She is unable to conc on her work and
b. SCID
c. Primary Tcell immunodeficiency
d. Primary Bcell immunodeficiency
e. Malabsorption
260. A 57yo man presents with weight if associated with another congenital heart
defect amenable to surgery then prostaglandin E1
loss, tiredness, fever and abdominal
is useful to keep the duct open until after surgical
discomfort. Exam: spleen palpable up to
repair.
the umbilicus. Labs: WBC=127, Hgb=8.7,
Plt=138.
What is the most likely dx?
262. A 6yo girl who has previously been
a. CML
well presented with a hx of tonic-clonic
b. AML
c. CLL
seizures lasting 4mins. Her mother
brought her to the hospital and she
appeared well. She is a febrile and didn’t
lose consciousness during the episode of Q. 1. What is the key?
seizure. She has no neurologic deficit. Q. 2. What is the diagnosis?
What is the most appropriate inv for her? Q. 3. What are the points in favour of your
a. ABG diagnosis?
b. Serum electrolytes
c. ECG Ans. 1. The key is A. ECT.
d. Blood glucose
Ans. 2. The diagnosis is post purtum psychosis.
Q. 1. What is the key?
Q. 2. What are the points in favour? Ans. 3. Points in favour: i) onset 3 weeks after
childbirth ii) Depressive symptoms (very low
mood, insomnia) iii) thought of harming her little
baby.
Ans. 1. The key is B. Serum electrolyte. Electroconvulsive therapy (ECT) is a medical
Ans. 2. In epilepsy patient becomes unconscious. treatment most commonly used in patients with
The child has no previous illness and she was severe major depression or bipolar disorder that
conscious during the episode of seizure. So has not responded to other treatments. ECT
electrolyte imbalance may be the cause. involves a brief electrical stimulation of the brain
while the patient is under anesthesia.
Most of the health concerns associated with 266. A 29yo male brought to ED in
polycythemia vera are caused by the blood being unconscious state. There is no significant
thicker as a result of the increased red blood cells.
past hx. Which of the following should be
It is more common in the elderly and may be
done as the initial inv?
symptomatic or asymptomatic. Common signs
a. CT
and symptoms include itching (pruritus), and
b. Blood glucose
severe burning pain in the hands or feet that is
c. ABG
usually accompanied by a reddish or bluish
coloration of the skin. Patients with polycythemia d. MRI
vera are more likely to have gouty arthritis.. e. CBC
Physical exam findings are non-specific, but may
include enlarged liver or spleen, plethora, or
gouty nodules.. Ans. The key is B. Blood glucose.
Common findings include an elevated
hemoglobin level and hematocrit, reflecting the
increased number of red blood cells; the platelet 267. A 45yo woman comes with red,
count or white blood cell count may also be swollen and exudating ulcer on the nipple
increased. The erythrocyte sedimentation rate and areola of right breast with palpable
(ESR) is decreased due to an increase in zeta lump under the ulcer.
potential. Because polycythemia vera results What do you think is causing this skin
from an essential increase in erythrocyte condition?
production, patients have normal blood a. Inflammatory cells releasing cytokines
oxygenation and a low erythropoietin (EPO) level. b. Infiltration of the lymphatics by the
Phlebotomy is the treatment, which often may be carcinomatous cells
combined with other therapies. c. Infiltration of the malignant skin cells to
the breast tissue
Low dose aspirin (75–81 mg daily) is often
prescribed. Research has shown that aspirin
reduces the risk for various thrombotic
Ans. The key is B. Infiltration of the lymphatics by
complications.
the carcinomatous cells.
Bone marrow transplants are rarely undertaken
in people with polycythemia.
Chemotherapy for polycythemia may be used, 268. A 20yo young lady comes to the GP
either for maintenance, or when the rate of for advice regarding cervical ca. she is
bloodlettings required to maintain normal worried as her mother past away because
hematocrit is not acceptable, or when there is of this. She would like to know what is the
significant thrombocytosis or intractable pruritus. best method of contraception in her case?
a. POP
b. Barrier method would be the most appropriate inv for
c. IUCD her?
d. COCP a. Hysterectomy
e. IUS b. Endometrial biopsy
c. CBC
d. High vaginal swab
Ans. The key is A. POP(progestogen-only pill). e. Coagulation profile
Probably wrong key! Correct key should be B.
Barrier method! [spermatozoa itself acts as a Ans. The key is E. Coagulation profile.
carcinogen!!! So barrier method is the best [Transvaginal US is normal i.e. no endometrial
protection from the given option!!]. hyperplasia, no fibroid or obvious cause for heavy
bleeding was found. So now most appropriate
investigation should be coagfulation profile].
Dyspareunia is the term for recurring pain in the 276. A 23yo presents with vomiting,
genital area or within the pelvis during sexual nausea and dizziness. She says her
intercourse. menstrual period has been delayed 4
weeks as she was stressed recently. There
are no symptoms present. What is the
274. A 38yo woman, 10d postpartum next appropriate management?
presents to the GP with hx of passing a. Refer to OP psychiatry
blood clots per vagina since yesterday. b. Refer to OP ENT
Exam: BP=90/40mmhg, pulse=110bpm, c. CT brain
temp=38C, uterus tender on palpation and d. Dipstick for B-hCG
fundus 2cm above umbilicus, blood clots + e. MRI brain
++.
Q. 1. What is the key?
Choose the single most likely dx.
Q. 2. What is the likely diagnosis?
a. Abruption of placenta 2nd to pre-
Ans. 1. The key is D. Dipstick for B-hCG.
eclampsia
b. Concealed hemorrhage Ans. 2. Likely diagnosis is pregnancy. [Features
c. Primary PPH like vomiting, nausea and dizziness are consistent
d. Secondary PPH with early pregnancy supported by delayed
e. Retained placenta menstruation].
f. Scabies 277. A 16yo girl came to the sexual clinic.
Q. 1. What is the key? She complains of painful and heavy
Q. 2. How the condition is defined? bleeding. She says she doesn’t a regular
Ans. 1. The key is D. Secondary PPH. cycle. What is the most appropriate
Ans. 2. Secondary PPH: Secondary PPH is defined
management?
a. Mini pill
as abnormal or excessive bleeding from the birth
b. Combined pill
canal between 24 hours and 12 weeks
postnatally. [www.rcog.org.uk/en/guidelines- c. IUS
research-services/guidelines/gtg52/]. d. Anti-prostoglandins
e. Anti-fibrinolytics
Ans. The key is B. Combined pill. Patients may, for example, believe that certain
news bulletins have a direct reference to them,
278. A 36yo man walks into a bank and that music played on the radio is played for them,
demands money claiming he owns the or that car licence plates have a meaning relevant
bank. On being denied, he goes to the to them.
police station to report this.
What kind of delusions is he suffering
from?
a. Delusion of reference
b. Delusion of control
c. Delusion of guilt 279. Which method of contraception can
d. Delusion of persecution
cause the risk of ectopic pregnancy?
e. Delusion of grandeur
a. COCP
Ans. The key is E. Delusion of grandeur. b. IUCD
Delusions of grandeur are one of the more c. Mirena
common ones. It's when you believe that you d. POP
have more power, wealth, smarts, or other grand
Ans. The key is B. IUCD.
traits than is true.
280. A woman has pernicious anemia. She
has been prescribed parenteral vitamin
Delusion of control: False belief that another B12 tx but she is needle phobic. Why is
person, group of people, or external force
oral tx not preferred for this pt?
controls one's general thoughts, feelings,
a. IM B12 is absorbed more
impulses, or behavior.
b. Intrinsic factor deficiency affects oral B12
Delusion of guilt or sin (or delusion of self- utilization
accusation): Ungrounded feeling of remorse or c. IM B12 acts faster
guilt of delusional intensity. d. IM B12 needs lower dosage
e. Pernicious anemia has swallowing
Delusion of mind being read: False belief that
other people can know one's thoughts. difficulties
Delusion of thought insertion: Belief that another Ans. The key is B. Intrinsic factor deficiency
thinks through the mind of the person. affects oral B12 utilization.
308. What advice would you give for the Ans. 2. If the patient experiences any of the
parents of a child with repeated UTI? serious problems described below his disease has
a. Surgery progressed from compensated cirrhosis to
b. Prophylactic antibiotics decompensated cirrhosis:
c. Increase fluids i) Bleeding varices (internal bleeding)
d. Toilet training
e. Laxatives ii) Ascites (fluid in the belly)
Ans. 2. There are 2 reasons for this acute cortical Q. 3. What are the points in favour?
necrosis. i) significant diminished arterial
Ans. 1. The key is A. Give antibiotics.
perfusion of the kidneys due to spasm of the
feeding artery secondary to profuse bleeding Ans. 2. The diagnosis is epididymo-orchitis.
from placental abruption ii) DIC secondary to
Ans. 3. Points in favour: i) No history of trauma ii)
placental abruption.
testicular pain with fever points towards
310. An alcoholic 56yo man had ascetic epididymo-orchitis.
fluid analysis done which was found to be 312. A 58yo lady presented with urinary
yellow color. What is the most incontinence. She looks anxious for her
appropriate cause? condition. Urine culture is sterile. Her
a. Alcoholic hepatitis
b. Decompensated cirrhosis
urodynamic study is normal. What is the
next step? 316. A 28 yo female who delivered 6
2. Antibiotics weeks ago feels sad and has no interest to
3. Topical estrogen feeding the baby. She has been eating
4. Systemic estrogen poorly and having difficulty sleeping. She
5. Duloxetine
feels weak throughout the day and has
6. Pelvic floor exercise
stopped taking the baby out of the house.
Ans. The key is E. Pelvic floor exercise. She also says that the baby has evil eyes.
What is the most likely diagnosis?
313. A 45yo lady came to family planning a. Postpartum blues
clinic for contraception advice. She is not b. Postpaetum depression
keen to be pregnant for the next 3yrs. Her c. Postpurtum psychosis
recent US showed multiple small d. Schizophrenia
submucosal fibroid. What is the best e. Psychotic depression
method of contraception for her?
1. Etonogestrol Q. 1. What is the key?
2. COCP Q. 2. What are the points in favour?
3. IUS Ans. 1. The key is C. Postpartum psychosis.
4. POP
Ans. 2. Points in favour: i) features of depression:
5. IUCD
feels sad, poor eating, difficulty sleep, feeling
Ans. The key is C. IUS. [IUS gives 3-5 yrs long weak ii) delusional ideas: thinks baby has evil eyes
contraception. It also helps to shrink the fibroid]. and not taking the baby out of the house. These
points to postpartum psychosis. [Postpartum
314. A child presents with eczema. She psychosis starts within 2 wks (occasionally later)
was given two creams by the GP – of delivery and it can take 6 -12 months or more
emollient and steroid. What advice would to recover from postpartum psychosis].
you give her regarding application of the 317. A 44yo man presents with periorbital
cream? and pedal edema. 24h urine shows 8g of
a. Sparingly use both the cream
protein/d and serum
b. First use emollient, then steroid
cholesterol=7mmol/L. Renal biopsy results
c. Apply steroid then emollient
are awaited.
d. Mix emollient & steroid before use
What would be the most likely dx?
e. Emollient at night with steroid
a. Minimal change disease
Ans. The key is B. First use emollient, then a. b. Glomerulonephropathy
steroid. [emmolient 30 minutes before steroid].
b. Membranous glomerulonephropathy
c. FSGS
315. All the following drugs do not cause
d. IgA nephropathy
bronchoconstriction except?
e. Mesangiocapillary
1. Atenolol
2. Salbutamol
Ans. The given key is C. Membranous
3. Salmetrol
glomerulonephritis. [Some authority claims FSGS
4. Ipratropium bromide
5. Cocaine as more common cause of nephrotic syndrome].
Ans. Given key is E. relative risk = 2. [RR= Number Ans. The key is B. Reassure. [Smoking and alcohol
of death in not treated group/number of death in excess can cause palpitation without any
treated group]. recognizable arrhythmia and for this no
treatment is required].
320. A 67yo woman has presented with
hard, irregular, poorly defined 5cm lump 323. A 47yo man comes to the GP with a
in her right breast. She has a bruise on the swelling in his left groin which disappears
surface and there is no discharge. on lying down. The swelling was bluish in
What is the most likely dx? color and felt like a bag of worms. He also
a. Fibroadenosis complains of a mass in the left loin along
b. Fat necrosis with hematuria occasionally.
c. Fibroadenoma What could be the possible dx?
d. Duct ectasia a. Left sided RCC
b. Varicosity 2nd to liver disease d. Left main stem, post descending artery
c. Testicular tumor e. Right coronary artery
d. UTI
Q. 1. What is the key?
e. IVC obstruction
Q. 2. Justify the key.
Q. 1. What is the key?
Q. 2. What is the condition described? Ans. 1. The key is E. Right coronary artery.
Q. 3. What is the link between these two Ans. 2. If the posterior discending artery is
conditions?
supplied by the circumflex artery then it is left
dominant and if posterior descending artery is
Ans. 1. The key is A. Left sided Renal cell
supplied by the right coronary artery then it is
carcinoma.
right dominant. As in 85% of population posterior
Ans. 2. Left sided varicocele. descending artery is supplied by right coronary
artery it is called the dominant that is right
Ans. 3. Most common secondary cause of left
coronary artery is dominant.
sided varicocele is RCC. Newly diagnosed
varicocele over the age of 40yrs are very much
suggestive of RCC. Varicocele is common on left
side as left testicular veins drain to the left renal
vein, while the right testicular vein drain directly 326. A 54 yo lady presents with sudden,
into IVC. severe pain in the left half of her skull. She
324. A man presents with muffled hearing also complains of pain around her jaw.
and feeling of pressure in ear with tinnitus What is the next likely step?
a. CT
and vertigo. He also complains of double
b. MRI
vision when looking to the right. What is
c. Fundoscopy
the most appropriate dx?
a. Meniere’s disease d. ESR
b. Acoustic neuroma e. Temporal artery biopsy
c. Acute labyrinthytis Q. 1. What is the key?
d. Meningioma Q. 2. What is the diagnosis?
e. Otosclerosis Q. 3. What are the points in favour of your
diagnosis?
Q. 1. What is the key? Ans. 1. The key is ESR.
Q. 2. Justify the key.
Ans. 2. The diagnosis is Giant cell arteritis or
Ans. 1. The key is B. Acoustic neuroma. temporal arteritis.
Ans. 2. Hearing loss, feeling of pressure in the ear Ans. 3. Points in favour: i) Age >50yrs ii) Female
with tinnitus, vertigo and involvement of cranial sex iii) Severe pain in the left half of skull iv) Pain
nerve i.e. right abducent nerve are suggestive of around the jaw (jaw claudication).
acoustic neuroma.
327. A teenage girl who was ‘fine’ until
325. In 85% of the population this artery is her boyfriend said he didn’t want the
dominant. What is the single most relationship anymore. She took 10 tablets
appropriate option? of paracetamol in front of his mother after
a. Left ant descending artery taking alcohol. What should you do?
b. Coronary sinus a. Refer to psychiatry
c. Circumflex artery b. Counselling
c. GP to sort out family issues 331. An old lady 72yo staying at a nursing
d. Return to work to relieve her anger home for a few years, a known HTN on
reg tx presented with sudden dysphagia
Ans. The key is A. Refer to psychiatry. [1o tablets
of paracetamol is not a life threatening toxic dose
while eating with drooling of saliva and
and simultaneous drug overdose and alcohol req urgent inv. What would be your next
consumption needs psychiatric evaluation]. step?
a. Ba swallow
328. A 6yo fell on outstretched hand while b. Chest CT
playing. He feels tender at the elbow but c. Endoscopy
otherwise well. What is the most likely dx? d. Laryngoscopy
a. Spiral fx e. CXR
b. Green stick fx f. Endoscopy with biopsy
c. Compound fx
d. Supracondylar fx Ans. The key is C. Endoscopy. [Probable
e. Pulled elbow impacted food bolus (usually meat)which can be
visualized and removed with the aid of
Ans. The key is B. Green stick fracture. endoscopy].
Ans. The key is B. Intubation and ventilation [ABC Q. 1. What is the key?
protocol]. Q. 2. What is the diagnosis?
Ans. 1. The key is A. Gentamycin topical.
356. A 1m boy has been brought to the 358. A 35yo construction worker is dx with
ED, conscious but with cool peripheries indirect inguinal hernia. Which statement
and has HR=222bpm. He has been below best describes it?
a. Passes through the superficial inguinal
irritable and feeding poorly for 24h.
ring only
CXR=borderline enlarged heart with clear
b. Lies above and lateral to the pubic
lung fields. ECG=regular narrow complex
tubercle
c. Does not pass through the superficial consolidation in the left lung. What is the
inguinal ring next appropriate step?
d. Passes through the deep inguinal ring a. PPI IV
b. Alendronate
Ans. The key is D. Passess through the deep
c. IV antibiotics
inguinal ring.
d. Analgesic
359. A woman has numerous painful e. PPI PO
ulcers on her vulva. What is the cause?
Ans. Here is two key C. IV antibiotics and E. PPI
1. Chlamydia
2. Trichomonas PO. Correct key is C. IV antibiotics. [Pneumonia
3. Gardenella should be treated first].
4. HSV
363. A 66yo man has the following ECG.
5. EBV
Ans. The key is D. HSV. What is the most appropriate next step in
management?
360. A 72 yo man has been on warfarin
for 2yrs because of past TIA and stroke.
What is the most important complication
that we should be careful with?
a. Headache
b. Osteoporosis
c. Ear infection
d. Limb ischemia
e. Diarrhea a. Metoprolol
b. Digoxin
Ans. The given key is E. Diarrhoea which is
considered as a wrong key and A. Headache is the c. Carotid sinus massage
correct key. [Headache is the warning sign of d. Adenosine
hemorrhagic stroke]. e. Amiodarone.
361. A 55yo man has been admitted for Ans. The key is A. Metoprolol. [P waves are
elective herniorraphy. Which among the replaced by fibrillatory f-waves. Irregular R-R
following can be the reason to delay his intervals. Dx atrial fibrillation].
surgery? 364. A 22yo sexually active male came
a. Controlled asthma
with 2d hx of fever with pain in scrotal
b. Controlled atrial fib
area. Exam: scrotal skin is red and tender.
c. DVT 2yrs ago
What is the most appropriate dx?
d. Diastolic BP 90mmHg a. Torsion of testis
e. MI 2 months ago b. Orchitis
Ans. The key is E. MI 2 months ago [better go for c. Inguinal hernia
surgery 6 months post MI]. d. Epididymo-orchitis
Ans. 1. The key is B. Normal finding. 382. A 50 yo woman who was treated for
Ans. 2. Stress during operation can cause breast cancer 3 yrs ago now presents with
transient hyperglycemia causing glycosuria increase thirst and confusion. She has
secondary to stress induced rise of cortisole become drowsy now.
which becomes normal after some time. What is the most likely metabolic
abnormality?
a. Hypercalcemia
380. A pt presented with hx of swelling in b. Hyperkalemia
the region of the sub-mandibular region, c. Hypoglycemia
d. Hyperglycemia swelling in the stoma. What is the
e. Hypocalcemia management option for her?
a. Local exploration
b. Exploratory laparotomy
Q. 1. What is the key?
Q. 2. Justify the key c. Open laparotomy
d. Reassure
Ans. 3. At potassium level of >5.5mEq/L occurs 389. A 60yo man presented with a lump in
tall tented T waves and at potassium level the left supraclavicular region. His
>7mEq/L occurs wide QRS complex with bizarre appetite is decreased and he has lost 5kg
QRS morphology.
recently. What is the most probably dx?
387. A 54yo pt 7 days after a total hip a. Thyroid carcinoma
replacement presents with acute onset b. Stomach carcinoma
breathlessness and raised JVP. Which of c. Bronchial carcinoma
d. Mesothelioma
the following inv will be most helpful in
e. Laryngeal carcinoma
leading to a dx?
a. CXR Ans. The key is B. Stomach carcinoma.
b. CTPA [Mentioned lump in the left supraclavicular
c. V/Q scan region is Vershow’s gland, has long been
d. D-Dimer regarded as strongly indicative of the presence
e. Doppler US of legs of cancer in the abdomen, specifically gastric
cancer].
Q. 1. What is the key?
Q. 2. Justify the key. 390. A 64yo man has presented to the ED
Ans. 1. The key is B. CTPA. with a stroke. CT shows no hemorrhage.
Ans. 2. The patient has a +ve two level PE Wells ECG shows atrial fib. He has been
score (if it was negative we should do D-Dimer) thrombolysed and he’s awaiting
and there is no renal impairment or history discharge.
suggestive of allergy to contrast media (if these What prophylactic regimen is best for
present we should have go for VQ scan) the him?
investigation of choice is PTCA. NICE guideline. a. Warfarin
b. Heparin
388. A 7yo girl has been treated with
c. Aspirin
penicillin after sore throat, fever and
d. Statins
cough. Then she develops skin rash and
e. Beta blockers
itching. What is the most probable dx?
a. Erythema nodosum Ans. The key is A. Warfarine. [Atrial fibrillation:
b. Erythema multiforme post stroke- following a stroke or TIA warfarine
c. SJS should be given as the anticoagulant of choice.
d. Erythema marginatum NICE guideline].
e. Erythema gangernosum
393. A 21yo girl looking unkempt, 396. A 56 yo lady with lung cancer
agitated, malnourished and nervous came presents with urinary retention, postural
to the hospital asking for painkillers for hypotension, diminished reflexes and
her abdominal pain. She is sweating, sluggish pupillary reaction.
shivering and complains of joint pain. What is the most likely explanation for her
What can be the substance misuse here? symptoms?
a. Alcohol a. Paraneoplastic syndrome
b. Heroin b. Progression of lung cancer
c. Cocaine c. Brain metastasis
d. LSD d. Hyponatremia
e. Ecstasy e. Spinal cord compression
Ans. The key is B. Heroin. [agitation, nervousness, Ans. The key is A. Paraneoplastic syndrome.
abdominal cramp, sweating, shivering and [Features given are well known features of
autonomic neuropathy which can be a result of on palpation and only one testis was
paraneoplastic syndrome]. normal in size and position.
397. An old woman having decreased What would be your next step?
a. Analgesia
vision can’t see properly at night. She has
b. Antibiotics
changed her glasses quite a few times but
c. Refer urgently to a surgeon
to no effect. She has normal pupil and
d. Reassurance
cornea.
e. Discharge with analgesics
What is the most likely dx?
a. Cataract Ans. The key is A. Analgesia. [According to some
b. Glaucoma US sites it is analgesia but no UK site support
c. Retinal detachment this!!! So for Plab exam the more acceptable
d. Iritis option is C. Refer urgently to a surgeon].
e. GCA
400. A child suffering from asthma
Ans. The key is B. Glaucoma. It is a wrong key. presents with Temp 39C, drooling saliva
Correct key should be A. Cataract. [Age and on to the mother’s lap, and taking oxygen
normal pupil and cornea are suggestive of by mask. What sign will indicate that he is
cataract. If it was glaucoma pupil would be a bit deteriorating?
dilated and/or oval in shape]. a. Intercostal recession
b. Diffuse wheeze
398. A pt comes with sudden loss of vision.
c. Drowsiness
On fundoscopy the optic disc is normal.
What is the underlying pathology? Ans. The key is A. Intercostal recession. This is
a. Iritis wrong key. Correct key is C. Drowsiness.
b. Glaucoma [Intercostal recession is a sign of severe asthma
c. Vitreous chamber but it can be seen at a lesser degree as well. So
d. Retinal detachment drowsiness is more appropriate answer].
412. A 45yo man who is diabetic and HTN 414. A 70yo man had a right
but poorly compliant has chronic SOB, hemicolectomy for ceacal carcinoma
develops severe SOB and chest pain. Pain 6days ago. He now has abdominal
is sharp, increased by breathing and distension and recurrent vomiting. He has
relieved by sitting forward. What is the not opened his bowels since surgery.
single most appropriate dx? There are no bowel sounds. WBC=9,
a. MI Temp=37.3C.
b. Pericarditis What is the single most appropriate next
c. Lung cancer management?
d. Good pastures syndrome a. Antibiotic therapy IV
e. Progressive massive fibrosis b. Glycerine suppository
c. Laparotomy
d. NG tube suction and IV fluids
Ans. The key is B. Pericarditis. [Nature of pain i.e. e. TPN
sharp pain increased by breathing and relieved by
sitting forward is suggestive of pericarditis]. Ans. The key is D. NG tube suction and IV fluids.
[The patient has developed paralytic ileus which
Nature of pericardial pain: the most common should be treated conservatively].
symptom is sharp, stabbing chest pain behind the
sternum or in the left side of your chest. 415. A 60yo man with a 4y hx of thirst,
However, some people with acute pericarditis urinary freq and weight loss presents with
describe their chest pain as dull, achy or pressure- a deep painless ulcer on the heel. What is
like instead, and of varying intensity. the most appropriate inv?
The pain of acute pericarditis may radiate to your a. Ateriography
left shoulder and neck. It often intensifies when b. Venography
you cough, lie down or inhale deeply. Sitting up c. Blood sugar
and leaning forward can often ease the pain. d. Biopsy for malignant melanoma
e. Biopsy for pyoderma
413. A 6m boy has been brought to ED
following an apneic episode at home. He Ans. The key is C. Blood sugar. [The patient
is now completely well but his parents are probably developed diabetic foot].
anxious as his cousin died of SIDS (Sudden 416. A 16yo boy presents with rash on his
Infant Death Syndrome) at a similar age. buttocks and extensor surface following a
The parents ask for guidance on BLS for a
sore throat. What is the most probable management for type A dissection is surgery and
dx? for type B is conservative].
a. Measles 419. A 54yo myopic develops flashes of
b. Bullous-pemphigoig light and then sudden loss of vision. That
c. Rubella is the single most appropriate tx?
d. ITP a. Pan retinal photo coagulation
e. HSP b. Peripheral iridectomy
c. Scleral buckling
Ans. The key is D. ITP. It’s probably a wrong key! d. Spectacles
The correct key should be E. HSP. [In HSP rash e. Surgical extraction of lens
typically found in buttocks, legs and feets and
may also appear on the arms, face and trunk. But Ans. The key is C. Scleral buckling. [It is a case of
in ITP it mostly occurs in lower legs. HSP usually retinal detachment with treatment option of
follow a sorethroat and ITP follow viral infection scleral buckling].
like flue or URTI. HSP is a vasculitis while ITP is
deficiency of platelets from more destruction in 420. A 40yo chronic alcoholic who lives
spleen which is immune mediated]. alone, brought in the ED having been
found confused at home after a fall. He
417. A 34yo man with a white patch on complains of a headache and gradually
the margin of the mid-third of the tongue. worsening confusion. What is the most
Which is the single most appropriate LN likely dx?
involved? a. Head injury
a. External iliac LN b. Hypoglycemia
b. Pre-aortic LN c. Extradural hematoma
c. Aortic LN
d. Subdural hematoma
d. Inguinal LN
e. Iliac LN e. Delirium
f. Submental LN
Ans. The key is D. Subdural hematoma. [subdural
g. Submandibular LN
h. Deep cervical LN hematoma may be acute or chronic. In chronic
symptoms may not be apparent for several days
Ans. The key is G. Submandibular LN. or weeks. Symptoms of subdural hematomas are:
fluctuating level of consciousness, ± insidious
418. A 50yo lady presents to ED with physical or intellectual slowing, sleepiness,
sudden severe chest pain radiating to both headache, personality change and unsteadiness.
shoulder and accompanying SOB. Exam: Tx. Irrigation/evacuation e.g. via barr twist drill
cold peripheries and paraparesis. and barr hole craniostomy 1st line. Craniotomy if
What is the single most appropriate dx? the clot organized 2nd line].
a. MI
421. A 54yo man with alcohol
b. Aortic dissection
dependence has tremor and sweating
c. Pulmonary embolism
3days into a hosp admission for a fx
d. Good pastures syndrome
femur. He is apprehensive and fearful.
e. Motor neuron disease
What is the single most appropriate tx?
Ans. The key is B. Aortic dissection. [Cold a. Acamprossate
peripheries due to reduced blood flow to dista b. Chlordiazepoxide
parts of dissection and reduced perfusion of c. Lorazepam
nerves resulted in paraparesis. Usual d. Lofexidine
e. Procyclidine muscles tightness on 3 occasions in the
last 6 wks. He doesn’t complain of
Ans. The key is B. Chlordiazepoxide. C. Lorazepam
is also correct key! [According to NICE 1 st line
headache and his BP is WNL.
treatment is oral lorazepam and if symptom What is the single most appropriate long-
persists or oral medication is declined, give term tx for him?
parenteral lorazepam, haloperidol or olanzapine. a. Diazepam
According to OHCM 1st line treatment is b. Olanzapine
chlordiazepoxide. c. Haloperidol
d. Fluoxetine
Chlordiazepoxide should only be used at the
e. Alprazolam
lowest possible dose and for a maximum of up to
four weeks. This will reduce the risks of Ans. The key is D. Fluoxetine. [Recommended
developing tolerance, dependence and treatment for panic disorder is i) CBT ii)
withdrawal]. Medication (SSRIs or TCA). NICE recommends a
total of seven to 14 hours of CBT to be completed
within a four month period. Treatment will
422. A 5yo child complains of sore throat usually involve having a weekly one to two hour
and earache. He is pyrexial. Exam: tonsils session. When drug is prescribed usually a SSRI is
enlarged and hyperemic, exudes pus when preferred. Antidepressants can take two to four
pressed upon. weeks before becoming effective].
What is the single most relevant dx?
a. IM
b. Acute follicular tonsillitis 424. A 28yo man presents with rapid
c. Scarlet fever pounding in the chest. He is completely
d. Agranulocytosis conscious throughout. The ECG was taken
e. Acute OM (SVT). What is the 1st med to be used to
manage this condition?
Ans. The key is B. Acute follicular tonsillitis. a. Amiodarone
[Tonsillitis is usually caused by a viral infection b. Adenosine
or, less commonly, a bacterial infection. The c. Lidocaine
given case is a bacterial tonsillitis (probably d. Verapamil
caused by group A streptococcus). There are four
e. Metoprolol
main signs that tonsillitis is caused by a bacterial
infection rather than a viral infection. They are: Ans. The key is B. Adenosine. [Management of
SVT: i) vagal manoeuvres (carotid sinus message,
a high temperature
white pus-filled spots on the tonsils valsalva manoeuvre) transiently increase AV-
no cough block, and unmask the underlying atrial rhythm. If
swollen and tender lymph nodes (glands). unsuccessful then the first medicine used in SVT is
adenosine, which causes transient AV block and
works by i) transiently slowing ventricles to show
the underlying atrial rhythm ii) cardioverting a
junctional tachycardia to sinus rhythm. OHCM].
427. A 63yo lady with a BMI=32 comes to 430. A 45yo woman complains of pain in
the ED with complaints of pigmentation her hands precipitated by exposure to the
on her legs. Exam: dilated veins could be cold weather. She has breathlessness on
seen on the lateral side of her ankle. walking. When she is eating, she can feel
Which of the following is involved? food suddenly sticking to the gullet. It
a. Short saphenous vein seems to be in the middle of the
esophagus but she can’t localize exactly Ans. The key is D. ITP. [As the patient is otherwise
where it sticks. It is usually relieved with a well acute leukemia, HIV and SLE is unlikely.
drink of water. Normal wbc count excludes aplastic anemia. So
likely diagnosis is ITP].
Choose the single most likely cause of
dysphagia from the options?
a. Esophageal carcinoma
b. Systemic sclerosis 433. A 30yo man complains of episodes of
c. SLE hearing music and sometimes threatening
d. Pharyngeal carcinoma voices within a couple of hours of heavy
e. Globus hystericus drinking. What is the most likely dx?
a. Delirium tremens
Ans. The key is B. Systemic sclerosis. [Raynods b. Wernicke’s encephalopathy
phenomena, pulmonary involvement, c. Korsakoff’s psychosis
oesophageal dysmotility are suggestive of d. Alcohol hallucinosis
systemic sclerosis]. e. Temporal lobe dysfunction
Ans. The key is D. Inguinal LN. Ans. The key is A. Head roll test. [this is a case of
“benign paroxysmal positional vertigo” for which
the diagnosis is made by head roll test].
448. A 76yo woman has become tired and 451. A 65yo woman says she died 3m ago
confused following an influenza like and is very distressed that nobody has
illness. She is also breathless with signs of buried her. When she is outdoors, she
consolidation of the left lung base. hears people say that she is evil and needs
What is the most likely dx? to be punished. What is the most likely
a. Drug toxicity
explanation for her symptoms?
b. Delirium tremens
a. Schizophrenia
c. Infection toxicity
b. Mania
d. Hypoglycemia
c. Psychotic depression
e. Electrolyte imbalance
d. Hysteria
A. The key is C. Infection toxicity. [Infection e. Toxic confusional state
toxicity or toxic shock syndrome is likely here as
Ans. The key is C. Psychotic depression. [Psychotic
preceding flue like illness points towards toxin
depression, also known as depressive psychosis,
(enterotoxin type B) from Staphylococcus
is a major depressive episode that is accompanied
aureus].
by psychotic symptoms (hallucinations,
delusions). In this patient nihilistic delusion Ans. The key is C. 4 x weight(kgs) x area of burn =
favours the diagnosis of psychotic depression. It ml of fluids.
can occur in the context of bipolar disorder or
majordepressive disorder].
454. A 65yo male presents with dyspnea
and palpitations. Exam: pulse=170bpm,
452. A 50yo woman presents following a BP=120/80mmHg. Carotid massage has
fall. She reports pain and weakness in her been done as first instance.
hands for several months, stiff legs, What is the next step of the
swallowing difficulties, and has bilateral management?
wasting of the small muscles of her hands. a. Adenosine
Reflexes in the upper limbs are absent. b. Amlodipine
Tongue fasciculations are present and c. DC cardioversion
both legs show increased tone, pyramidal d. Lidocaine
weakness and hyper-reflexia with e. Beta blocker
extensor plantars. Pain and temp
Ans. The key is A. Adenosine. [The likely diagnosis
sensation are impaired in the upper limbs.
is SVT. 1st vagal manoeuvres, if fails iv adenosine.
What is the most likely dx?
a. MS • Vagal manoeuvres (carotid sinus massage,
b. MND Valsalva manoeuvre) transiently increase
c. Syringobulbia
AV block, and may unmask an underlying atrial
d. Syringomyelia rhythm.
e. Myasthenia gravis
• If unsuccessful, give adenosine, which causes
Ans. The key is C. Syringobulbia. [In MS there are transient AV block; OHCM, 9th edition].
characteristic relapse and remission which is
absent here; In MND there is no sensory deficit;
Syringomyelia doesn’t cause cranial nerve lesion 455. A 48yo farmer presented with fever,
and in myasthenia there is muscular weakness
malaise, cough and SOB. Exam:
without atrophy. Here the features described well
tachypnea, coarse endinspiratory crackles
fits with syringobulbia].
and wheeze throughout, cyanosis. Also
complaint severe weight loss. His CXR
453. Which of the following formulas is shows fluffy nodular shadowing and there
is PMN leukocytosis. What is the single
used for calculating fluids for burn pts?
most appropriate dx?
a. 4 x weight(lbs) x area of burn = ml of a. Ankylosing spondylitis
fluids b. Churg-strauss syndrome
b. 4 x weight(kgs) x area of burn = L of fluids c. Cryptogenic organizing
c. 4 x weight(kgs) x area of burn = ml of d. Extrinsic allergic alveolitis
fluids e. Progressive massive fibrosis
d. 4 x weight(lbs) x area of burn = L of fluids
e. 4.5 x weight(kgs) x area of burn = dL of
fluids Ans. The key is D. Extrinsic allergic alveolitis.
456. A 35yo lady is admitted with pyrexia, Ans. The key is D. Vancomycin.
weight loss, diarrhea and her skin is lemon [Pseudomembranous colitis is treated with
yellow in color. CBC = high MCV. What is metronidazole or vancomycin].
the most probably dx?
a. Aplastic anemia
b. Pernicious anemia 459. A 56yo man has symptoms of sleep
c. Leukemia apnea and daytime headaches and
d. ITP somnolence. Spirometry shows a
e. Lymphoma decreased tidal volume and vital capacity.
What is the single most appropriate dx?
Ans. The key is B. Pernicious anemia. [It may be a. Ankylosing spondylitis
graves with pernicious anemia. Lemon yellow b. Churg-strauss syndrome
pallor occurs in pernicious anemia. c. Good pasture syndrome
Hyperthyroidism may cause persistently raised d. Motor neuron disease
body temperature. Both are autoimmune disease
e. Progressive massive fibrosis
which favours this association].
f. Spinal cord compression
458. An old lady had UTI and was treated 461. A 55yo woman was found collapsed
with antibiotics. She then developed at home, paramedics revived her but in
diarrhea. What is the single most likely tx? the ambulance she had a cardiac arrest
a. Co-amoxiclav and couldn’t be saved. The paramedic’s
b. Piperacillin + tazobactam report tells that the woman was immobile
c. Ceftriaxone lately due to hip pain and that they found
d. Vancomycin
ulcers on the medial side of ankle. She had
DM and was on anti-diabetics.
What is the cause of her death? 464. A 30yo female attends OPD with a
a. Acute MI fever and dry cough. She says that she
b. DKA had headache, myalgia and joint pain like
c. Pulmonary embolism one week ago. Exam: pulse=100bpm,
d. Acute pericarditis temp=37.5C. CXR: bilateral patchy
e. Cardiac tamponade consolidation.
Ans. The key is C. Pulmonary embolism.
What is the single most likely causative
[Immobilization due to hip pain may resulted in organism?
DVT and later pulmonary embolism]. a. Pneumococcal pneumonia
b. Legionella
c. Mycoplasma
d. Klebsiella
462. An 18yo previously well student is in
e. Chlamydia pneumonia
his 1st year at uni. He has been brought to
the ED in an agitated, deluded and Ans. The key is C. Mycoplasma. [mycoplasma give
disoriented state. What is the most flu-like symptoms and has a gradual onset plus on
probable reason for his condition? cxr shows bilateral consolidation].
a. Drug toxicity
b. Delirium tremens
c. Infection toxicity
465. A 46yo man is being investigated for
d. Electrolyte imbalance
indigestion. Jejunal biopsy shows
e. Head injury
deposition of
macrophages containing PAS (Periodic
Ans. The key is A. Drug toxicity. [Young age and 1 st acid-schiff) +ve granules. What is the most
yr in university is likely to point towards drug likely dx?
toxicity]. a. Bacterial overgrowth
b. Celiac disease
c. Tropical sprue
463. A young adult presents to the ED d. Whipple’s disease
after a motorcycle crash. The pt has e. Small bowel lymphoma
bruises around the left orbital area.
Ans. The key is D. Whipple’s disease. [periodic
GCS=13, examination notes alcoholic
acid-schiff +ve granules containing macrophages
breath. Shortly afterwards, his GCS drops
in jejuna biopsy is diagnostic of whipples disease].
to 7.
What is the single most important initial
assessment test? 466. A 32yo woman of 38wks gestation
a. MRI brain complains of feeling unwell with fever,
b. CT brain rigors and abdominal pains. The pain was
c. CXR initially located in the abdomen and was
d. CT angio brain a/w urinary freq and dysuria. The pain has
e. Head XR now become more generalized specifically
radiating to the right loin. She says that
Ans. The key is B. CT brain. [Likely cause is
she has felt occasional uterine tightening.
epidural hematoma].
CTG is reassuring. Select the most likely Ans. The key is A. Alcohol consumption.
dx? [abstinence from alcohol in the hospital lead to
a. Acute fatty liver of pregnancy delirium tremens].
b. Acute pyelonephritis
c. Roung ligament stretching
d. Cholecystitis 469. A 10yo girl presents with pallor and
e. UTI features of renal failure. She has
hematuria as well as proteinuria. The
Ans. The key is B. Acute pyelonephritis. [Fever, serum urea and creat are elevated. These
rigor, abdominal pain a/w frequency and dysurea
symptoms started after an episode of
and radiation to the rt loin suggests rt sided
bloody diarrhea 4days ago.
pyelonephritis].
What is the most probable dx?
a. TTP
467. A 32yo pt presents with cervical b. HUS
lymphadenopathy and splenomegaly. c. ITP
What is the single most appropriate d. HSP
option? e. ARF
a. Hemophilus
b. Streptococcus Ans. The key is B. HUS. [Most cases of hemolytic
uremic syndrome develop in children after two to
c. Toxoplasmosis
14 days of diarrhea often bloody, due to infection
d. NHL
with a certain strain of E. coli. Features may be i)
e. Pneumocystis jerovcii
abdominal pain, ii) pale skin, iii) hematuria and
Ans. The key is D. NHL. [ Here only two points are proteinuria, iv) features of renal failure like-
mentioned- cervical lymphadenopathy and nausea/vomiting, swelling of face, hand, feet or
splenomegaly! This combination makes NHL as entire body etc. v) elevated urea and creatinine
the most likely cause thouh splenomegaly is a etc.].
relatively uncommon feature of it!!! This
combination does not fit in other options!]
470. A 40yo woman has had intermittent
tension, dizziness and anxiety for
468. A 62yo man who was admitted for 4months. Each episode usually resolves
surgery 3days ago suddenly becomes after a few hours. She said she takes
confused. His attn span is reduced. He is alcohol to make her calm. She is in a
restless and physically aggressive and loving relationship and has no probs at
picks at his bed sheets. What single aspect work or home.
of the pt’s hx recovered in his notes is What is the next step in her
most likely to aid in making the dx? management?
a. Alcohol consumption a. Collateral info
b. Head trauma b. CT b0rain
c. Hx of anxiety c. CBC
d. Prescribed med d. LFT
e. Obvious cognitive impairment e. TFT
471. A 45yo IV drug abuser is brought into Ans. The key is B. Trochlear nerve. [oculomotor
the ED with complaint of fever, shivering, may cause palsy of inferior rectus, medial rectus
malaise, SOB and productive cough. and superior rectus causing double vision in
Exam: temp=39C, pulse=110bpm, multiple gaze! But trochlear involving superior
BP=100/70mmHg. Inv: CXR=bilateral oblique only causes diplopia in downgaze only. So
cavitating bronchopneumonia. the answer is Trochlear nerve].
What is the single most likely causative
organism?
a. Mycoplasma 474. L1 level, what is the most
b. Staphylococcus appropriate landmark?
c. Chlamydia pneumonia
a. Mcburney’s point
d. Pseudomonas
b. Stellate ganglion
e. PCP
c. Deep inguinal ring
Ans. The key is B. Staphylococcus. [Among the d. Termination of the spinal cord
given causes Staphylococcus and PCP are e. Transpyloric plane
recognized cause of cavitating pneumonia. This
Ans. The given key is D. Which is a wrong key. The
case is with productive cough which goes more
correct key is E. Transpyloric plane. [
with staphylococcus as PCP is not productive but
The termination of the spinal cord is between L1
rather associated with dry cough. Drug abuse can
and L2 (variable between people). L1 landmark-
support both staphylococcus and PCP].
duodenum first part, superior mesenteric artery,
hila of both kidneys, upper border of pancreas,
splenic artery, pylorus and fundus of gall
472. A 71yo woman looks disheveled, bladder].
unkempt and sad with poor eye contact.
She has recently lost her husband. Which
of the following describes her condition? 475. A 32yo woman presents to the ED
a. Anxiety
with headache and vomiting. She was
b. Hallucination
decorating her ceiling that morning when
c. Mania
the headache began, felt mainly occipital
d. High mood
with neck pain. Some 2hs later she felt
e. Low mood
nauseated, vomited and was unable to
Ans. The key is E. Low mood. walk. She also noticed that her voice had
altered. She takes no reg meds and has no
significant PMH. Exam: acuity, field and
473. A 62yo male comes to the GP fundi are normal. She has
complaining of double vision while upbeat nystagmus in all directions of gaze
climbing downstairs. Which of the with normal facial muscles and tongue
following nerve is most likely involved? movements. Her uvulas deviated to the
a. Abducens nerve right and her speech is slurred. Limb
b. Trochlear nerve exam: left arm past-pointing and
c. Oculomotor nerve dysdiadochokinesis with reduced pin prick
sensation in her right arm and leg. 477. A 55yo man has a chronic cough and
Although power is normal, she can’t walk sputum, night sweats and weight loss.
as she feels too unsteady. What is the single most likely causative
Where is the most likely site of lesion? organism?
a. Right medial medulla a. Coagulase +ve cocci in sputum
b. Left medial pons b. Gram -ve diplococci in sputum
c. Left cerebellar hemisphere c. Gram +ve diplococci in sputum
d. Right lateral medulla d. Pneumocystis carinii in sputum
e. Left lateral medulla e. Sputum staining for mycobacterium
tuberculosis
Ans. The key is Left lateral medulla. [ There is a
loss of pain and temperature sensation on the Ans. The key is E. Sputum staining for
contralateral (opposite) side of the body mycobacterium tuberculosis. [Chronic cough and
and ipsilateral (same) side of the face. There is sputum, night sweats and weight loss are classic
associated cerebellar symptoms and other cranial features of tuberculosis].
nerve involvement. The condition is known as
Lateral Medullary Syndrome].
478. A 20yo pregnant 32wks by date
presents to the antenatal clinic with hx of
476. A 28yo female presents with 1 wk hx painless vaginal bleeding after
of jaundice and 2d hx of altered sleep intercourse. Exam: P/A – soft and relaxed,
pattern and moods. She was dx with uterus=dates, CTG=reactive. Choose the
hypothyroidism for which she is receiving single most likely dx?
thyroxine. TFT showed increased TSH. a. Abruption of placenta 2nd to pre-
PT=70s. What is the most probable dx? eclampsia
a. Acute on chronic liver failure b. Antepartum hemorrhage
b. Hyper-acute liver failure c. Placenta previa
c. Autoimmune hepatitis d. Preterm labor
d. Acute liver failure e. Placenta percreta
e. Drug induced hepatitis
Ans. The key is C. Placenta previa.
Ans. The key is C. Autoimmune hepatitis.
[Autoimmune hepatitis may present as acute
hepatitis, chronic hepatitis, or well-established 479. A 30yo man presents to the ED with
cirrhosis. Autoimmune hepatitis rarely presents
difficulty breathing. He has returned from
as fulminant hepatic failure. One third may
India. Exam: throat reveals grey
present as acute hepatitis marked by fever,
hepatic tenderness and jaundice. Non specific membranes on the tonsils and uvula. He
features are anorexia, weight loss and has mild pyrexia. What is the single most
behavioural change (here altered sleep pattern relevant dx?
and moods). There may be coagulopathy (here a. Diphtheria
PT=70s.) leading to epistaxis, gum bleeding etc. b. IM
Presence of other autoimmune disease like c. Acute follicular tonsillitis
hypothyroidism supports the diagnosis of d. Scarlet fever
autoimmune hepatitis]. e. Agranulocytosis
Ans. The key is A. Diphtheria. [history of travel to 482. A 25yo woman presents with urinary
india, grey membrane in tonsil and uvula, low freq, dysuria and fever. Urine microscopy
grade fever, and dyspnoea support the diagnosis shows 20-50 RBC and 10-20 WBC in each
of diphtheria].
field. What is the most probable dx?
a. Schistosmiasis
b. Kidney trauma
480. A 23yo man comes to the ED with a c. Ureteric calculus
hx of drug misuse. He recognizes that he d. Bladder calculi
has a prb and is willing to see a e. Cystitis
psychiatrist. Which of the following terms
best describes this situation? Ans. The key is E. Cystitis. [Hematuria and
a. Judgement significant WBC in urine (>10 per HPF) makes
b. Thought insertion cystitis the most likely diagnosis].
c. Thought block
d. Mood
483. A 65yo presents with dyspareunia
e. Insight
after sex. She in menopause. She
Ans. The key is E. Insight. complains of bleeding after sex. What is
[in psychiatry, the patient's awareness and under the most probably dx?
standing of the origins and meaning of his a. Cervical ca
attitudes, feelings, and behavior and of his disturb b. Endometrial ca
ing symptoms (self-understanding) is known as c. Ovarian ca
insight]. d. Breast ca
e. Vaginal ca
Ans. The key is cholecystitis. [Fat, female, fare, 492. A 40yo woman presents to the GP
forty and fertile are the pneumonic for with low mood. Of note, she has an
cholecystitis! Here the presentation of severe increased appetite and has gone up 2
upper abdominal pain with fever along with the dress sizes. She also complains that she
pneumonic features points towards the diagnosis can’t get out of bed until the afternoon.
of cholecystitis]. What is the most likely dx?
a. Pseudo depression
b. Moderate depression
490. A child has just recovered from c. Severe depression
meningitis. What inv will you do before d. Dysthymia
discharge? e. Atypical depression
a. CT scan
b. EEG Ans. The key is E. Atypical depression. [Atypical
c. Blood culture depression is a subtype of major
d. Repeat LP depression or dysthymic disorder that involves
e. Hearing test several specific symptoms, including increased
appetite or weight gain, sleepiness or
Ans. The key is E. Hearing test. [deafness is a excessive sleep, marked fatigue or weakness,
common complication of meningitis, so hearing moods that are strongly reactive to
test is suggested before discharge]. environmental circumstances, and feeling
extremely sensitive to rejection].
515. Tip of the 9th costal cartilage. What 518. A 70yo woman presents with
is the single most appropriate landmark? recurrent episodes of parotid swelling. She
complains of difficulty in talking and
a. Fundus of the gallbladder
speaking and her eyes feel gritty on
b. Deep inguinal ring
waking in the morning. What is the single
c. Termination of the spinal cord
most likely dx?
d. Transpyloric plane
a. C1 esterase deficiency
e. Vena cava opening in the diaphragm
b. Crohns disease
Ans. The key is A. Fundus of the gallbladder. c. Mumps
d. Sarcoidosis
516. A child complains of RIF pain and e. Sjogrens syndrome
diarrhea. On colonoscopy, granular
transmural ulcers are seen near the ileo- Ans. The key is E. Sjogrens syndrome. [parotid
cecal junction. What should be the swelling, difficulty talking and speaking (due to
management? dryness or less salive), eyes feeling gritty on
a. Sulfasalazine
waking in the morning due to dryness of eye are 521 A 22yo man has rushed into the ED
suggestive of Sjogrens syndrome]. asking for help. He describes recurrent
519. A 39yo woman has not had her episodes of fearfulness, palpitations,
period for 10months. She feels well but is faintness, hyperventilation, dryness of the
anxious as her mother had an early mouth with peri-oral tingling and
menopause. Choose the single most cramping of the hands. His symptoms last
appropriate initial inv? 5-10 mins and have worsened since their
a. Serum estradiol conc. onset 3months ago. He is worried he may
b. Serum FSH/LH be having a heart attack. An ECG shows
c. Serum progesterone conc. sinus tachycardia.
d. None What is the single most appropriate
e. Transvaginal US immediate intervention?
a. High flow oxygen
Ans. The key is B. Serum FSH/LH [here serum b. IV sedation
oestrogen is also important as i) low oestrogen + c. Rebreathe into a paper bag
low FSH + low LH suggest hypothalamic d. Refer for anxiety management course
amenorrhoea and i) low oestrogen + high FSH + e. Refer for urgent cardiology opinion
high LH suggest premature ovarian failure! So the
main determinant is serum FSH/LH. Likely cause
here is premature ovarian failure].
Ans. The key is C. Rebreathing into paper bag.
520. A 50yo man with DM suddenly [Patient has anxiety disorder (panic) which causes
develops persistent crushing central chest hyperventilation and CO2 washout leading to
pain radiating to the neck. What is the respiratory alkalosis. Symptoms will improve by
single most appropriate dx? rebreathing into paper bag as it will cause gradual
a. Angina increase of CO2 in paper bag and decrease the
b. Costochondritis (tietz’s disease) severity of respiratory alkalosis].
c. Dissecting aneurysm 522. An 8yo boy has longstanding asthma.
d. MI He has admitted with a severe episode
e. Pulmonary embolism and is tired and drowsy. He has not
Ans. The key is C. Dissecting aortic aneurism. improved on oxygen, inhaled B2 agonist
Probably wrong key. Correct key should be D. MI. and IV hydrocortisone. CXR shows
[The features described is insufficient and can be bilateral hyperinflation. He is too
seen in both aortic dissection and MI. However breathless to use a peakflow meter and is
dissection pain is described as tearing and O2 sat <90%. What is the single most
crushing pain is often used for mi pain. Both appropriate inv?
dissection and mi can have pain radiation to neck. a. CBG
History of diabetes goes with mi as it is a b. CXR
recognized risk factor for mi. Some may argue in c. CT chest
DM mi will be painless! But it is not always the d. Pulse oximetry
case. MI is only painless when autonomic e. Spirometry
neuropathy becomes well established].
Ans. The key is A. CBG. [It will point towards
acidosis and indicate whether assisted ventilation
is needed or not].
523. A man was operated for colorectal Ans. The key is B. Evaluate patients capacity to
ca. His pain is relieved with morphine refuse treatment.
60mg bd PO but now he can’t swallow 527. A 23yo woman with painless vaginal
medications. What will be the next bleeding at 36wks pregnancy otherwise
regimen of analgesic administration? seems to be normal. What should be done
a. Oxycodone
next step?
b. Fentanyl patch
a. Vaginal US
c. Morphine 60mg IV/d
b. Abdominal US
d. Morphine 240mg IV/d
c. Vaginal exam
Ans. The key is B. Fentanyl patch. [Here S/C d. Reassurance
morphine 1/2 the dose of oral can be given (not
Ans. The key is B. Abdominal US. This is a wrong
present in option) or I/V morphine 1/3rd the oral
key. The correct key is A. Vaginal US.[Painless
dose can be given. Here I/V doses are not
vaginal bleeding at 36 weeks indicates the
appropriate so we should go for B. Fentanyl patch
diagnosis of placenta previa, which can be better
as required morphine dose is known].
evaluated by vaginal US].
524. Just above the mid-inguinal point.
What is the single most appropriate
landmark?
a. Femoral artery pulse felt
b. Mcburney’s point
c. Stellate ganglion
d. Deep inguinal ring
e. Transpyloric plane
Ans. The key is D. Deep inguinal ring. 528. A 29yo lady admitted with hx of
repeated UTI now developed hematuria
525. 5th ICS in the ant axillary line. What
with loin pain. What is the most probable
is the single most appropriate landmark?
a. Apex beat dx?
a. Acute pyelonephritis
b. Chest drain insertion
b. Chronic pyelonephritis
c. Stellate ganglion
c. UTI
d. Transpyloric plane
d. Bladder stone
e. Vena cava opening into the diaphragm
Ans. The key is A. Acute pyelonephritis. [In a
Ans. B. Chest drain insertion.
patient having hematuria and loin pain with
526. A 34yo man with MS has taken an history of repeated UTI suggest acute
OD of 100 tablets of paracetamol with pyelonephritis].
intent to end his life. He has been brought 529. A 45yo chronic smoker attends the
to the ED for tx but is refusing all OPD with complaints of persistent cough
intervention. and copious amount of purulent sputum.
a. Assessment
He had hx of measles in the past. Exam:
b. Evaluate pt’s capacity to refuse tx
finger clubbing and inspiratory
c. Establish if pt has a prv mental illness
crepitations on auscultation. What is the
single most likely dx?
a. Interstitial lung disease feeling tired and lethargic, sorethroat,
b. Bronchiectasis hepatosplenomegally all are well known features
c. Asthma of ALL].
d. COPD
532. A 65yo man presents with back pain.
e. Sarcoidosis
Exam: splenomegaly and anemia. Blood:
Ans. The key is B. Bronchiectasis. [Persistent WBC=22, Hgb=10.9, Plt=100, ESR=25. He
cough with copious purulent sputum and finger has been found to have Philadelphia
clubbing points towards the diagnosis of chromosome. What is the single most
bronchiectasis. Severe lung infections such as likely dx?
tuberculosis (TB), whooping cough, pneumonia or a. ALL
measles can damage the airways at the time of b. AML
infection. Bronchiectasis may then develop c. CML
(WHO)]. d. CLL
e. Lymphoma
530. A 68yo man has had malaise for 5 Ans. The key is C. CML. [anaemia, raised WBC
days and fever for 2 days. He has cough count, low platelet (platelet may be variable) are
and there is dullness to percussion at the known features of CML, splenomegaly
(particularly if massive) is very suggestive of CML
left lung base. What is the single most
and Philadelphia chromosome is characteristic of
appropriate inv?
CML].
a. Bronchoscopy
b. CXR 533. A 24yo woman has 8wk amenorrhea,
c. CT right sided pelvic pain and vaginal
d. MRI bleeding. She is apyrexial. Peritonism is
e. V/Q scan elicited in the RIF. Vaginal exam reveals
right sided cervical excitation. What is the
most probable dx?
Ans. The key is B. CXR. [Given presentation is a. Ectopic pregnancy
suggestive of pneumonia for which investigation b. Salpingitis
of choice is CXR]. c. Endometriosis
d. Ovarian torsion
e. Ovarian tumor
531. A 5yo child was admitted with hx of
feeling tired and lethargic all the time, Ans. The key is A. [Salpingitis, Endometriosis,
bleeding gums and sore throat since the overian torsions do not associated with
amenorrhoea. In ovarian tumour three main
last 3months. Exam: hepatosplenomegaly.
features are i) increased abdominal size and
What is the most probable dx?
persistent bloating (not bloating that comes and
a. ALL
goes) ii) persistent pelvic and abdominal pain iii)
b. AML
difficulty eating and feeling full quickly, or feeling
c. CML
nauseous. Patient with pelvic pain and vaginal
d. CLL bleeding, peritonism and cervical exitation
e. Lymphoma obviously points towards Ectopic pregnancy].
Ans. The key is A. ALL. [Commonest leukemia in
children is ALL. Bleeding gums (low platelet),
534. A 64 yo woman has been treated for a. CXR
breast cancer with tamoxifen. What other b. None
drug should be added to her tx regime? c. Peak flow rate diary
a. Bisphosphonates d. Pulse oximetry
b. Calcium
Ans. The key is C. Peak flow rate diary. [Peak flow
c. Vit D
rate diary shows diurnal variation. This diary
d. Calcitonin
shows when the bronchoconstriction remains
e. Phosphate binders
worse and guides to use bronchodilators prior to
that times].
Ans. The key is G. CTPA. [Prolonged plane journey 550. A 55yo woman with a persistent
is a recognized risk factor for thromboembolism cough and hx of smoking develops left
and hence pulmonary embolism also! Sharp chest sided chest pain exacerbated by deep
pain and breathlessness after 3 days of plane breathing with fever and localized
journey is highly suggestive of pulmonary crackles.
embolism the investigation of choice for which is What is the single most appropriate dx?
CTPA]. a. Dissecting aneurysm
b. Pericarditis
548. A tall thin young man has sudden
c. Pneumonia
pain in the chest and becomes breathless
d. Pneumothorax
while crying. What is the single most
e. Pulmonary embolism
appropriate inv?
a. Cardiac enzymes Ans. The key is C. Pneumonia. [chest pain
b. CXR exacerbated with deep breathing, fever and
c. CT localized crackles are highly suggestive of
d. ECG pneumonia].
e. V/Q scan
551. A 40yo woman complains of
dysphagia for both solids and liquids. She
Ans. The key is B. CXR. [tall thin young men are sometimes suffers from severe
particularly prone to develop pneumothorax. retrosternal chest pain. Barium swallow
Sudden pain and breathlessness in this young reveals a dilated esophagus which tapers
man are highly suggestive of pneumothorax. So
to a fine distal end. What is the best
investigation of choice is CXR].
management strategy?
a. Reassurance
b. Antispasmodics d. Thalamus
c. Dilatation of the LES e. Brain stem
d. Endoscopic diverticulectomy
Ans. The key is C. internal capsule. [Hippocampal
e. Barium swallow
lesion causes mainly memory impairment.
Ans. The key is C. Dilatation of LES. [Dysphagia for Cerebellum has its diagnostic features that is not
both solids and liquids suggest neuromuscular present here. Thalamic lesion can lead to
dysphagia while dysphagia only for solid suggests impairment of arousal, orientation, learning and
mechanical obstruction. Here features are memory, facial paresis, language deficit,
consistent with achalasia for which lower hemispatial neglect, hemisensory loss,
oesophageal sphincter dilation (balloon hemiparesis, hemiataxia and visual field defect.
dilatation) is a treatment modality]. Brainstem stroke causes impaired consciousness,
disorder of blood pressure, and breathing
552. A 38yo female G4 at 32wks of abnormality. Given picture is typical of lesion in
pregnancy presented with thick white internal capsule].
marks on the inside of her mouth for
3wks. Her mouth including her tongue 554. A 72yo man brought to the ED with
appeared inflamed on examination. She onset of paraplegia following a trivial fall.
smokes 20 cigarettes/day despite advice He was treated for prostatic malignancy
to quit. She attends her ANC regularly. in the past. What is the single most
What is the most probable dx? probable dx?
a. Paget’s disease
a. Lichen planus
b. Osteoporotic fx of vertebre
b. Aphthous ulcer
c. Secondary
c. Smoking
d. Multiple myeloma
d. Candidiasis
e. Spondylosis
e. Leukoplakia
Ans. The key is E. Magnesium sulphate IV. Ans. The key is B. Chronic secretory OM with
[patient has established eclampsia. So she should effusion.
be treated with Magnesium sulphate as with 4g
magnesium sulfate in 100mL 0.9% saline IVI over
5min + maintenance IVI of 1g/h for 24h. Beware 560. A thin 18yo girl has bilateral parotid
depressed respiration. If recurrent seizure give 2g swelling with thickened calluses on the
IVI magnesium sulfate over 5 min. Check tendon dorsum of her hand. What is the single
reflexes and respiratory rate every 15min. Stop most likely dx?
magnesium sulfate IVI if respiratory rate <14/min a. Bulimia nervosa
or tendon reflex loss, or urine output <20mL/h]. b. C1 esterase deficiency
[OHCS, 9th edition, page 49]. c. Crohn’s disease
d. Mumps
e. Sarcoidosis
and his stools are pale. What is the most
Ans. The key is A. Bulimia nervosa. [18 year thin likely dx?
girl, bilateral parotid swelling and thickened a. Galactosemia
calluses on the knuckles from self induced b. Biliary atresia
vomiting are suggestive of Bulimia nervosa. c. G6PD deficiency
Bulimia often is associated with bilaterak parotid d. Rh incompatibility
swelling (parotid hypertrophy)]. e. Congenital viral infection
561. A 48yo presents with severe chest Ans. The key is B. Biliary atresia. [Deep jaundice at
pain since the last 40mins. In the ED he is 6th week with pale stools suggests obstructive
given oxygen, GTN, morphine. ECG=ST jaundice. So most likely diagnosis here is biliary
elevation. Bloods=increased troponin atresia].
levels. What is the next step of
management?
a. Beta blockers 564. A 45yo man with colon cancer now
b. Percutaneous angiography develops increased thirst, increased
c. Anticoagulant & heparin frequency in urination and weight loss. His
d. Clopidogrel fasting blood glucose=9mmol/L.
e. Aspirin what is the most appropriate
management?
a. Oral hypoglycemic
Ans. The key is B. Percutaneous angiography. b. Insulin long acting
c. Short acting insulin before meal
d. IV insulin
562. A 34yo female presents with a foul e. Subcutaneous insulin
smelling discharge. What set of organisms
are we looking for to be treated here?
a. Chlamydia, gonorrhea Ans. The key is A. Oral hypoglycemic. [Increased
b. Chlamydia, gardenella thirst and increased frequency in urination along
c. Chlamydia, gonorrhea, gardenella with weight loss is suggestive of DM supported by
fasting blood glucose of 9 mmol/L. At the age of
d. Gonorrhea, gardenella
45 most likely type of diabetes is NIDDM or type 2
e. Gardenella only
DM which is treated by oral hypoglycemic
agents].
570. An 18m boy has been brought to the 572. A pt presents with fever, dry cough
ED because he has been refusing to move and breathlessness. He is tachypneic but
his left arm and crying more than usual chest is clear. Oxygen saturation is normal
for the past 24h. He has recently been at rest but drops on exercise.
looked after by his mother’s new bf while What is the single most likely dx?
she attended college. Assessment shows a. CMV infection
multiple bruises and a fx of the left b. Candida infection
humerus which is put in plaster. What is c. Pneumocystis carinii infection
the single most appropriate next step? d. Cryptococcal infection
a. Admit under care of pediatrician e. Toxoplasma abscess
b. Discharge with painkillers
c. Follow up in fx clinic
d. Follow up in pediatric OPD Ans. The key is C. Pneumocystis carinii infection.
e. Follow up with GP [Fever, dry cough, breathlessness, tachypnoea
with clear chest is seen in pneumocystis carinii
pneumonia. Normal oxygen saturation which
Ans. The key is A. Admit under care of drops on exercise is characteristic of
pediatrician. [This is NAI. So the child cannot be pneumocystis carinii pneumonia].
handover to the risk again and should be
admitted to protect him from further injury done
by mothers boyfriend while serial x-rays and 573. A 14yo boy fell and hit his head in the
relevant investigations done and asked for child playground school. He didn’t lose
protection unit’s help]. consciousness. He has swelling and
tenderness of the right cheek with a
subconjuctival hemorrhage on his right
571. A 74yo female presents with eye.
headache and neck stiffness to the ED. What is the most appropriate initial inv?
Following a LP the pt was started on IV a. CT brain
ceftriaxone. CSF culture = listeria b. EEG
monocytogenes. c. MRI
What is the appropriate tx? d. Skull XR
a. Add IV amoxicillin e. Facial XR
b. Change to IV amoxicillin + gentamicin
c. Add IV ciprofloxacin
d. Add IV co-amoxiclav Ans. The key is E. Facial X-ray. This is a wrong key
e. Continue IV ceftriaxone as mono-therapy Correct key is A. CT brain. [With the risk of basal
fracture we should do CT scan to diagnose this. In
present case CT is better than MRI. Skull X-rays
Ans. The key is B. Change to IV amoxicillin + are no longer recommended as first line
gentamycin. [From the given option B is the most investigation].
574. A 15m child is due for his MMR d. Sarcoidosis
vaccine. There is a fam hx of egg allergy. e. Sjogren’s syndrome
He is febrile with acute OM. What is the
single most appropriate action?
a. Defer immunization for 2wks Ans. The key is C. HIV disease. [The lesion
b. Don’t give vaccine described is leukoplakia which is likely association
c. Give half dose of vaccine of HIV disease].
d. Give paracetamol with future doses of
the same vaccine
e. Proceed with standard immunization 577. A 3m baby was miserable and cried
schedule for 2h following his 1st routine
immunization with DTP, HiB and
meningitis. What is the single most
Ans. The key is A. Defer immunization for 2 wks. appropriate action?
a. Defer immunization for 2wks
b. Don’t give vaccine
c. Give half dose of vaccine
575. A 33yo lady with Hodgkin’s d. Give paracetamol with future doses of
lymphoma presents with temp=40C, left the same vaccine
sided abdominal pain and lymphadenitis. e. Proceed with standard immunization
Blood was taken for test. What will you do schedule
next?
a. Wait for blood test
b. Start broad spectrum IV antibiotics Ans. The key is E. Proceed with standard
c. Oral antibiotics immunization schedule.
d. CBC
e. Monitor pyrexia
578. A 65yo man with HTN develops
gingival hyperplasia. What is the single
Ans. The key is B. Start broad spectrum IV most likely dx?
antibiotics. [The patient is immunocompromized a. ACEi
with signs of infection (temp=40◦C, left sided b. Beta blockers
abdominal pain and lymphadenitis) broad c. Crohns disease
spectrum IV antibiotic should be started d. Nifedipine
empirically while waiting for blood reports]. e. Sarcoidosis
576. A 40yo man with marked weight loss Ans. The key is D. Nifedipine. [Gingival
over the preceding 6m has bilateral white, hyperplasia is a recognized side effect of calcium
vertically corrugated lesion on the lateral channel blockers].
surfaces of the tongue.
What is the single most likely dx?
a. C1 esterase deficiency 579. A 65yo woman is undergoing
b. Crohns disease coronary angiography. What measure will
c. HIV disease protect her kidneys from contrast?
a. Furosemide adrenal insufficiency (here vomiting, abdominal
b. Dextrose pain and sudden falls in the morning secondary to
c. 0.45% saline postural hypotension on getting up from bed
d. 0.9% saline points towards the diagnosis of adrenal
insufficiency)].
Ans. The key is D. Phosphate enema. [In feacal 583. A 30yo man is suffering from fever,
impaction oral laxative is not the choice but rash and photophobia. Doctors are
phosphate enema is the best option here]. suspecting he is suffering from meningitis.
Which is the best medication for this
condition?
581. A 26yo woman being treated for a a. Ampicilling
carcinoma of the bronchus with steroids b. Cefotaxime
presents with vomiting, abdominal pain c. Tetracycline
and sudden falls in the morning. What is d. Acyclovir
the most specific cause for her symptoms? e. Dexamethasone
a. Steroid side effects
b. Postural hypotension
c. Adrenal insufficiency Ans. The key is B. Cefotaxime. [The patient is
d. Conn’s disease getting probable meningococcal meningitis.
e. Cushing’s disease Before confirming the diagnosis suggested
treatment is, where
600. Which artery runs in the anterior Ans. The key is C. CMV infection. [Large multiple
cotton wool spots are seen in early stage of CMV
inter-ventricular groove?
retinitis].
603. A 53yo had a dental extraction after
which he recently had a mitral valve Ans. The key is B. 3rd person auditory
prolapse, high temp of 39C, cardiac failure hallucinations. [Third person hallucinations are
and new cardiac murmur. auditory hallucinations in which patients hear
What is the single most likely dx? voices talking about themselves, referring to
a. Atheroma them in the third person, for example "he is an
b. Congenital evil person".].
c. Regeneration
d. Infection
e. Neoplastic 606. A 65yo woman had an excision of
colonic tumor 3yrs ago. Now she is losing
weight and feels lethargic. Exam: pale but
Ans. The key is D. Infection. [Infective no abdominal findings.
endocarditis]. What is the most appropriate inv?
a. CA 125
604. A 12yo boy with a hx of fall on an
b. CA 153
outstretched hand was brought to the ED c. CA 199
with swelling and pain around the elbow. d. CEA
His radial nerve was affected. e. AFP
What is the type of fx?
a. Angulated fx
b. Epiphyseal fx
Ans. The key is D. CEA. [CA 125 = ovarian cancer;
c. Compound fx
CA 153 = cancer breast; CA 199 = pancreatic
d. Spiral fx
cancer; CEA = colorectal carcinoma; AFP =
hepatocellular carcinoma].
a. Anal reflex – S1
b. Ankle jerk – L5 Ans. The key is D. Drop attacks. [Drop attacks are
c. Biceps jerk – C7 & C8 sudden spontaneous falls while standing or
d. Knee jerk – L3 & L4 walking, with complete recovery in seconds or
e. Triceps jerk – T1 minutes. There is usually no recognized loss of
consciousness, and the event is remembered].
Ans. The key is D. Extensor pollicis brevis. [Action 613. A pt with prv hx of HTN, the
of extensor pollicis brevis = extension of thumb at membranes have ruptured and the cervix
metacarpophalangeal joint. Extensor pollicis is 3cm dilated. 4h later on examination
longus = extends the terminal phalanx of the showed that the cervix was still 3cm
thumb]. dilated.
What is the single most appropriate
management for her labor?
a. Repeat vaginal examination in 4h Ans. The key is B. Bendroflumethiazide. [High
b. CTG blood sugar is a well known side effect of
c. C-section bendroflumethiazide].
d. External rotation
e. IV syntocin drip
616. A 27yo waitress has pelvic pain,
dysmenorrhea and increasingly heavy
Ans. The key is E. IV syntocinon drip. [There is no periods. She also complains of
progress of labour in 4 hours. Hence syntocinon dyspareunia. There is generalized pelvic
drip should be given]. tenderness without peritonism. Pelvic US
is normal. What is the most likely dx?
a. Endometriosis
614. A 6yo girl has had 2 short episodes of b. Uterine fibroid
cough and wheeze over the last 12m. c. Pelvic congestion syndrome
These 2 acute episodes responded quickly d. PID
to bronchodilator, she has no symptoms e. Tubal pregnancy
or abnormal physical signs. She has slight
eczema and her mother has asthma.
What is the single most appropriate inv? Ans. The key is C. Pelvic congestion syndrome. [In
a. CXR pelvic congestion syndrome there develops
b. Peak flow rate diary varicose veins in the lower abdomen from
c. Pulse oximetry prolonged standing (as occurred here in a
d. Spirometry waitress who remains standing for long) with
e. Sweat test some pain syndromes like pelvic pain,
dysmenorrea, dyspareunia and generalized pelvic
tenderness without peritonism. Also there may
be associated menorrhagia].
Ans. The key is D. Spirometry. [spirometry is the
preferred initial test (if available) to assess the
presence and severity of airflow obstruction less
effort dependent and more repeatable though
617. A 14yo girl is clinically obese. She has
less applicable in acute severe asthma]. not started her periods yet and has severe
acne. Among her inv, a high insulin level is
found. What is the most probable dx?
a. Cushing’s syndrome
615. A 45yo man had recently started
b. Grave’s disease
taking anti-HTN therapy. 6m later his
c. Acquired hypothyroidism
RBS=14mmol/l. Which single drug is most
d. PCOS
likely to have caused this?
e. Addison’s disease
a. Amlodipine
b. Bendroflumethiazide
c. Doxazosin
d. Lorsartan Ans. The key is D. PCOS. [It is not cushing’s as
e. Ramipril insulin levels in cushing’s are not usually raised!
Here obesity, primary amenorrhea , acne and
particularly high level of insulin makes the likely
diagnosis to PCOS].
believes that the staff and other pts know
exactly what she is thinking all the time.
618. An 18yo girl with primary
What is the most likely symptom this pt is
amenorrhea complains of severe
suffering from?
abdominal pain every 4-8weeks which is
a. Thought insertion
now getting worse. Exam: lower
b. Thought withdrawal
abdominal mass is felt. c. Thought block
What is the most probable dx? d. Though broadcasting
a. Ectopic pregnancy
e. Hallucination
b. Ovarian carcinoma
c. Hematometrium
d. Biliary colic
Ans. The key is D. Thought broadcasting.
e. Renal carcinoma
624. A man sat cross-legged for about 626. A 19yo man presents for the 1st time
30mins. After this he was unable to with a firm and unshakable belief that he
dorsiflex his left foot and had loss of is being followed by terrorists who are
sensation in the web space between the plotting against him.
big toe and the 2nd toe. He also has What is the single best term for this man’s
sensory loss on the same side of the foot condition?
a. Delusion of persecution
after 2h.
b. Delusion of grandeur
Which of the following was affected?
a. Femoral nerve c. Delusion of control
b. Sural nerve d. Delusion of reference
c. Peroneal nerve e. Delusion of nihilism
d. Sciatic nerve
Ans. The key is C. Punch biopsy. [As the lesion is 645. A 56yo man was recently put on anti-
on the surface punch biopsy can be well
HTN meds and recent biochemistry on 2
obtained].
occasions showed: Na+=132, K+=7.6,
643. A 50yo man presents with low mood, Urea=11.3, Creat=112.
poor concentration, anhedonia and Which of the following drugs is
insomnia. He has had 2 episodes of responsible for this result?
increased activity, promiscuity and a. Amlodipine
aggressive behavior in the past. He was b. Bendroflumethiazide
arrest 8m ago for trying to rob a bank c. Doxazosin
claiming it as his own. d. Atenolol
Which drug is most likely to benefit him? e. Ramipril
a. Haloperidol
b. Citalopram
c. Desipramine Ans. The key is E. Ramipril. [ACEI and ARB are
d. Carbamazepine known to raise the serum potassium level].
e. Ethosuximide
646. A 46yo woman has offensive yellow 649. A 55yo woman who attends the clinic
discharge from one nipple. She had a hx of has recently been dx with a depressive
breast abscess 3yrs ago. What is the episode. She complains of unintentionally
possible dx? waking early in the morning, a recent
a. Duct papilloma disinterest in sex and a loss of appetite,
b. Duct ectasia losing 5kg weight in the last month. She
c. Duct fistula feels that her mood is worse at the
d. Breast cancer beginning of the day. What is the most
likely dx for this pt?
a. Mild depression
a. Ans. The key is C. Duct fistula. b. Moderate depression
c. Severe depression
d. Low mood
647. A 35yo woman undergoing tx for TB e. Pseudo depression
presents with malar rash, photosensitivity
and hematuria. What is the single most
likely positive antibody? Ans. The key is B. Moderate depression. [Sleep
a. Anti Ds DNA disturbance, disinterest in sex and loss of appetite
b. Anti Sm points towards the diagnosis of moderate
c. Anti Histone depression].
d. Anti La
e. Anti centromere
650. An employer sent his worker to the
ED after having hit his head on a machine.
Ans. The key is C. Antihistone antibody. [Anti O/E: normal. What is the single most likely
histone antibody is present in 95% cases of drug inv you would do?
induced lupus]. a. Skull XR
b. CT head
c. MRI head
648. A 6wk child with profuse projectile d. Reassure
vomiting. What is the first thing you will Ans. The key is A. Skull XR. This is wrong key!
do? Correct key should be B. CT head.
a. US
b. Check serum K+ level
c. ABG 651. A lady with fam hx of ovarian
d. NG tube carcinoma has a pelvis US that fails to
e. IV fluids reveal any abnormality. What is the single
most appropriate inv?
a. Pelvic CT
b. CA 125
Ans. The key is B. Check serum K+ level. This is a
c. CA 153
wrong key! Correct key is E. IV fluid. [We shall
d. Laparoscopy
resuscitate first with normal saline].
e. MRI
Ans. The key is A. CRAO. [Pale retina with cherry
red spot in macular region is seen in CRAO].
Ans. The key is B. CA 125. [CA 125 is tumour
marker for ovarian tumour].
e. CXR
Ans. The key is A. Anterior MI. [Here ST elevation
in L1, aVL, v2,v3,v4 and v5. So the diagnosis is
Ans. The key is A. ECG. [Here sudden anterior MI (as L1 and aVL and v2-v5,
unconciuosness without any provocation, which anterolateral would be more appropriate
makes arrhythmia the most likely cause; hence description)].
we should do ECG].
685. A schizophrenic says: life is unfair. I
like fairs. Fairs have food. It must be good.
What term describes this pt’s speech?
a. Neologism
b. Flight of ideas
Ans. The key is B. Hemophilia. [This is a
c. Broc1a’s aphasia
controversial question with description of both
d. Wernicke’s aphasia factor deficiency and platelet disorder!! Purpuric
e. Clang association rash on buttock does not support hemophilia and
other features like hemarthrosis in knee, isolated
raised APTT support hemophilia. Hopefully in
Ans. The key is E. Clang association. [The rhymic
PLAB this mixed picture will not be seen in a
use of words as described is known as clanging or
single question. Better to be with given key i.e.
clang association often seen in schizophrenia].
assuming the better option is B. Hemophilia].
696. A 4yo boy has a cough and arthritis 698. A young man has diarrhea, loss of
followed by rash on legs which are non- weight and flatulence for 3 days.
blanching on glass test. No hx of fever. What is the most imp tx?
PT=13, APTT=31, Hgb=12, WBC=6.5, a. Metronidazole
Plt=300. What’s the most likely dx? b. Fluconazole
a. Meningitis septicemia c. Vancomycin
b. Hemophilia d. Amoxicillin
c. HSP
Ans. The key is A. Metronidazole. [An incomplete
d. ITP
question!!]
e. TTP
720. A 65yo known alcoholic is brought Ans. The key is B. Osteoporosis. It is a wrong key.
into the hospital with confusion, The correct option is A. Headache. [Headache
aggressiveness and ophthalmoplegia. He from intracranial hemorrhage is more important
is treated with diazepoxide. complication about which patient should be
What other drug would you like to careful with (it is more important than
prescribe? osteoporosis)].
a. Antibiotics
b. Glucose
723. A 24yo woman is afraid to leave her
c. IV fluids
house as whenever she goes out, she
d. Disulfiram
tends to have SOB and sweating. She has
e. Vit B complex
stopped going out except with her
husband.
What is the most likely dx?
Ans. The key is E. Vit B complex. [This is a case of
a. Social phobia
Wernicke’s encephalopathy which is treated with
b. Claustrophobia
Vit B complex].
c. Depression
d. Panic disorder
721. A pt suffering from schizophrenia e. Agoraphobia
laughs while talking about his father’s
death. Which term best describes his
Ans. The key is E. Agoraphobia. [Agoraphobia:
condition?
Extreme or irrational fear of open or public
a. Depression
places].
b. Flat affect
c. Emotional liability
d. Incongruent affect 724. A pt on HTN drugs develops
e. Clang association hyperkalemia. Which anti-HTN is likely to
cause it?
a. Ramipril
Ans. The key is D. Incongruent affect. b. Lorsartan
[Incongruent affect means inappropriate c. Thiazide
emotional response like loughing hearing a sad d. Nifedipine
news or crying hearing a good news]. e. Furosemide
Ans. The key is A. Ramipril. [Both ramipril and Ans. The key is A. Stop smoking.
losartan can cause hyperkalemia].
725. A young man develops itching worse 728. A 64yo male was admitted to the
at night and following bathing. Exam: medical ward with complaint of diarrhea,
greysish white linear rash can be seen on abdominal pain and weight loss for few
the wrist and periumbilical area. What is months. Exam: clubbing, perianal skin
the dx? tags and abdominal tenderness.
a. Scabies Colonscopy reveals transmural
b. Polycythemia granulomatous inflammation involving
c. Urticarial ileocecal junction. He was dx with what?
d. Atopic eczema a. CD
e. Lichen planus b. UC
c. Bowel cancer
d. Gastric cancer
Ans. The key is A. Scabies. e. IBD
731. An old man having T2DM with 734. A 40yo teetotaler woman is
increased skin tanning, heart failure and recovering from a hysterectomy 2days
having high ferritin (hemochromatosis) ago. At night she becomes agitated and
level is refusing tx. Where is the first site complains of seeing animals and children
of cancer? walking around the ward. What is the
a. Testes most likely dx?
b. Adrenal a. Delirium tremens
c. Liver b. Toxic confusional state
d. Pancreas c. Hysteria
d. Mania
e. Drug induced personality disorder
Ans. The key is C. Liver.
Ans. The key is A. Compartment syndrome. Ans. The key is A. Take it after meal.
[Acute compartment syndrome occurs after a
traumatic injury such as a car crash. The trauma
causes a severe high pressure in
766. A 52yo man with hx of ant MI 3 wks
the compartment which results in ago developed sudden onset of dyspnea.
insufficient blood supply to muscles and nerves. Exam:
Acute compartment syndrome is a medical BP=100/60mmHg, pulse=100bpm,
emergency that requires surgery to correct. If SaO2=88%, chest=bilateral crackles.
untreated, the lack of blood supply leads to Which inv would you do to find the
permanent muscle and nerve damage and can underlying cause?
result in the loss of function of the limb]. a. CXR
b. Echo
c. D-dimer
764. You are a FY doctor in the ED when a d. Radionucleotide lung scan
mother brings her 2yo son to you with a e. Troponin
1h hx of noisy breathing. She state that
although he had mild coryza over the last
week, he was improving and so they had Ans. The key is A. CXR. It is wrong key. Correct key
gone to a children’s picnic with nursery should be B. Echo. [The question does not want
friends. Another parent had found him to know the features of pulmonary oedema
coughing and spluttering, and ever since rather it is clearly mentioned “which investigation
his breathing has remained noisy. Though would you do to find out the UNDERLYING
he appears well in the ED, his current CAUSE”. As in this patient sudden papillary
muscle rupture or VSD may be the likely cause
observations demonstrate a raised RR and
(ventricular aneurism may take a more slower
sat=91% on air. What is the most likely course) so echo seems to be more logical
dx? approach].
a. Anaplyaxis
b. Croup
c. Foreign body aspiration
767. A 49yo man lost his job and now is
d. Epiglottitis
homeless. He was found wandering in the
park. He is muttering that some people
are after him. Alcohol was tested and it
Ans. The key is C. Foreign body aspiration.
was negative. What will your next step
be?
a. Thiamine
765. A pt taking doxycycline complains of b. Neuropsycho analysis
nausea, indigestion, abdominal pain and c. Mini mental state
vomiting. What will you advise? d. CT head
a. Take it after meals
e. MRI head
b. Take it before meals
c. Stop the drug
Ans. The key is B. Neuropsychoanalysis. [As the 770. A 45yo woman presents with
patient has persecutory delusion he is likely rotational vertigo, nausea and vomiting,
suffering from psychosis for which especially on moving her head. She also
neuropsychoanalysis is the logical approach].
had a similar episode 2yrs back. These
episodes typically follow an event of runny
nose, cold, cough and fever. What is the
768. A pt with SNHL and loss of corneal
most probable dx?
reflex on the left side. What is the most a. Acoustic neuroma
definitive inv? b. Meniere’s disease
c. Labyrinthitis
a. CT of internal auditory meatus
d. BPPV
b. Nuclear imaging of brain
e. Vestibular neuronitis
c. MRI of internal auditory meatus
d. Radio isotope scan
e. XR skull
Ans. The key is E. Vestibular neuronitis. [Here no
sensoryneural hearing loss (SNHL present in
acoustic neuroma, Meniere’s disease and
Ans. The key is C. MRI of internal auditory
labyrinthitis). Runny nose, cold, cough and fever
meatus. [The likely cause is acaustic neuroma for
are recognized trigger of vestibular neuronitis but
which appropriate investigation is MRI internal
not BPPV].
acoustic meatus].
Ans. The key is B. pH = 7.4, PaCO2 = 1.5 792. A female pt with HTN and serum
K+=2.7mmol/l. which hormone would you
789. A man presents with abdominal pain, be looking for?
vomiting, pulse=120bpm, BP=90/60mmHg
a. Aldosterone
and a rigid abdomen. His chest is clear. b. Cortisol
What is the immediate management? c. Thyrotoxin
a. Call radiology
d. Renin
b. Admit to medical ward
e. Testosterone
c. Urgent admission to ITU (Intensive
Therapy Unit)
790. A 60yo man presents with weight 793. A 25yo male with a hx of frequent
loss and Hgb=6. Hx reveals that he has binge drinking presents 4h after having
abdominal pain and diarrhea for the past had a take away meal following a nights
6m. What is the most appropriate inv? heavy drinking. He complains of nausea
a. Barium enema
and has vomited on several occasions.
b. Colonoscopy
After the last vomiting episode, he
c. Sigmoidoscopy
vomited approximately a cupful of blood.
On admission, he smells of alcohol,
Ans. The key is B. Colonoscopy. [Likely diagnosis is pulse=100bpm, BP=140/80mmHg. He has
IBD. So Colonoscopy should be done]. some tenderness in the epigastrium.
What is your dx?
a. Gastric carcinoma
791. A 35yo primigravida post C-section
b. Mallory-weiss tear
complains of inability to void. She denies
c. Esophageal carcinoma
dysuria but complains of fullness. She was
d. Esophageal varices
treated with an epidural for analgesia.
e. Esophageal varices
What is the single most appropriate inv?
f. Peptic ulceration 48. What is the most appropriate
management for this pt?
a. IV fluid
Ans. The key is Mallory-weiss tear. [Repeated b. Needle thoracocentesis
retching and vomiting is a recognized cause of c. 100% oxygen
Mallory-weiss tear which is supported here by d. Portable XR
vomiting a cupful of blood].
795. A 53yo man with prv hx of COPD Ans. The key is B. Subarachnoid hemorrhage.
presents with breathlessness and purulent [Cerebral aneurisms are recognized association of
sputum. O2 stat=85% on air. ABG: PCKD which may lead to subarachnoid
PaO2=7.6, PaCOS=7. What is the hemorrhage].
appropriate management for his
condition? 798. A 36yo male dx with glioblastoma
a. 24% oxygen since last 5m has cerebral edema and is
b. Mechanical ventilation
on dexamethasone. He has diarrhea and
c. 100% oxygen
vomiting for the last 3days. He has been
d. Nebulized salbutamol
suffering from repeated falls today.
What could be the possible cause for his
falls?
Ans. The key is A. 24% oxygen. [Patient has a. Adrenal insufficiency
hypoxemia and hypercapnea (type2 respiratory b. Dehydration
failure). Hence he should be put on 24% oxygen.
c. Dexamethasone therapy
100% oxygen will abolish hypoxic drive and
d. Raised ICP
worsen hypercapnea].
796. A 34yo man was involved in a RTA Ans. The key is A. Adrenal insufficiency.
and whilst in the ambulance his GCS [Prolonged dexamethason use suppresses
deteriorated and RR increased from 30- adrenal axis and intercurrent illness can lead to
adrenal insufficiency (addisonian crisis) in such a. Aortic aneurysm
cases]. b. Mesenteric cyst
c. Umbilical hernia
799. A 2yo child is brought by his mother. Ans. The key is A. Aortic aneurism. [There is
features of shock with pulse 120bpm and bp
The mother had hearing impairment in
70/40mmHg and sudden collapse of the patient
her early childhood and is now concerned
with pulsatile mass in abdomen makes the likely
about the child. What inv would you do? diagnosis of ruptured aortic aneurism].
a. Audiometry
b. Distraction testing 802. A woman prv in good health presents
c. Scratch test with sudden onset of severe occipital
d. Tuning fork headache and vomiting. Her only physical
sign on exam is a stiff neck.
Choose the most likely dx.
Ans. The key is A. Audiometry. [Conditioned a. Subarachnoid hemorrhage
response audiometry 2-4 years, pure tone audio b. Subdural hematoma
gram for greater than 5 years, less than 6 months c. Cerebellar hemorrhage
otoacoustic emission or brainstem evoked d. Migraine
response, distraction test for 6-18 months, OHCS e. Cerebral embolus
ENT].
Ans. The key is A. Subarachnoid hemorrhage.
[Sudden onset severe occipital headache and
800. An 8yo child presents with recurrent vomiting with stiffness makes the likely diagnosis
to be subarachnoid hemorrhage].
abdominal pain, occasional headaches but
maintains a good appetite. Exam: normal. 803. A 34yo housemaid presents with
CBC, BUE, etc are normal. headaches in the back of her head for
What would you do for her next? several days and pain on flexing her neck.
a. US abdomen What is the most likely cause?
b. CT head a. Subdural hemorrhage
c. Reassure b. Cervical spondylosis
d. Analgesics c. Subarachnoid hemorrhage
d. Meningitis
e. Cluster headache
Ans. The key is C. Reassure. [Recurrent abdominal
pain and headaches with no abnormal finding on Ans. The key is B. Cervical spondylosis. [Headache
examination and investigation points to in the back of head and pain on flexing neck is
Abdominal Migraine. Management is early feature of cervical spondylosis which
reassurance]. gradually progress to later symptoms like
radiculopathies due to root compression in arms
and hands].
801. A 78yo gentleman suddenly 804. A 40yo man complains of thirst and lethargy.
collapsed. His HR=120bpm, His BP=140/90mmHg, corrected Ca2+=3.7mmol/l.
BP=70/40mmHg. Exam: pulsatile mass in What is the most appropriate management at this
abdomen. What is the most appropriate stage?
a. IV fluids
dx?
b. Prednisolone
c. IV hydrocortisone 808. A 55yo lady with swelling on the
d. Calcium prep abdomen below the umbilicus on the right
side. What is the single most appropriate
Ans. The key is A. IV fluids. [Hypercalcemia is
treated by IV fluids].
lymph node?
a. External iliac LN
805. A 75yo man on digoxin develops weakness in b. Pre-aortic LN
the right upper and lower limbs which resolves c. Aortic LN
within a few hours. What is the most definitive d. Inguinal LN
inv for this condition? e. Iliac LN
a. Carotid Doppler f. Submental LN
b. Angiography g. Submandibular LN
c. CT head h. Deep cervical LN
d. Digoxin level
Ans. The key is D. Inguinal LN.
Ans. The key is B. Angiography. [Most definitive 809. A 66yo male presents with painful
angiography; initial or most appropriate dopplar].
swallowing. What is the most likely dx?
806. A 35yo man presents with balance
a. Nesseria meningitides
problems, headache, SNHL and loss of b. Cryptococcus neoformans
coreneal reflex on the left side. What is c. Candida albicans
the most definitive inv? d. Isospora belli
a. CT scan of internal auditory meatus
e. Mycobacterium avium
b. Nuclear imagine of the brain
c. MRI of internal auditory meatus Ans. The key is C. Candida albicans. [Oesophageal
d. MRI brain candidiasis].
Ans. The key is D. MRI brain. Wrong key! “Most 810. A 45yo lady complains of expressable
definitive” is MRI internal auditory meatus. galactorrhea, decreased libido and
[Probable diagnosis is left sided acaustic amenorrhea, weight gain and easy
neuroma. For this most definitive investigation is fatigue. Her serum prolactin levels are
MRI internal auditory meatus]. 1100um/l. what is the likely cause of
807. A 52yo man has a painful, red, hyperprolactinemia?
a. Hypothyroidism
photophobic right eye with slightly blurred
b. Stress
vision and watering for 3days. He has had
c. Pregnancy
no such episodes in the past. On slit lamp
d. Prolactin secreting pituitary tumor
examination there are cells and in the ant
e. PCOS
chamber and pupil is sluggish to react.
What is the single most appropriate
clinical dx?
a. Acute close-angle glaucoma Ans. The key is D. Wrong key. Correct key should
be A. Hypothyroidism. [ It is likely to be
b. Acute conjunctivitis
hypothyroidism because prolactin level in
c. Acute dacrocystitis
hypothyroidism may be 30-200ng/ml which
d. Acute iritis
means 630-4200 mIU/L. Though weight gain can
e. Corneal foreign body
occur but fatigue is not a feature of prolactinoma
Ans. The key is D. acute iritis. unless it is a macroadenoma causing suppression
of other pituitary hormone! given prolactin level
of 1100 IU/L is definitely from a microadenoma chest x-ray should be done to evaluate
(in macroadenoma prolactin level is > 10,000 respiratory problem first].
MU/L) which does not cause fatigue. So the case
most likely seems to be hypothyroidism]. 814. A 68yo man has had increasing
dysphagia for solid food for 3m and has
***811. A 42yo female had sudden severe lost 5kgs in weight. What single inv is
headache and vomiting. She took most likely to lead to a def dx?
paracetamol and then collapsed. What is a. Barium swallow
the most likely dx? b. CXR
a. SAH c. CT chest
b. Viral encephalitis d. Endoscopy and biopsy
c. Meningitis e. Video-fluoroscopy
d. Anaphylaxis
Ans. The key is D. Endoscopy and biopsy. [Likely
cause is malignant stricture for which endoscopic
biopsy is the definitive investigation].
Ans. The key is A. SAH. [Sudden headache,
vomiting, collapse are more suggestive of SAH]. 815. A 24yo male was trying to move his
812. Parents of a 3m baby are worried wardrobe but it fell on his thigh. It was
about cot death. What advice would you there for a very long time before someone
give? was able to remove it. When he was seen
a. Lay on the back with feet towards head in ED he had casts in his urine but no
end RBCs. Other inv showed hypocalcemia and
b. Lay on the back with feet towards feet high serum creatinine. What is the cause
end for his renal failure?
c. Lay on side a. Acetylcholine
d. Lay on stomach b. Myoglobin
c. Myotroponin
Ans. The key is B. Lay on the back with feet d. Acetyl acetate
towards feet end.
Ans. The key is B. Myoglobin. [The likely diagnosis
813. A child born at 36wks developed is Rhabdomyolysis where renal failure occurs due
difficulty in breathing with intercoastal to myoglobin (myoglobins are released after
recession and nasal flaring. His temp is breakdown of skeletal muscles)].
normal but his mother had PROM 48h
816. Anatomical structure to be pierced
ago.
during surgery midline port during
What is the most likely inv that will lead to
gallstone removal.
tx? a. External iliac muscle
a. Blood culture
b. Cricoid cartilage
b. CXR c. Linea alba
c. Stool culture d. Rectus sheath muscle
d. Sputum culture e. Duramater
f. 1st tracheal cartilage
Ans. B. CXR. [Probable pneumonia as there was g. Conjoined tendon
prom! In prematurity there may also occur h. Intercostal muscles
respiratory distress syndrome. So at this stage Ans. The key is C. Linea alba.
817. A 48yo man has continuous ant chest start N-acetylcysteine, stopping it if level below
pain which is worse on inspiration and has treatment line and INR/ALT normal. OHCM, 9 th
edition, page 856].
temp of 37.5C since 4wks after a MI. His
819. A pt dx with DVT is taking warfarin.
ESR=45mm/h.
What is the single most likely explanation What is his cut off INR limit?
for the abnormal inv? a. <1
a. Acute pericarditis b. 1-2
b. Cardiac tamponade c. 2-3
c. Atrial thrombus d. 3-4
d. Left ventricular aneurysm
e. Dressler syndrome Ans. The key is C. 2-3. [This range is appropriate
for the prophylaxis or treatment of venous
The key is E. Dressler syndrome. [Dressler thromboembolism and reduction of the risk of
syndrome usually occurring two to five weeks systemic embolism for people with atrial
after the initial event but it can be delayed for as fibrillation and valvular heart disease].
long as three months. It is characterised by
pleuritic chest pain, low-grade fever and 820. Inserting a drain in the mid-axillary
pericarditis]. line. What is the single most appropriate
anatomical structure?
818. An 8yo child swallowed 12 tablets of a. External iliac muscle
paracetamol 4h ago. Serum paracetamol b. Linea alba
levels when tested were at critical level. c. Rectus sheath muscle
What would you do next? d. Conjoined tendon
a. Activated charcoal e. Intercostal muscles
b. IV N-acetylcysteine
c. Gastric lavage Ans. The key is E. Intercostal muscles.
d. Observation only
821. A 29yo man presents with hx of
Ans. The key is B. IV N-acetylcystine. [As 4 hour cough, weight loss and night sweats.
paracetol level is at critical level start IV N- Exam: pansystolic murmur. What is the
acetyltine at once]. most appropriate dx of underlying cause?
a. Malaria
Manaement of paracetamol poisoning:
b. HSP
GI decontamination is recommended in those c. HIV
presenting <4h after OD. Activated charcoal 1g/kg d. Dengue fever
(max 50g) is the treatment of choice, reducing
serum levels more than gastric lavage and limiting Ans. The key is C. HIV. [IV drug abusers may have
liver injury. tricuspid regurgitation causing pansystolic
murmur and HIV also very common in this group]
• Glucose, U&E, LFT, INR, ABG, FBC, HCO3
–; blood paracetamol level at 4h post-ingestion. 822. A 15yo boy presents with recurrent
• If <10–12h since overdose, not vomiting, and breathlessness and wheeze especially
plasma paracetamol is above the line after exercise. What is the most diagnostic
on the graph, start N-acetylcysteine.
• If >8–24h and suspicion of large overdose
inv?
a. CXR
(>7.5g) err on the side of caution and
b. Lung function test
c. PEFR
d. CT scan dysfunction) and there may also cerebellar sign
like ataxia be present].
Ans. The key is B. Lung function test. [Assessment
of severity of airflow obstruction can diagnose
asthma and COPD]. ***825. A 67yo man being managed for a
malignancy develops neutropenic fever.
He has been commenced on Ticarcillin,
823. A 34yo man was walking along the
Tazobactam and Gentamicin. He has also
country side when an insect bit him. After
recently commenced on Meropenem but
which he started to complain of an
on the 3rd day his temp still remains >39C.
annular rash spreading upwards.
a. Penicillin PO
2 blood tests and urine cultures show no
b. Doxycycline PO organism. Inv: Hgb=104g/dl, WBC=<0.5,
c. Flucloxacillin PO Plt=15.
d. Gentamicin PO What will you do next?
e. Ciprofloxacin PO a. Continue IV antibiotics and add oral
f. Antihistamine PO antifungals
g. Antihistamine IV b. Continue antibiotics and add IV
h. Corticosteroid IV
antifungals
i. Corticosteroid IM
j. Adrenaline IM c. Stop antibiotics
k. Adrenaline IV d. Continue only present antibiotics
l. Atropine IV
m. Reassurance Ans. The key is B. Continue antibiotics and add IV
antifungals.
Ans. The key is B. Doxycycline PO. [Described 826. A young girl who is known to have
rash is erythema migrans which is characteristic T1DM presented with drowsiness and
skin lesion seen in lyme disease. Lyme disease is deep breathing. Her sugar level=20. Her
treated with Doxycycline]. BP=120/80mmHg and her mucous
membranes are dry. What would be the
824. A 60yo man was brought in by his next appropriate step?
a. Serum urea
wife complaining of ataxia, urinary
b. Blood culture
incontinence and erectile dysfunction. He
c. CT
also complains of rigidity and slowing of
d. HbA1c
movement with a pill rolling tremor of the
e. ABG
hands. What is the likely dx?
a. Parkinson’s disease Ans. The key is E ABG. [Likely iagnosis is DKA
b. Idiopathic parkinson’s disease where ABG is done to demonstrate acidosis].
c. Shy-drager syndrome
d. Huntington’s disease 827. Removal of a glioma, which single
most appropriate anatomical structure
Ans. The key is C. Shy-drager syndrome. [Shy- will be pierced?
drager syndrome is characterized by a. Cricoid cartilage
parkinsonism (rigidity, slowing of movement with b. Rectus sheath muscle
a pill rolling tremor of the hands) plus a more c. Duramater
pronounced failure of the autonomic nervous d. Conjoined tendon
system (urinary incontinence and erectile e. Intercostal muscles
Ans. The key is C. Duramater. syndrome) or primary HIV infection there is
generalized lymphadenopathy].
828. A child is not breathing and
intubation failed. At what anatomical site
should the incision be made? 831. A 6yo child presents with edema and
a. External iliac muscle
b. Cricoid cartilage
mild proteinuria. No hematuria. What is
c. Linea alba the most likely dx?
d. Rectus sheath muscle a. PSGN
e. Duramater b. Membranous GN
f. 1st tracheal cartilage c. Minimal change GN
g. Conjoined tendon d. RPGN
h. Intercostal muscles
Ans. The key is C. Minimal change GN. [Minimal
Ans. The key is B. Cricoid cartilage. [Incision for change GN is usually seen in young children. It
tracheostomy is done which goes down from presents as edema and proteinuria].
cricoids cartilage].
832. An 80yo woman suffering from RA
829. A 10yo child who presented with fx of presents with severe epigastric pain and
the radius which was treated with a vomiting. She also complains of shoulder
plaster cast, complains of pain. Exam: tip pain. What is the single most
limb is swollen but warm and well discriminatory inv?
perfused, pulses are present. What should a. US Abdomen
you do next? b. Sigmoidoscopy
a. Send for repeat XR c. Colonscopy
b. Remove cast d. Barium meal
c. Replace cast with more padding e. Upper GI endoscopy
f. Erect CXR
d. Give analgesic
Ans. The key is B. Between 7.8-11.0mmol/l. Ans. The key is B. XR sacro-iliac joints. [x-ray
sacroiliac joint shows fusion of both SI joints and
847. A young man who has no PMH thin, symmetrical syndesmophytes bridging the
presented with jaundice, low Hgb, retics intervertebral disc spaces].
8% and other indices WNL but occasional
spherocytes were seen on blood film.
What is the single most appropriate inv? ***850. A 4yo girl is taken by her mother
a. G6PD enzyme assay to the ED and complains of feeling unwell,
b. Direct coombs test urinary urgency and temp=39C. What is
c. Repeat blood film the single next best inv?
d. Indirect coombs test a. Catheter catch of urine
e. BMA b. Clean catch of urine
c. US
Ans. The key is B. Direct Coombs test. [The direct d. IVU
Coombs test, is used to determine whether the e. Suprapubic catch of urine
cause of hemolytic anemia, is due
to antibodies attached to RBCs which are seen in Ans. The key is B. Clean catch of urine. [The
autoimmune-related hemolytic anemia]. clinical features described are consistent with
urinary tract infection for which clean catch of
848. A 22yo man came to the hosp after urine is the next best investigation. Routine
an injury in his hand while playing investigations that are done in UTI are:
basketball. Exam: avulsion of extensor
Dipstick analysis of urine - may treat as
tendon from the distal phalanx.
bacterial if there are positive results for
What is the single most probable nitrite and/or leukocytes.
deformity? Urine microscopy - leukocytes indicate
a. Dinner fork deformity presence of infection.
b. Game keeper thumb Urine culture - to exclude the diagnosis or
c. Mallet finger if high-risk (eg, pregnant,
d. Gun-stock deformity immunosuppressed, renal tract anomaly
e. Garden spade deformi or if failed to respond to earlier empirical
treatment).
Ans. The key is C. Mallet finger. [A finger that An ultrasound evaluation of the upper
bends down at the end joint and cannot be urinary tract is recommended to rule out
straightened is called a mallet finger. It is caused urinary obstruction or renal stone disease
in acute uncomplicated pyelonephritis].
by an injury to the extensor tendon that
straightens (extends) the finger. A splint worn day
and night for 6-8 weeks will cure the problem in
851. A 2yo girl presents with a 4d hx of
most cases]. fever that started with a cough. Her
RR=45bpm, sat=94%, temp=38.9C, characteristic hyperintensity in fronto-temporal
capillary refill time=1s. There are region.
In the given case there are features of raised
crepitations at the left base on
intracranial pressure like drowsiness and seizer
auscultation. Urine shows negative and so we cannot proceed for LP unless guided by
dipstick. CT (or MRI). On the other hand there is no other
What is the single inv most likely to lead option that can be diagnostic of the given
to dx? condition. So we have to happy with CSF analysis
a. Blood for C&S as the key though at this moment we have to
b. ESR withheld this procedure].
c. CXR
853. A 6m boy is admitted with persistent
d. Urine for C&S
irritability. He is lethargic and is not
e. CSF analysis
feeding as well as usual. His RR=30bpm,
Ans. The key is C. CXR. [The features are sat=97%, temp=38.0C, capillary refill
consistent with RTI (probable pneumonia) for time=2s. Urine reveals leucocytes on
which CXR is the investigation of choice]. dipstick. What is the single inv most likely
to lead to dx?
a. Blood for C&S
852. A 3yo girl presents with fever for 2d.
b. ESR
She is drowsy and had a seizure causing c. CXR
twitching of the right side of the body for d. Urine for C&S
4mins. Her RR=30bpm, sat=90%, e. CSF analysis
temp=38.9C, capillary refill time=2s. Urine
negative on dipstick. Ans. The key is D. Urine for culture and sensitivity.
What is the single inv most likely to lead [Dipstick test if show leucocytes or nitrites is
to dx? suggestive of UTI. So to confirm it we should do
a. Blood for C&S urine C&S].
b. ESR
854. A 3yo boy presents with a 1d hx of
c. CXR
being unwell. He appears shocked and has
d. Urine for C&S
3h old rash made up of urticarial and
e. CSF analysis
purpural spots. His RR=30bpm, sat=94%,
Ans. The key is E. CSF analysis. [It is really very temp=39C, capillary refill time=1s. Urine is
difficult to differentiate between encephalitis and clean on dipstick.
meningitis. Encephalitis is mostly viral and in UK What is the single inv most likely to lead
herpes simplex virus is the main cause. Meningitis to dx?
also has a viral predominance though less than
a. Blood for C&S
encephalitis.
b. ESR
Because encephalitis involves infection of the
brain itself, symptoms of altered brain function-- c. CXR
like confusion or decreased alertness--are usually d. Urine for C&S
present, while in cases of meningitis the patient is e. CSF analysis
initially alert and, though understandably
distracted by pain and misery, still in command of Ans. The key is A. Blood for C&S. [Likely diagnosis
their mental processes. is septicemia for which Blood culture is the
As CSF can not differentia between meningitis investigation of choice].
and encephalitis we have to take help of imaging
like MRI. In herpes simplex encephalitis there is
855. A child is dx with VUR. What would What is the single most appropriate next
you tell his parents? action?
a. Local exam
a. Requires antibiotic prophylaxis b. Exam under GA
b. Most will require surgery c. Continue regular child care
c. Most will have kidney scarring by 5yo d. Inform child protection services
d. Nothing can be done e. Coag profile
e. Reassure
857. A 13yo girl complains of a 2d hx of 860. A 6yo child fell on his nose 2d ago.
hoarseness of voice a/w dry cough. She His parents have now brought him with
feels feverish. On direct laryngoscopy, her difficulty in breathing. Exam: fever, nasal
vocal cords are grossly edematous. bones are straight. What is the single
What is the single most appropriate inv? most likely dx?
a. None req a. Nasal polyp
b. Sputum for AFB b. Septal hematoma
c. Laryngoscopy c. Septal abscess
d. Bronchoscopy d. Deviated nasal septum
e. XR cervical spine e. Fx nose
Ans. The key is A. None required. Ans. The key is C. Septal abscess.
886. A 63yo female with a hx of 889. A woman with a prv hx of pain at the
osteoporosis suddenly falls on her left wrist following a fall 4m ago for which
outstretched hand while shopping. XR she didn’t seek any tx now presented with
shows fx at distal radius with backward pain in the same wrist below the thumb
shift of the distal fragment. What is the and the pain is aggravated whenever she
single most probable deformity? holds her baby. What is the cause?
a. Dinner fork deformity a. Fx radial head
b. Coxavara b. Scaphoid fx
c. Mallet finger c. Carpal tunnel syndrome
d. Cubitus valgus d. Colles fx
e. Garden spade deformity e. Ulnar fx
887. A 60yo man presents with severe Ans. The key is B. Scaphoid frature.
colicky pain from his right flank radiating
to his groin. His urinalysis reveals trace
blood cells. What is the single most 890. A 29yo man was involved in an RTA.
discrimatory inv? He presents with distended neck veins,
a. US abdomen clear breath sounds and a trachea which
b. XR KUB is in the midline. His RR=34bpm,
c. Colonoscopy BP=60/0mmHg. What is the most likely
d. Upper GI endoscopy dx?
e. Laproscopy a. Simple pneumothorax
b. Tension pneumothorax
Ans. The key is A. US abdomen. [Features are of c. Cardiac tamponade
ureteric colic. X-ray KUB may miss radiolucent d. Pericarditis
stones so US abdomen is the discriminatory inv].
888. A 45yo man has been admitted for an Ans. The key is C. Cardiac tamponade. [Distended
elective hernia surgery. 3d later he neck vein, clear breath sound and no tracheal
presents with agitation, sweating, shift and BP of 60/0 points towards cardiac
aggressiveness, and complains of seeing tamponade].
snakes on the hosp wall. Chlordiazepoxide
has been started for this pt. What is the
891. An elderly woman is found anemia.
most appropriate next step?
a. Diazepam
As part of her exam, she had a barium
b. Acamprosate
enema which reveals a mass lesion in the
c. Disulfiram
ascending colon. What is the single most
d. Thiamine appropriate dx?
a. Sigmoid volvulus
b. Anal fissure
c. Sigmoid carcinoma
d. Cecal carcinoma digoxin and isosorbide mononitrate. What drug is
e. Diverticular disease going to help him?
a. Ramipril
b. Bendroflumethiazide
c. Atenolol
Ans. The key is D. Cecal carcinoma. [Mass in
assending colon and anaemia makes cecal d. Amlodipine
carcinoma the likely diagnosis from the given e. Diltiazem
options].
892. A 55yo male after gastrectomy Ans. The key is B. Bendroflumethiazide. [As
developed anemia. His MCV=106fl. Exam: patient is still in heart failure and pulmonary
loss of proprioception and vibration sense. edema with the usage of loop diuretic...a thiazide
What is the most likely dx? diuretic could be added with loop when response
a. IDA is inadequate( Refernce;patient.info) this will
b. Folate def reduce the volume overload and improve
c. Vit B12 def breathing].
d. Anemia of chronic disease
Ans. The key is A. Fx neck of femur. [Use of Ans. The key is A. Diaphramatic rupture.
alendronate indicates osteoporosis where 902. A 62yo man dx with T2DM with
fracture neck of femur is more common].
BMI=33. Lifestyle modifications have
899. A young male met with a RTA and is failed to control blood sugar. Labs:
suspected to have a femur fx. His BP is urea=3.6mmol/l, creatinine=89mmol/l.
90/60mmHg. What is the next immediate what is the next appropriate
action? management?
a. XR a. Biguanide
b. IV fluids b. Sulfonylurea
c. Put leg splint c. Insulin
d. Glitazone
e. Sulfonylurea receptor binder tenderness. Exam: tympanic membance
normal. Aural toilet has been done. What
Ans. The key is A. Biguanide. [Patient is obese
type 2 diabetic. So biguanide is the tx of choice].
is the next appropriate med?
a. Antibiotic PO
903. A pt presents with progressive b. Antibiotic IV
dyspnea. He complains of cough, wheeze c. Steroid PO
and a table spoonful of mucopurulent d. Steroid drop
sputum for the last 18m. Spirometry has e. Antibiotic drop with steroid
been done. FEV1/FVC=2.3/3.6. After
Ans. The key is E. Antibiotic drop with steroid.
taking salbutamol, the ratio=2.4/3.7. [Discharge from ear and tragal tenderness are
What is the most likely dx? features of otitis externa. Key treatment is aural
a. Chronic bronchitis toileting. Drop advised is Sofradex (Framycetin +
b. Asthma dexamethasone) OHCS, 9th edition, page 542].
c. Bronchiectasis
d. Lung fibrosis 906. A 23yo man sprained his right ankle
e. Sarcoidosis 6wks ago while playing football. He was tx
with a below knew walking cast. On
Ans. The key is A. Chronic bronchitis. [Progressive removal of the cast, the pt noted to have
dyspnoea, wheeze productive cough and the right foot drop. He has weakness of
result of spirometry points towards the diagnosis
extensors of the ankle and toes and
of chronic bronchitis].
diminished pin prick sensation over the
904. A 62yo man presents with cough, dorsum of the foot. The ankle jerk is
breathlessness and wheeze. 24% O2, present and plantar reflex is flexor.
salbutamol and hydrocortisone were What is the most likely cause of the foot
given. The symptoms haven’t improved drop?
and so nebulized bronchodilator was a. Compression of common peroneal nerve
repeated and IV aminophylline was given. b. Compression of the tibial nerve
ABG: pH=7.31, RR=32. What is the next c. Compression of the S1 nerve root
appropriate management? d. Rupture of Achilles tendom
a. Nasal IPPV e. Tx of the medial collateral lig of the ankle
b. Intubation and ventilation
Ans. The key is A. Compression of common
c. LABA
peronial nerve. [Foot drop, weakness of extensors
d. Toxapram
of the ankle and toes and diminished pin prick
e. Amoxicillin PO sensation over the dorsum is sugestive of
compression of common peroneal nerve].
Ans. The key is A. Nasal IPPV. [Here given case is
COPD has following indications of Nasal IPPV i) 907. A young man was knocked down
Tachypnea (>24 breaths/min) and ii) during a fight in the waiting room of the
Hypercapnic respiratory acidosis (pH range 7.10-
ED. He is now unconscious and
7.35)].
unresponsive. What is the 1st thing you
would do?
a. Turn pt and put in recovery position
905. A young girl returns from holidays in b. Put airway
Spain. She complains of discharge from c. Endotracheal intubation
her ear and complains of tragal d. Assess GCS
e. Start CPR Ans. No key is given!! Correct answer is D. Large
cell ca. [Large cell carcinoma is, by definition, a
Ans. The key is B. Put airway. [ABC protocol]. poorly differentiated malignant epithelial tumor.
It consists of sheets or nests of large polygonal or
908. A 52yo man underwent a
giant multinuclear cells and probably represents
hemicolectomy. After a few days he
SCC ."]
complains of left ventricular pain and
fever. ECHO has been done and shows a 911. A 27yo man presents with chest pain
systolic murmur. and respiratory distress. Exam:
What is the next appropriate inv? tachycardia, hypotension and neck vein
a. CT distension. Trachea is deviated to the left
b. US side, breathing sounds on right side are
c. CXR absent and diminished on left side.
d. Blood culture What is the next appropriate
e. LFT management?
a. CXR
Ans. The key is D. Blood culture. [Dx a case of
b. Right side aspiration (16G)
infective endocarditis. Therefore the next
c. Left side aspiration (16G)
appropriate investigation is blood culture].
d. Right side drain with a small tube (12F)
909. A 19yo man has exercised induced e. Left side drain with a small tube (12F)
asthma and is using a salbutamol inhaler
as req and beclamethasone 400ug BD. He
Ans. The key is B. Right side aspiration (16G). [The
complains that he has to wake up at night features described is diagnostic of right sided
for his inhaler. What is the single most tension pneumothorax. Next appropriate
appropriate tx? management is To remove the air, insert a large-
a. Beclo bore (14–16G) needle with a syringe, partially
b. Regular salbutamol and budesonide filled with 0.9% saline, into the 2nd intercostal
c. Sodium cromoglycate interspace in the midclavicular line on the side of
the suspected pneumothorax. Remove plunger to
d. Oral steroid
allow the trapped air to bubble through the
e. Inhaled steroid syringe (with saline as a water seal) until a chest
tube can be placed. Alternatively, insert a large-
bore Venfl on in the same location OHCM, 9th
Ans. The key is C. Sodium cromoglycate. edition, page 824].
913. A 68yo woman dx with T2DM and Ans. The key is E. None. [A case of chickenpox.
BMI=33. Lab: GFR=29, urea=13, None of the given treatment is used in
creatinine=390mmol/L. what is the next chickenpox. Symptomatic treatment like,
appropriate management? acetaminophen if fever, antihistamine and
a. Biguanide calamine lotion is given].
b. Sulfonylurea
c. Insulin
915. A woman came with the complaint of
d. Glitazone
pain in her right arm when she abducts it.
e. Sulfonylurea receptor binder She has recently moved to a new house.
There is no history of trauma.
What is the likely cause of her pain?
a. Rupture of the long head of biceps
Ans. The key is C. insulin. [Insulin is devoid of
b. Sprain of the acromio-clavicular ligament
significant side effect than Glitazones (like fluid
retention). In renal failure there is reduced GFR c. Tendinitis of the abductor sheat
and some fluid retention. It is not desirable that d. Supraspinatus tendinitis
glitazone to cause more fluid retention by causing e. Shoulder dislocation
oedema. The oral agents that are thought to be
relatively safe in patients with nondialysis CKD
include short-acting sulfonylureas (eg, glipizide)
Ans. The key is D. Supraspinatus tendinitis.
and repaglinide. If an oral agent is used, the
short-acting sulfonylurea, glipizide, is the [Tendinitis and partial tears in the supraspinatus
preferred agent among nondialysis CKD patients tendon causes a ‘painful arc’ since as the person
who have an estimated glomerular filtration rate elevates his arm sideways, the tendon begins to
(eGFR) <30 mL/min/1.73. The dose for glipizide is impinge under the acromion throught the middle
2.5 to 10 mg/day. Glyburide and other long-acting part of the arc, and this is usually relieved as the
sulfonylureas are generally not recommended in arm reaches 180 degrees (vertical)].
any CKD patient with type 2 diabetes, because of
the risk of hypoglycemia. Some clinicians
recommend the use of the meglitinide
repaglinide (starting with a dose of 0.5 mg) for
916. An 83yo man with longstanding
nondialysis CKD patients since these agents are COPD has become progressively
not renally cleared. Nondialysis CKD patients with breathless over the last 2yrs. He is on
type 2 diabetes may be treated with an oral salbutamol, ipratropium, salmetarol,
agent, although many patients end up on insulin beclomethasone and theophylline. His
therapy because it is more effective. So it may be
FEV1<30%.
that we can go for insulin as the answer!!].
What is the next appropriate
management?
914. A 5yo boy was brought to GP with a. Lung transplant
high temp and many vesicles on his back. b. Trial of CPAP
What is the most appropriate c. Trial of non-invasive ventilation
management? d. Assessment for long term O2 therapy
a. Topic acyclovir e. Short course of O2 therapy
Ans. The key is D. Assessment for long term O2 918. A man had a soft mass on his
therapy. [Patient is progressively breathless with mandible. Mass is freely mobile and has
present FEV1 of <30%. So his respiratory
started growing progressively over the
deterioration indicates progressive respiratory
failure for which he should be assessed for long past 6m. The mass still moves freely.
term O2 therapy. Long-term oxygen therapy What is the best inv for this pt?
(LTOT) for more than 15 h/day improved a. FNAC
mortality and morbidity in a well-defined group of b. CT
patients with chronic obstructive pulmonary c. XR
disease. Requirement of condition to proceed to d. MRI
LTOT is patient should be stable and on
e. ESR
appropriate optimum therapy (as in given case)
and having stopped smoking tobacco. Patient
should be shown to have a PaO2 less than 7.3 kPa Ans. The key is A. FNAC.
and/or a PaCO2 greater than 6 kPa on two
occasions at least 3 weeks apart. FEV1 should be
less than 1.5 litres, and there should be a less 919. A 63yo man has been brought to the
than 15% improvement in FEV1 after
bronchodilators. Patients with a PaO2 between
hosp after collapsing during a wedding.
7.3 and 8 kPa who have polycythaemia, right His ECG is below. What is the most likely
heart failure or pulmonary hypertension may gain dx?
benefit from LTOT].
For most patients, antibiotics have little Ans. The key is B. None.
effect on the duration of the condition or [Stones less than 5 mm in diameter pass
the severity of symptoms. The National spontaneously in up to 80% of people.
Institute for Health and Care Excellence Stones between 5 mm and 10 mm in
(NICE) suggests that indications for diameter pass spontaneously in about
antibiotics include:[1] 50% of people.
Stones larger than 1 cm in diameter
Features of marked systemic usually require intervention (urgent
upset secondary to the acute sore intervention is required if complete
throat. obstruction or infection is present).
Two thirds of stones that pass
Unilateral peritonsillitis.
spontaneously will do so within four
A history of rheumatic fever. weeks of onset of symptoms]. Ref:
patient.info
An increased risk from acute 932. A 4yo boy presents with fever, severe
infection (such as a child with ear ache, vomiting and anorexia. He also
diabetes mellitus or
has mod tonsillitis. Exam: tympanic
immunodeficiency).
membrane bulging. He came to the GP a
Acute tonsillitis with three or few days ago and was dx with URTI. What
more of the following Centor is the most appropriate dx?
criteria present: a. OE
b. Acute OM loss. CXR shows bilateral fibrosis and left
c. Serous otitis side pleural effusion.
d. Chronic suppurative OM What is the best inv that will lead to dx?
e. Mastoiditis a. CXR
b. Pleural fluid aspiration of cytology
c. MRI
Ans. The key is B. Acute OM. [High fever, severe d. Pleural biopsy
earache, vomiting, bulging tympanic membrane e. CT
and H/O associated URTI is highly suggestive of
acute OM].
Ans. The key is D. Pleural biopsy. [There is
asbestos exposure in builders and associated
smoking greatly increases the possibility of
933. A 3yo girl presents with complains of
developing mesothelioma and the given
sudden right facial weakness and presentation is typical of mesothelioma. Best
numbness and pain around her ear. There investigation is pleural biopsy].
are no symptoms. What is the most
appropriate dx?
a. SAH 936. During a basketball match, one of
b. Bell’s palsy the players suddenly collapsed to the
c. Stroke ground with coughing and SOB. What is
d. TIA the inv of choice?
e. Subdural hemorrhage a. CXR
b. CT
c. MRI
Ans. The key is B. Bell’s palsy. d. V/Q scan
e. CTPA
934. A 6yo boy fell in the playground and Ans. The key is A. CXR. [Likely diagnosis is
has been holding his forearm complaining pneumothorax. So investigation of choice is CXR].
of pain. Exam: no sign of deformity or
swelling. However, there is minimal
937. A 57yo man having HTN on oral anti-
tenderness on exam. What is the dx?
HTN. However, he is finding it difficult to
a. Fx mid radius
mobilize as he feels dizzy whenever he
b. Fx mid ulnar
c. Fx neck of humerus
tries to get up. What is the most
d. Fx shaft of humerus
appropriate inv for him?
a. Ambulatory BP
e. Green stick fx of distal radius
b. ECG
c. MRI
d. CXR
Ans. The key is E. Green stick fx of distal radius.
e. CT
935. A 62yo man has been smoking about
15 cigarettes/day for 45yrs, and has been Ans. The key is A. Ambulatory BP. [Ambulatory BP
working as a builder since he was 24yo. to document low BP as cause of presenting
He presents with chest pain, SOB, weight symptom. The case seems to be of postural
hypotension and low BP as a result of given anti c. Lateral collateral
hypertensive]. d. Post cruciate
e. Meniscus
938. A 33yo female complains of diplopia
on upright gaze. Exam: ptosis can be seen.
There are no other complains or any Ans. The key is B. Medial collateral. [The valgus
stress test involves placing the leg into extension,
significant PMH. What is the most
with one hand placed as a pivot on the knee.
appropriate inv for him?
With the other hand placed upon the foot
a. Ophthalmoscopy
applying an abducting force, an attempt is then
b. Visual field test
made to force the leg at the knee into valgus. If
c. TFT the knee is seen to open up on the medial side,
d. CT this is indicative of medial collateral ligament
e. Checking red reflex damage].
Ans. The key is B. IV calcium gluconate. [In severe Ans. The key is D. Gangrene.
hyperkalemia IV calcium gluconate is given to
protect the heart from cardiac arrest or life-
threatening arrythmias]. 948. After surgery a pt’s left leg has
become swollen and tender. The diameter
of the calf has increased and passive
movements cause pain. What is the most Ans. The key is A. Paracetamol. [Pain on
probable dx? movement, pain worse at end of day and
a. DVT decreased joint space are characteristic of
b. Lymphedema osteoarthritis supported by no significant raise in
c. Peripheral vascular disease inflammatory marker (CRP=12). 1st line medicine
is paracetamol].
d. Hematoma
e. Superficial thrombophlebitis 951. A 68yo man presents with muscle
weakness. He is not able to climb stairs.
He also complains of mild breathlessness.
Ans. The key is A. DVT. [Swelling, tenderness and He says that he sometimes feels difficulty
enlarged calf diameter are features of DVT in swallowing food. Labs: ALP=216,
supported by positive Homan’s test (pain on
AST=49, ALT=43, CK=417, ESR=16.
passive movement)].
What is the most likely dx?
949. 2h after an appendectomy, a pt a. Polymyositis
complains of a rapid HR and fever. He says b. Polymyalgia rheumatic
there is also abdominal pain and pain in c. Muscular dystrophy
the shoulder area. What is the first step in d. Esophageal carcinoma
the management? e. Osteoarthritis
a. Maintain IV access and give IV fluids
b. Start IV antibiotics
c. Insert NGT for intestinal decompression Ans. The key is A. Polymyositis. [Proxymal
d. Cross match blood myopathy (not able to climb stairs), mild
e. Emergency exploratory laparotomy breathlessness (involvement of thoracic muscles
or interstitial lung disease), dysphagia due to
involvement of the oropharyngeal striated
muscles and upper oesophagus)and raised CK
Ans. The key is A. Maintain IV access and give IV
suggests the diagnosis of Polymyositis].
fluids. [Features given are of internal bleeding! So
maintain iv access and giving iv fluid is the first
step in management of this patient].
952. A 67yo builder presents with a
persistent nodular lesion on upper part of
950. A pregnant woman presents with pinna with some telangiectasia around
knee pain on movements. The pain the lesion. What is the dx?
a. Basal cell
becomes worse at the end of the day.
b. Squamous cell
Radiology shows decreased joint space.
c. Keratocanthoma
Labs: CRP=12.
d. Actinic keratosis
What is the 1st line med?
e. Bowens disease
a. Paracetamol
b. NSAIDs
c. Oral steroid
Ans. The key is A. Basal cell ca. [Any ulcer which is
d. Intra articular steroid
located above the neck is always basal cell
e. DMARDs
carcinoma until proven otherwise. Source:
SAMSON notes].
953. A 68yo pt wakes up with slurred 355. A 24yo woman known to be suffering
speech and right sided weakness. CT from panic disorder presents to the
shows cerebral infarct. What is the most hospital with tingling and numbness in her
appropriate tx? fingers. ABG: pH=7.52, PCO2=2.2kPa,
a. Aspirin PO2=11kPa, Bicarb=20. What is the most
b. Alteplase likely condition?
c. Warfarin a. Acute metabolic alkalosis
d. Clopidogrel b. Acute resp alkalosis
e. Dipyridamole c. Compensated resp alkalosis
d. Compensated metabolic acidosis
e. Acute metabolic acidosis
Ans. The key is B. Alteplase. It is a wrong key! The
correct key is A. Aspirin. [The window period to
administer alteplase is 4.5 hours. If we cannot Ans. The key is B. Acute respiratory alkalosis. [In
certain this period we cannot proceed for panic attack there occurs hyperventilation which
alteplase]. causes washout of CO2 and leads to acute
respiratory alkalosis resulting in raised pH >7.45
(here 7.52), low PCO2 (here 2.2 kPa) with
954. A 73yo man who is recovering from compensatory decrease in HCO3 (here 20meq/l)].
surgery on the left carotid artery in his
neck. He has slurred speech. On protusion
of his tongue, the tip deviated to the left. 956. A 65yo man on dexamethasone
What is the single most appropriate underwent surgery. During and after the
option? surgery, his blood glucose was around 17-
a. Accessory nerve 19mmol/l. What will you give the pt?
b. Facial nerve a. Insulin
c. Glossopharyngeal nerve b. Oral hypoglycemic
d. Hypoglossal nerve c. Remove dexamethasone
e. Vagus nerve d. IV Saline
e. IX dextrose
960. Post gastric ulcer got perforated Ans. The key is C. COCP. It is a wrong key! Correct
leading to bleeding involving the gastro- key should be E. Clonidine. [In a history of MI
Oestrogen and COCP better avoided. Evening
duodenal artery. Where would fluid
primrose is not used in post menopausal
accumulate in the cavity?
symptoms. Raloxifene is a SERM which rather
a. Left paracolic gutter
worsen hot flash and vasomotor syndrome. On
b. Pelvic cavity
the other hand clonidine is the best option which
c. First part of duodenum
improves hot flash and vasomotor symptoms].
d. Under the diaphragm
e. Retroperitoneal
963. A 73yo man who was a smoker has
quit smoking for the past 3yrs. He now
Ans. D. Under the diaphragm. [Correct option
presents with hoarseness of voice and
probably A. Left paracolic gutter].
cough since past 3wks. XR: mass is visible b. Refer to social services
in the mediastinum. What is the best inv c. Refer to psychology
to confirm the dx? d. CBT
a. Bronchoscopy
b. Thoracoscopy
Ans. The key is B. Refer to social services.
c. US
d. CT thorax
e. LN biopsy 966. A young girl presented to OBGYN
assessment unit with lower abdominal
pain and per vaginal bleeding after a hx of
Ans. The key is E. LN biopsy. hysterosalpingograph as a part of her
infertility tx. Observation:
BP=90/50mmHg, pulse=120bpm, exam
964. A 52yo man known DM presents to revealed rigid abdomen.
ED with sudden onset of pain in the left What is the most appropriate next inv?
loin and hematuria. Inv: 8mm stone in left a. CT
lower ureter. Nifedipine with steroids was b. XR erect and supine
prescribed as initial tx with supportive c. US abdomen
therapy. He returned complaining of d. Coag profile
worsening pain, vomiting with passing of e. CXR
2 stones. Renal function tests indicate
impending ARF. Ans. The key is C. US abdomen. [Likely cause of
How will you manage this pt? bleeding and shock is ruptured fallopian tube for
a. Continue same tx which appropriate next investigation is US
b. Start alpha blocker abdomen].
c. ESWL
d. Percutaneous nephrolithotomy
967. A 21yo woman who is on COCP had
e. Percutaneous nephrostomy
to take azithromycin. What should be
f. Open surgery
advised for her contraception?
a. Using 7d condoms after antibiotics and
avoid pill free break
Ans. The key is E. Percutaneous nephrostomy. b. Using 14d condoms after antibiotics and
[Percutaneous nephrostomy, or
avoid pill free break
nephropyelostomy, is an interventional
c. Using 7d condoms after antibiotics
procedure that is used mainly in the
d. No extra precaution
decompression of the renal collecting system.
e. Using 14d condoms after antibiotics
Percutaneous nephrostomy catheter placement
has been the primary option for the temporary
drainage of an obstructed collecting system. Here
impending ARF indicates obstructive uropathy]. Ans. The key is D. No extra precaution. [Before it
was thought that antibiotics like azithromycin
965. A lady who is alcohol dependent inhibits the enzyme and reduce the efficacy of
wants to quit but wants someone to COCP. But later it was established that practically
encourage her. What would you do? no significant changes occur and so no need of
a. Medication any extra precaution].
c. Increased risk of osteoporosis
d. Increased risk of ovarian ca
968. A 60yo woman presented with radial
fx and had a colle’s fx and supracondylar Ans. The key is A. Increased incidence of
fx in the past. What inv is req to detect her endometrial carcinoma.
possibility of having the same prb later?
a. Dexa scan
971. A 45yo male complains of tremors in
b. MRI
hands. Exam: tremors are absent at rest
c. Nuclear bone scan but present when arms are held
d. CT outstretched and persist on movement.
e. Bone biopsy What is the most probable dx?
a. Parkinsonism
Ans. The key is A. Dexa scan. [The likely cause of b. Benign essential tremor
these multiple fracture is osteoporosis (post c. Cerebellar disease
menopausal women) for which we should do d. Liver failure
Dexa scan to establish the diagnosis]. e. Stroke
981. An 8yo boy has his tonsils and Ans. The key is A. Urgent exploration. [As sudden
adenoids removed. On the 7th post-op onset of severe pain likely diagnosis is torsion of
day, he comes back to the ED with testis. (infection takes a more prolonged course).
Next step is urgent exploration].
hemoptysis and fever. What is the most
appropriate management? 984. An 8wk pregnant woman presents
a. Admit for IV antibiotics
with persistent vomiting and weight loss.
b. Prescribe oral antibiotics and discharge
Exam: HR=110bpm. Dehydration was
c. Packing
corrected with NS infusion and K+. The
d. Surgery
e. Reassurance
condition didn’t improve so IM cyclizine
was given. She is still vomiting.
What is the next appropriate
Ans. The key is A. Admit for IV antibiotic. [Most management?
secondary hemorrhage occurs due to infection a. IV fluids
which erodes a vessel. So patient should be b. IV antiemetics
admitted for IV antibiotics]. c. IV steroids
982. A 50yo female had swelling in her d. Terminate pregnancy
e. Thiamine
ankles. She is a known alcoholic. Now she
presented with breathlessness and
palpitations.
What is the most likely cause of her Ans. The key is C. IV Steroid. [Steroids may be
used in patient's of hyperemesis gravidarum
condition?
refractory to standard therapy].
a. VT
b. SVT
c. A-flutter 985. A 28yo lady presents with
d. A-fib dyspareunia and dysmenorrhea. She is
very obese. She now wants reversible patients are tall males. So the likely diagnosis is
contraceptive method. Turner’s syndrome].
Which of the following will be most
suitable for her?
a. Minera
988. A 32yo woman wants reversible form
b. COCP
of contraception. She has one child
c. POP
delivered by emergency C-section. She
d. Copper T
also suffers from migraine and heavy
e. Barrier method
periods.
What is the most suitable form of
contraception for this lady?
Ans. The key is A. Mirena.
a. COCP
b. Mini pill
c. IUCD
986. A young lady who is 28wks pregnant d. Barrier method
presents with vaginal bleeding. She has e. Abstinence
lost about 200ml of blood. Exam: uterus is
tender. Resuscitation has been done.
What is the most imp inv to establish the Ans. The key is C. IUCD. [In migraine can not give
dx? COCP. In menorrhagia mirena is effective. So the
a. US answer is IUCD (mirena coil)].
b. CT
c. D-dimer 989. A 45yo known hypertensive man
d. Clotting profile presents with hematuria, proteinuria and
e. None edema. What is the definitive dx test for
him?
a. Urine protein
Ans. The key is A. US. [Antepartum hemorrhage. b. Renal biopsy
Most important investigation to establish the dx c. Renal function test
is US]. d. Urine microscopy
e. Serum protein
993. A 12m child with AIDS is due for his 996. A 64yo man complains of increasing
MMR vaccination. What is the single most SOB and cough for the past 18m. He
appropriate action? coughs up a Tbsp of mucopurulent sputum
a. Defer immunization for 2wks with occasional specks of blood.
b. Don’t give vaccine What is the most likely underlying cause?
c. Give half dose of vaccine a. Acute bronchitis
b. Bronchiectasis
c. Chronic bronchitis b. Fluphenazine
d. Lung cancer c. Haloperidol
e. Pneumonia d. Paraldehyde
e. Risperidone
998. A 32yo man with schizophrenia and a Ans. There are two keys A. BPH and C. Prostatic
carcinoma. [There are no constitutional features
hx of violence and distressing auditory
of carcinoma. Bleeding is more common in BPH
hallucinations was admitted to the ward
and occurs in a minority of cases and much less
with aggressive behavior and has already common in prostatic carcinoma. Features given
smashed his room. He is refusing any oral are of prostatism only which favours the
meds. What is the single most appropriate diagnosis of BPH].
injection?
a. Flupenthixol
1000. A 60yo man presents with mass in a. Thyrotoxicosis
the groin. Exam: mass lies below the b. Hyperthyroidism
midpoint of the inguinal ligament and is c. Vocal cord nodules
pulsatile. What is the most probable dx? d. Carcinoma bronchus
a. Direct inguinal hernia e. TB
b. Saphenavarix
Ans. The key is E. TB.
c. Femoral hernia
d. Irreducible hernia 1003. A 30yo woman presents with acute
e. Femoral aneurysm headache. She complains of seeing halos
especially at night. What is the single
Ans. The key is C. Femoral hernia. Controvertial
key! May be femoral aneurism . [Hernia below
most likely defect?
a. Paracentral scotoma
pubic tubercle (below inguinal ligament) is
b. Mono-ocular field loss
femoral hernia. But below mid-inguinal point and
pulsatile mass may be femoral aneurism as c. Tunnel vision
well!!]. d. Central scotoma
e. Cortical blindness
1001. An 82yo man has woken up with
incoherent speech and difficulty in finding Ans. The key is C. Tunnel vision. [The diagnosis is
the right words. Exam: otherwise normal, glaucoma (headache and haloes especially at
night). In glaucoma there occurs tunnel vision].
good comprehension.
Which anatomical site is most likely to be 1004. A 35yo man presents with a
affected? headache that worsens on bending his
a. Broca’s area head forward. What is the most likely dx?
b. Wernicke’s area a. Chronic sinusitis
c. Midbrain b. SAH
d. Parietal cortex c. Migraine
e. Pons d. Cluster headache
e. Tension headache
Ans. The key is A. Brocha’s area. [A person with
expressive aphasia will exhibit halting and Ans. The key is A. Chronic sinusitis.
effortful speech. Speech may only include
important content words. Word comprehension 1005. A 20yo man presents with painful
is preserved. The person may still be understood, swallowing. Exam: trismus and unilateral
but sentences will not be grammatical. This enlargement of his tonsils. The
contrasts with receptive or Wernicke’s aphasia, peritonsilar region is red, inflamed and
which is distinguished by a patient's inability to swollen.
comprehend language or speak with
What is the most appropriate tx?
appropriately meaningful words though fluency
a. Oral antibiotics
may be preserved].
b. IV antibiotics and analgesics
1002. A 25yo woman has a recent cough, c. I&D with antibiotics
hoarseness and swelling in the neck. There d. Analgesics with antipyretics
are several nontender swellings on both e. Tonsillectomy
sides of her neck. She has lost 13kgs. She
Ans. The key is C. I&D with antibiotics. [The dx is
takes recreational drugs. What is the most peritnsillar abscess and tx is I&D with antibiotics].
probable dx?
1006. A 40yo manual worker presents Ans. The key is C. Cerebellum. [Difficulty speech
with a swelling in the groin. Exam: mass is (cerebellar dysarthria) and nystagmus are
found to be just above and lateral to the suggestive of cerebellar lesion].
pubic tubercle. It is reducible. On applying 1009. A 75yo man presents with Bell’s
pressure on the internal ring there is no palsy. His PMH is significant for late onset
cough impulse seen. What is the most asthma and heartcfailure. He also reports
probable dx? to have consulted his GP for generalized
a. Direct inguinal hernia
rash prv. CXR: multiple soft shadows and
b. Indirect inguinal hernia
CBC: eosinophilia.
c. Femoral hernia
What is the single most likely positive
d. Strangulated hernia
antibody?
e. Femoral aneurysm a. P ANCA
Ans. The key is B. Indirect inguinal hernia. b. C ANCA
[Swelling in the groin; mass just above and lateral c. Anti Ro
to the pubic tubercle means inguinal hernia. It is d. Anti DS DNA
reducible. On applying pressure on the internal e. Anti centromere
ring there is no cough impulse; it means the
hernia enters through deep ring, and enters Ans. The key is A. p-ANCA. [The dx is Charg
scrotum passing through the superficial ring. That Strauss Syndrome (CSS).
means it is indirect inguinal hernia].
1007. A 34yo male presents with There are six criteria for dx of CSS:
headache and vomiting. Exam:
1. Asthma (wheezing, expiratory rhonchi).
temp=38.5C, neck stiffness, discharge 2. Eosinophilia of more than 10% in
from left ear and right sided hyper-reflexia peripheral blood.
with an extensor plantar response. What 3. Paranasal sinusitis.
is the most likely dx? 4. Pulmonary infiltrates (may be transient).
a. Cerebral tumor 5. Histological confirmation of vasculitis
b. Meningitis with extravascular eosinophils.
6. Mononeuritis multiplex or
c. Cerebellar tumor
polyneuropathy.
d. Cerebral abscess The presence of four out of six of these features
e. Normal pressure hydrocephalus has a high specificity and sensitivity for the
diagnosis of CSS.
Ans. The key is D. Cerebral abscess.
In the given case there are 1. Bell’s palsy
1008. A 26yo male presents with speech (mononeuritis multiplex) 2. Asthma 3. Multiple
difficulties. Exam: nystagmus. Which soft shadows on CXR (pulmonary infiltrates) 4.
anatomical site is most likely to be Eosinophilia. Presence of these 4 features are
affected? diagnostic of CSS. In CSS 70% patient is p-ANCA
a. Midbrain positive!].
b. Pons
c. Cerebellum 1010. A 50yo man complains of visual
d. Cerebrum prbs and dull pain in the left eye.
e. Vestibule cochlear nerve Fundoscopy reveals papilloedema. He was
dx with MS 2yrs ago. There is no
consensual light reflex of the right eye. Cefotaxime if <55yrs and Cefotaxime + Ampicillin
What is the single most likely defect? if >55yrs ].
a. Paracentral scotoma
1013. An 89yo pt has lung cancer. His
b. Mono-ocular field loss
Na+=122mmol/l. What is the tx for this?
c. Homonymous upper quadrantanopia
7. Demeclocycline
d. Central scotoma
8. Vasopressin
e. Homonymous lower quadrantanopia
9. Restrict fluids
Ans. The key is B. Mono-ocular field loss. 10. Reassure
1011. A 54yo pt wakes up with right sided Ans. There are two keys. A. Demeclocycline and
weakness. His current medication is C. Restrict fluids. *both are correct!![•
Asymptomatic chronic hyponatraemia fluid
bendroflumethiazide for HTN.
restriction is often suffi cient if asymptomatic,
Pulse=92bpm, BP=160/90mmHg. CT
although demeclocycline (ADH antagonist) may
shows left cerebral infarct. What is the be required. Ref: OHCM, 9th edition, page 686].
most appropriate tx?
a. Alteplase
b. Aspirin 1014. A 25yo woman who is 11wks
c. Clopidogrel pregnant had central abdominal pain for
d. Dipyridamole 36h. The pain is now colicky. There is no
e. Simvastatin vaginal bleeding. She has vomited once
and has had an episode of loose motion.
Ans. The key is A. Alteplase. It is a wrong key!
She looks ill, temp=37.8C and there is
Correct option is B. Aspirin. [In stroke window
rebound tenderness in the RIF. What is the
period to use alteplase is 4.5 hours. In the given
history we can no way certain the time of stroke most probable dx?
a. Salpingitis
and we can not proceed for alteplase unless
b. PID
window period is confirmed. Hence we can not
give alteplase but Aspirin]. c. Appendicitis
d. Ovarian torsion
1012. A 33yo man presented to the GP e. Uterine fibroid
with hx of headaches and photophobia.
The GP examines him and finds a rash and
Ans. The key is C. Appendicitis. [No vaginal
is now ringing you at the hospital for
bleeding and pain above pelvis makes pregnancy
advice.
complication less likely. Loose motion, low grade
What would you advice the GP? temperature and positive Mc Burney’s sign makes
a. Send pt home
the diagnosis of appendicitis more probable].
b. Start IV benzylpenicillin
c. Conduct LP
d. Start IV ceftriaxone 1015. A 42yo man presents with stroke.
He is not able to walk straight and his
Ans. The key is B. Start IV benzylpenicillin. [Before speech is slurred. What is the initial
hospitalization IV benzylpenicillin. In hospital appropriate inv?
Ceftriaxone can be given but not in calcium a. CT brain
containing fluid instead give Cefotaxime (NICE).
b. PET brain
According to OHCM hospital management is
c. MRI brain
d. Carotid angiography
e. Monitor for 24h What drug is the appropriate choice?
a. Disulfiram
b. Acamprosate
Ans. The key is C. MRI brain. [Ataxia and slurred
c. Vitamin supplement
speech are features of cerebellar lesion which is
d. Naloxone
posterior fossa organ. For posterior fossa lesion
MRI is the investigation of choice! So key is e. Naltrexone
correct. It is C. MRI brain].
Ans. The key is B. Acamprosate. This is a wrong
***1016. A 24yo woman has severe key! Correct key is A. Disulfiram. [Acramposate is
depression 3m after the birth of her first not a deterrent (which keeps away from drinking
by making it unpleasant) but Disulfirum!]
child. She is breastfeeding but is otherwise
unable to look after the baby and is 1019. A 68yo woman presents to the ED
convinced that her family is likely to kill with confusion. Temp=39.3C and
her. She has no interest in anything and productive cough. Sputum is rusty colored
keeps crying. after 2 days. CXR shows right lower lobe
What is the most appropriate tx? consolidation. What is the most likely
a. Fluoxetine organism?
b. Citalopram a. Streptococcus pneumonia
c. CBT b. Staphylococcus aureus
d. ECT c. Coxiella burnetti
e. Haloperidol d. Mycoplasma pneumonia
Ans. The key is D. ECT. [ [In question it is Ans. The key is A. Streptococcus pneumonia.
mentioned she has severe depression and also in [Typical presentation of streptococcal
history she has delusion of persecution which pneumonia].
makes likely diagnosis of psychotic depression!
However in both severe and psychotic depression 1020. A 70yo man with prostatic cancer
the tx is ECT]. has had severe acute back pain waking
him up at night for 6wks. What is the
1017. A 20yo woman with no prv hx of ear
most appropriate inv?
complains, presents with 1d hx of severe a. MRI spine
pain in the right ear which is extremely b. Radionuclide bone scan
tender to examine. What is the single c. DEXA scan
most likely dx? d. Serum ALP concentration
a. Chondromalasia e. Serum calcium concentration
b. Furuncle
c. Myringitis Ans. The key is B. Radionuclide bone scan. [[MRI
d. OE is good for soft tissue but not for bone. If it was
e. OM radiculopathy, spinal cord compression or
prolapsed disc creating pressure on nerve MRI
Ans. The key is D. OE. [Extreme tenderness to would be fine but not for bony metastasis. Here
examine indicate otitis externa]. investigation of choice is radionuclide bone scan].
1018. A couple has just finished their 1021. An asymptomatic 56yo man who
detox regime and wants a drug with a has never consumed alcohol came for a
pharmacological action to serve as a routine checkup. Exam: increased skin
deterrent when they take alcohol. pigmentation, spider angioma,
cardiomegaly, S3 gallop, liver firm with d. Dosulepin
8cm span, no ascites. He is in the risk of e. Diazepam
which condition?
a. Cerebellar degeneration
b. Werniecke’s encephalopathy Ans. The key is B. Citalopram. [Mild depression =
c. Renal failure CBT, Moderate depression = Antidepressant,
d. Hepatoma Severe depression & Psychotic depression = ECT,
e. Hepatic vein thrombosis Amitryptiline and Dosulepine causes urinary
retention (which is comperatively less in
Ans. The key is D. Hepatoma. [Increased skin citalopram) so not suitable in a patient with lower
pigmentation, features of chronic liver disease urinary symptoms. Hence Citalopram is the
(spider angioma), Cardiomyopathy (cardiomegaly, preferred option].
S3 gallop), Enlarged liver these are suggestive of
Haemochromatosis. (OHCM: Slate-grey skin
pigmentation; signs of chronic liver disease; 1024. A 48yo pt after surgical removal of
hepatomegaly; cirrhosis; dilated cardiomyopathy;
mandibular ca presents with perioral
osteoporosis; also endocrinopathies are features
paresthesia and severe pain which is not
of haemochromatosis). If cirrhosis, 22–30% get
hepatocellular cancer].
relieved by oral morphine.
What is the next step in treating this pt?
a. Oral amitryptiline (2nd line)
b. Oral oxycodone
1022. A 39yo male presents with visual
c. PCA
symptoms. Ophthalmoscopy shows
d. IV morphine
papilloedema. Which anatomical site is
e. Fentanyl patch
most likely to be affected?
f. Gabapentine (1st line)
a. Optic nerve
b. Optic disc
c. Optic radiation
Ans. The key is F. Gabapentine.
d. Occulomotor nerve
e. Optic chiasma
2. Grade 2: 1 + AV nipping Ans. The key is B. Serum PSA. [Serum PSA is used
to assess the progress of treatment. (If a man’s
3. Grade 3: 2 + flame shaped
PSA level rises after prostate cancer treatment,
hemorrhage
his doctor will consider a number of factors
4. Grade 4: 3 + optic disc before recommending further treatment.
edema + macular star]. Additional treatment based on a single PSA test is
not recommended. Instead, a rising trend in PSA
1039. A pt whose pain is not relieved by level over time in combination with other
oral codeine. What is the best findings, such as an abnormal result on imaging
management? tests, may lead a man’s doctor to recommend
a. Oral oxycodone further treatment)].
b. Co-codamol
1042. A middle aged woman has some anorexia (BMI 15–17.5, no evidence of system
weakness of hand after an injury. Which failure) routine referral can be to
vertebra will be the lowest to be included the local community mental health team
on cervical XR to dx the injury? (CMHT)/adolescent unit or Eating Disorder Unit
a. C7/T1 (EDU) if available. So the key is A. Eating disorder
b. C8/T1 clinic]. OHCS, 9th edition, page-349.
c. C5/C6
d. C6/C7 1045. A 36yo woman has an injury to the
right external laryngeal nerve during a
Ans. The key is A. C7/T1. thyroid surgery. What symptom would be
expected in this pt?
1043. A 50yo man with a known hx of
a. Stridor
stroke. He can’t remember anything about b. Hoarseness
his life. What is the single most likely c. Aphonia
defect? d. Dysphonia
a. Homonymous hemianopia
e. Aphasia
b. Homonymous upper quadrantanopia
c. Bitemporal hemianopia
d. Binasal hemianopia Ans. The key is D. Dysphonia. [External laryngeal
e. Homonymous lower quadrantanopia nerve (smaller, external branch of the superior
laryngeal nerve) lesion causes mono toned voice
(loss of ability to produce pitched sound) that is
Ans. The key is B. Homonymous upper dysphonia].
quadrantanopia. [Memory processing occur in
temporal lobe. So as the patient lost memory his 1046. A 75yo woman has weakness of the
temporal lobe is involved. We know temporal left side of her face. She has had a painful
lobe lesion causes “homonymous upper ear for 48h. There are pustules in the left
quadrantanopia” which is the key here!] ear canal and on the eardrum. What is the
1044. An 18yo girl has been dx with single most likely dx?
a. Chronic serous OM
anorexia nervosa and has mild depressive
b. Herpes zoster infection
symptoms. She has cut down her food
c. Impacted earwax
intake for the last 18m and exercises 2h
d. Perforation of eardrum
everyday. Her BMI=15.5, BP=90/60mmHg.
e. Presbycusis
What would be the single most
appropriate management? Ans. The key is B. Herpes zoster infection. [A case
a. Refer to eating disorder clinic of Ramsay Hunt syndrome defined as an acute
b. Refer to psychodynamic therapy peripheral facial neuropathy associated with
c. Refer to acute medical team erythematous vesicular rash of the skin of the ear
d. Prescribe antidepressant canal, auricle (also termed herpes zoster oticus),
and/or mucous membrane of the oropharynx
caused by Herpes zoster infection].
Ans. The key is C. Refer to acute medical team.
This is a wrong key! Correct key is A. Eating 1047. An 8wk baby boy is noted to be
disorder unit. [Anorexia nervosa is moderate jaundiced. He is breast-feeding well and
when BMI is 15-17.5 as in given case. In moderate has gained 300g since birth. His stools are
yellow and his urine is pale straw colored.
What is the most likely dx? 8mm in size. Which tx modality will be
a. Galactosemia most effective?
b. Biliary atresia a. Fluids and alpha blockers
c. G6PD deficiency b. ESWL
d. Breast milk jaundice c. CCB
e. Congenital viral infection d. Dormier basket
e. PCNL
Ans. The key is A. Galactosemia. [Biliary atresia
causes obstructive picture where stools are pale Ans. The key is B. ESWL. [Though for 8 mm stone
and urine becomes dark. Hemolytic disorder like we can use medical expulsive therapy but for this
G6PD or spherocytosis causes appearance of patient with agonizing pain “most effective”
jaundice in 1st 24 hours. In breastmilk jaundice, therapy seems to be ESWL].
jaundice develops in 2nd week. Also congenital
viral infection (TORCH) occurs in 1st 24 hours. 1051. A 37yo woman believes that her
Here there is failure to thrive also. So likely cause neighbours have been using her shower
in the presented case is Galactosemia]. while she is away from home. Her 42yo
partner is convinced about this and calls
1048. A lady developed breast abscess
the police.
after delivery. What is the most likely
What term best describes this situ?
organism? a. Capgras syndrome
a. Staph aureus
b. Cotard syndrome
b. Staph albus
c. Delusion of persecution
c. GBS
d. Folie a deux
d. Strep pyogenes
e. Munchausen syndrome
e. Strep faecalis
Ans. The key is D. Folie a deux. [Folie a deux is
Ans. The key is A. Staphylococcus aureus. symptoms of a delusional belief
1049. A 32yo man suffering from MS and hallucinations that are transmitted from one
individual to another as here from wife to
presents with blurring of vision.
husband].
Ophthalmoscopy shows pallor of the optic
disc. Which anatomical site is most likely 1052. A 45yo woman has dull pain in her
to be affected? right ear which has been present for
a. Optic nerve several weeks. There is no discharge.
b. Optic disc Chewing is uncomfortable and her
c. Optic radiation husband has noticed that she grinds her
d. Trigeminal teeth during sleep. The eardrum appears
e. Oculomotor nerve normal.
Ans. The key is A. Optic nerve. [ MS--optic What is the single most likely dx?
neuropathy which affects optic nerve and long a. Dental caries
standing optic neuropathy can be seen as pallor b. Mumps
of optic disc in opthalmoscope]. c. OM
d. Temporomandibular joint pain
1050. A 23yo man presents with severe e. Trigeminal neuralgia
pain in the right flank radiating to his
groin. He is rolling about on the floor. An
IVU confirms a stone in the ureter which is
Ans. D. Temporomandibular joint pain. lesion also. These features only can be explained
[Temporomandibular joint pain may cause pain in by cranial nerve involvement in brainstem lesion].
ear and teeth grinding is a recognized cause of
this symptom]. 1055. A 30yo woman has experienced
restlessness, muscle tension and sleep
disturbance on most days over the last
1053. A 42yo lady had corrective surgery 6m. She worries excessively about a
for cyanotic congenital heart disease at number of everyday events and activities
the age of 3y, after a palliative operation and is unable to control these feelings
during infancy. There is a parasternal which are impairing her ability to hold
impulse and an early diastolic murmur. down her job. What is the most likely dx?
a. Panic disorder
What is the most probable dx?
a. Aortic regurgitation b. GAD
b. Ischemic mitral regurgitation c. Pheochromocytoma
c. Aortic stenosis d. Acute stress disorder
d. Pulmonary stenosis e. Social phobia
e. Pulmonary regurgitation
Ans. The key is B. GAD. [In GAD patient is worried
about different number of events every day.
Almost everything triggers the anxiety].
Ans. The key is E. Pulmonary regurgitation. [In the
present day, some patients with tetralogy of 1056. Which of the following is not a
Fallot have survived for longer than 15-20 years degenerative corneal disease?
after their first operation. The major problem
a. Band keratopathy
encountered by these individuals is the
b. Marginal dystropathy
development of pulmonary valvular regurgitation.
It appears that a number of these individuals c. Fatty/lipid degeneration
require pulmonary valve replacement (ref: d. Mooren’s ulcer
Medscape)]. e. Keratoconus
*1063. A pt was complaining of pain 1066. What is the most likely dx based on
within 6h after his appendectomy for
this ECG
gangrenous appendix. What med is the
best option for his pain relief?
a. IV morphine
b. Diclofenac per rectal
c. PCA
d. Tramadol
1084. A 24yo male who is sexually active Ans. The key is C. Low Na+, Low K+. [Frusemide
with other males with hx of discharge per causes hyponatremia, hypochloremic alkalosis,
hypokalemia, hypomagnesemia and
urethra. Dx of chlamydia has been made.
hypocalcemia].
What is the possible complication if left
untreated?
a. Orchitis
b. Balanitis
1087. A 70yo hypertensive white british
c. Epididymo-orchitis
man on thiazide diuretics needs a 2nd
d. Acute abdomen drug to control his BP. Which one of the
following is the best choice for him?
a. Amlodipine (CCB)
b. Enapril (ACEi)
Ans. The key is C. Epididymo-orchitis.
c. Propranolol (BB)
d. Increase dose of diuretic
e. Prazocin (Alpha blocker)
Ans. The key is B. Enalapril. [In patients >55 yrs Ans. The key is C. Mobitz type 1 block. [Gradual
CCB. If CCB not suitable (oedema, heart failure) prolongation of PR interval followed by a drop
thiazide is preferred. In patients <55yrs ACEI or beat].
ARB. Now if patient is on CCB or Thiazide 2nd
drug should be added it should be one of ACEI or
ARB and not one from CCB or Thiazide]. 1090. A 29yo woman presents with lid lag,
lid retraction and diplopia.
What is the most appropriate next step?
1088. A 74yo lady who has had a stroke in a. TFT
the past has an indwelling catheter for b. Tensilon test
10m. She presents with bluish-purple c. Fundoscopy
discoloration of the catheter bag. d. Autoantibodies
What is the most likely explanation for e. EMG
this?
a. Normal change
b. Catheter degradation Ans. The key is A. TFT. [Features are suggestive of
c. Acidic urine Grave’s disease hence TFT].
d. Alkaline urine
e. Bacterial colonization of the urinary tract
1091. A 41yo man presents with
longstanding foul smelling ear discharge
and progressive hearing loss. Otoscopy
Ans. The key is E. Bacterial colonization of the
urinary tract. [Purple Urine Bag Syndrome].
showed perforation of the pars flacida
and a mass in the upper part of the middle
ear. What is the most likely dx?
a. ASOM
1089. A 62yo man has slow palpitations
b. CSOM
and the following ECG. What is the most
c. Acquired cholesteatoma
likely dx?
d. Congenital cholesteatoma
e. Barotrauma
Ans. The key is D. Barium enema. [Features are 1097. A 55yo male has been admitted for
suggestive of IBD, so barium enema is the most elective herniorraphy.
relevant investigation among the given options]. Which among the following can be the
reason to delay his surgery?
a. Controlled asthma
1095. A 26yo young man presents with hx b. Controlled A-fib
of passing loose stools for the past 2m. He c. DVT 2yrs ago
says his stools contain blood and mucus d. DBP 90mmHg
and are a/w abdominal pain. He e. MI 2m ago
undergoes a colonscopy after which he
was started on tx. What is the most
appropriate tx for his condition? Ans. The key is E. MI 2m ago. [Following MI at
a. Mesalazine least 6 months should be elapsed for elective
operation. Prior this time surgery is associated progesterone=normal, LH=33.2. What is
with increased mortality]. the most probable dx?
a. PCOS
b. Pregnancy
1098. A 21yo female in her first pregnancy c. Cushing’s disease
at 38wks was brought to the ED with d. CAH
generalized tonic clonic seizure. IV MgSO4 e. POF
was given but fits was not controlled. She
is having fits again. What is the single
Ans. The key is A. PCOS. [Clinical features
most imp immediate management of this (obesity, hirsutism, amenorrhea etc.)are
pt? consistent with PCOS. We know in PCOS LH:FSH
a. IV MgSO4 ratio is 2:1 or 3:1. Here LH is 33.2 and FSH is 10.9
b. IV diazepam which also supports diagnosis of PCOS].
c. Immediate C-section
d. IV phenytoin
e. MgSO4 bolus 1100. A 17yo girl with a lump in her breast
f. IV lorezepam was seen in the clinic. Exam: the lump was
free and mobile and not attached to the
skin. Her mother wants further tests done.
Ans. The key is A. IV MgSO4. It is a wrong key! What should be the next step?
Correct key is E. MgSO4 bolus. [Treat a first a. CT
seizure with 4g magnesium sulfate in 100mL 0.9% b. US breast
saline IVI c. Punch biopsy
d. Reassure and send home
over 5min + maintenance IVI of 1g/h for 24h.
e. Stereotactic biopsy
Beware respiration.
Ans. The key is B. US breast. [Described lump is
If recurrent seizure give 2g IVI magnesium
sulfate over 5 min. fibroadenoma. Next investigation is either US
Check tendon reflexes and respiratory breast or mammography].
rate every 15min.
Stop magnesium sulfate IVI if respiratory
rate <14/min or tendon reflex loss, or 1101. A lady comes with a missing IUCD
urine output <20mL/h. Have IV calcium thread. Her LMP was 2wks ago. What is
gluconate ready in case of MgSO4
the single most appropriate next step in
toxicity: 1g (10mL) over 10 min if
respiratory depression. management?
Use diazepam once if fits continue (eg 5– a. Abdominal US
10mg slowly IV). If seizures continue, b. Prescribe contraceptives
ventilate and consider other causes c. CT
(consider CT scan). OHCS, 9th edition, d. Serum BHCG
page-49]. e. Vaginal exam
Ans. The key is B. Acute PID. [Fever, lower *1105. A 37yo lady stopped taking COCP
abdominal pain and vaginal discharge are 18m ago and she had amenorrhea for
features of PID].
12m duration. Labs: FSH=8, LH=7,
prolactin=400, estradiol=500. What is the
cause?
1103. A 40yo female was on COCP which a. Hypothalamic amenorrhea
she stopped 6m ago. But she has not had b. PCOS
her periods since then. Labs: FSH=22, c. Prolactinoma
LH=24, prolactin=700, estradiol=80. d. Post pill amenorrhea
What is the most appropriate dx? e. POF
a. Hypothalamic amenorrhea
b. Post pill amenorrhea
c. Prolactinoma
Ans. The key is D. Post pill amenorrhea. [Post pill
d. Pregnancy amenorrhea= failure to resume menses within 6
e. Premature ovarian failure months but here initial 6 months there was
menses!! again in post pill amenrrhea LH and FSH
low also Oestrogen should be low but here
Ans. The key is E. Premature ovarian failure. [ FSH oestrogen is high!! In post pill amenorrhea
and LH are raised in ovarian failure; an FSH level prolactin raised but in hypothalamic amenorrhea
≥20 IU/l in a woman aged under 40 with prolactin normal and oestrogen low. Actually the
secondary amenorrhoea indicates ovarian given picture does not fit to any condition given
failure]. in options and probably a bad recall or erroneous
question!!!].
*1106. A lady with a firm smooth breast d. Adenomyosis
lump in outer quadrant had a FNAC done. e. Incomplete abortion
Results showed borderline benign
changes. She also has a fam hx of breast Ans. The key is C. Fibroids. [Regular heavy
menstruation in a uterus of 14 wk size is highly
cancer.
suggestive of fibroid].
What is the your next?
a. Mammography 1109. A 29yo at 38wks GA presents with a
b. US 2h hx of constant abdominal pain. She
c. Core biopsy then passes 100ml of blood per vagina.
d. Genetic testing and counselling What is the next appropriate inv?
e. Punch biopsy a. USS
b. CTG
c. Clotting screen
Ans. The key is D. Genetic testing and counselling. d. Hgb
Probably wrong key! Correct one should be C. e. Kleihauer Betke test
Core biopsy. [ Genetic testing is done for
asymptomatic patients in their 30s and if positive Ans. The key is A. USS. [Abdominal pain and
SERMs are given which cause significant bleeding makes the likely diagnosis of abruption
reduction in cancer . placenta for which USS is the next appropriate
Since patient has firm breast lump in upper outer investigation].
quadrant ( most common location for cancer) and
FNAC has shown borderline benign changes plus
1110. A 26yo woman had amenorrhea for
patient is high risk for cancer , so I must go for 10wks and is pregnant. She experiences
core biopsy to confirm whether it is cancer or hyperemesis. Now she presents with
not]. vaginal bleed. Exam: uterus=16wks,
closed os. What is the most probable dx?
1107. A pt presents with mild dyskaryosis. a. Thyrotoxicosis
1y ago smear was normal. b. Hyperemesis gravidarum
What is the most appropriate next step? c. Twins
a. Cauterization d. Wrong dates
b. Repeat smear e. Molar pregnancy
c. Swab and culture
d. Cone biopsy Ans. The key is E. Molar pregnancy. [In
e. Colposcopy hytaditidiform mole uterus becomes more in size
than actual gestational age and due to production
Ans. The key is E. Colposcopy. [In mild dyscariosis of large amount of gonadotrophin by moles
colposcopy should be done with HPV test]. patient suffers from severe vomiting i.e.
hyperemesis].
1108. An African lady presents with heavy but
regular periods. Her uterine size correlates to 1111. A pregnant woman of G2, GA 11wks
14wks presents with heavy vomiting, headache
pregnancy. What is the most appropriate dx? and reduced urine output. Urine analysis
shows ketonuria. Choose the next best
a. Blood dyscrasia
step?
b. Hematoma a. US
b. Oral fluid replacement
c. Fibroids
c. Serum BHCG e. Karyotyping
d. Parental anti-emetics
e. IV fluids Ans. The key is E. Karyotyping. [The likely
diagnosis is Klinefelter’s syndrome for which
Karyotyping should be done to make the
diagnosis established].
Ans. The key is E. IV fluids. [Hyperemesis
gravidarum with oliguria and ketonuria. IV fluid 1115. A woman who is on regular COCP
should be started to prevent renal failure]. presented to you for advice on what to do
as she has to now start to take a course of
1112. A pt had inflammatory changes on
7d antibiotics. What would you advice?
cervical smear. There is no vaginal
a. Continue regular COC
discharge, no pelvic pain and no fever. b. Continue COCP and backup contraception
What is the next step? using condoms for 2d
a. Repeat smear in 6m
c. Continue COCP and backup contraception
b. Take swab
using condoms for 7d
c. Treat with antibiotics
d. Continue COCP and backup contraception
d. Colposcopy
using condoms for 2wks
e. Cone biopsy
Ans. The key is D. Continue COCP and backup
contraception using condoms for 2wks. This is
Ans. The key is B. Take swab. wrong key! Correct key should be A. Continue
regular COCP. [Before it was thought that
1113. A 37yo infertile lady with 5cm antibiotics alter the gut flora and
subserosal and 3cm submucosal fibroid is ethinyloestradiol is not conjugated. There is more
trying to get pregnant. Which is the most ethinyloestradiol passed in the stool. So
suitable option? pregnancy and breakthrough bleeding can occur.
a. Clomifen therapy But later it was established that except for strong
b. IVF enzyme inducer like Rifampicin and Rifabutin
c. Myomectomy other antibiotics practically does not reduce
d. Hysterectomy potency of COCP and hence except only
e. IU insemination rifampicin or rifabutin for other antibiotics no
additional precautions are needed].
Ans. The key is C. Myomectomy. [Fibroids are
preventing from her being pregnant. So 1116. A lady presents with hot flashes and
myomectomy is the most suitable option to let other symptoms of menopause.
her get pregnant]. What is the tx option?
a. Raloxifen
b. HRT
1114. A young tall man and his wife are c. Bisphosphonate
trying for babies and present at the d. COCP
infertility clinic. On inv the man has e. Topical estrogen
primary infertilitiy and azoopermia.
What other inv should be done? Ans. The key is B. HRT
a. Testosterone 1117. A 28yo woman at 34wks GA for her
b. LSH
first pregnancy attends antenatal clinic.
c. FSH
d. Estradiol
Her blood results:Hgb=10.6, MCV=95, d. Danazol
MCHC=350. What do you do for her? e. Mirena
a. Folate f. IUCD
b. Dextran
Ans. The key is E. Mirena.
c. Ferrous sulphate
d. None 1120. A 32yo woman comes with
e. IV FeSO4 intermenstrual bleeding. Her last cervical
f. Explain this physiologic hemodynamic smear was 1y ago and was negative.
anemia What test would you recommend for her
g. Blood transfusion initially?
a. Colposcopy
Ans. The key is F. Explain this physiologic
b. Cervical smear
hemodynamic anemia. [According to NICE, cut
offs for iron supplements: c. Endocervical swab
at booking (8-10 weeks)- if less than 11 d. Transvaginal US
at 28 weeks and further- if less than 10.5 e. Pelvic CT
if less than these values=> give iron].
Ans. The key is B. Cervical smear. This is a wrong
1118. A 34yo woman who never had fits key! Correct key is C. Endocervical swab. [Smear
or high BP developed fits 6h after delivery can only be done if it is due or overdue, not in any
of a term healthy child. What is the most time or not as needed basis].
likely dx?
a. Eclampsia
b. Preeclampsia 1121. A 20yo woman has had abdominal
c. Epilepsy pain in the LIF for 6wks duration. Over the
d. Pulmonary embolism past 48h, she has severe abdominal pain
e. Pregnancy induced HTN and has a fever of 39.1C. Pelvic US shows
a complex cystic 7cm mass in the LIF.
Ans. The key is A. Eclampsia. What is the most likely dx?
[In pregnant women, the convulsive stage of pree a. Endometriosis
clampsia-eclampsia b. Dermoid cyst
syndrome; the convulsions are not c. Ovarian ca
attributable to other cerebral conditions such as e
d. Tubo-ovarian abscess
pilepsy. It can occur from 20 weeks of gestation
e. Ectopic pregnancy
to post partum period (usually 10 days after
delivery)]. Ans. The key is D. Tubo-ovarian abscess.
1119. A 30yo lady who already has one 1122. A woman is 16wk pregnant and she
child through a prv C-section demands a is worried about abnormal chromosomal
reversible contraception. She presently anomaly in her child. What is the
experiences heavy and painful periods. definitive inv at this stage?
What is the most appropriate a. Amniocentesis
contraceptive you will recommend for b. CVS (Chorionic Villous Sampling)
her? c. Parents karyotyping
a. COCP d. Coomb’s test
b. POP e. Pre-implantation genetic dx
c. Implanon
Ans. The key is A. Amniocentesis. [Amniocentesis ***1125. A 36wk pregnant woman
is done between 14 to 16 weeks of gestation. CVS presents with sudden onset of uterine pain
is done prior to 15 weeks. So the option is A. and bleeding, uterus is tender, no prv
Amniocentesis].
LSCS. What is the most appropriate
1123. A 28yo lady with a fam hx of CF cause?
comes for genetic counselling and wants a. Preeclampsia
the earliest possible dx test for CF for the b. DIC
baby she is planning. She is not in favor of c. Placental abruption
d. Placental previa
termination. What would you recommend
e. Ectopic pregnancy
for her?
a. CVS (Chorionic Villous Sampling) f. Missed abortion
b. Amniocentesis g. Ectropion
c. Pre-implantation genetic dx
d. Chromosomal karyotyping
e. Maternal serum test Ans. The key is C. Placental abruption. [Painful
f. Reassure bleeding at 36th week is placental abruption
(either revealed or mixed type)].
1150. A 25yo woman presents with a Ans. The key is E. Mikulicz’s disease. [Mikulicz’s
painful shallow ulcer on the vulva. What disease (MD) is a well-known disorder
inv has to be done? characterized by enlarged lacrimal and parotid
a. HSV antibodies glands caused by infiltration with lymphocytes.
b. Syphilis serology When no specific cause is found it is called
c. Swab for hemophilus ducreyi Mikulicz’s disease; and if secondary to disease like
d. Urine culture sarcoidosis it is termed as Mikulicz’s syndrome].
e. Blood culture
1153. A woman has widespread
Ans. The key is C. Swab for hemophilus ducreyi. metastasis from a carcinoma. She
[Herpes simplex virus causes multiple painful presented with severe back pain. Where
vesicles and syphilis ulcer is painless. As here do you expect the cancer to be?
single painful ulcer probable dx is Chancroid a. Lungs
caused by Hemophilus ducreyi]. b. Cervix
c. Ovary
1151. A child was admitted with fever, d. Uterus
generalized skin lesion, some of them are e. Breast
Ans. The key is E. Breast. [In female breast and 1157. A 15m baby girl presented to the ED
lung cancer and in male prostate and lung cancer with difficulty in breathing. Exam: she has
are most common to metastasize to bone]. intercostals recessions and a wheeze.
1154. A 10yo child has got progressive Temp=normal. What is the most likely dx?
a. URTI
bilateral hearing loss. He has started to
b. Pneumonia
increase the TV volume. All other
c. Bronchiolitis
examination is normal. What is the most
d. RDS
likely dx?
e. Alpha 1 antitrypsin deficiency
a. Wax
b. Foreign body Ans. The key is C. Bronchiolitis. [Asthma and
c. Bilateral OM with effusion bronchiolitis has similar sign symptoms. The
d. SNHL difference is in pathology. In asthma there is
e. Meningitis due to meningococcus bronchoconstriction but in bronchiolitis there is
oedema of the airway wall. In infants and
Ans. The key is C. Bilateral OM with effusion. younger children bronchiolitis is commoner than
1155. A child had a patchy rash following asthma and it is seen that many of the children
with bronhiolitis usually followed by asthma].
tx for sore throat & cervical LN
enlargement. Which is the most 1158. An 8yo boy develops a seizure
appropriate antibiotic? affecting his right arm, seizure lasts for
a. Ampicillin several mins. He doesn’t remember
b. Erythromycin anything what happened. On his CT:
c. Cefuroxime lesion in left hemisphere. What is the most
d. Metronidazole probable dx?
e. Tetracycline a. Epilepsy
b. Space occupying lesion
Ans. The key is A. Ampicillin. [Infectious
mononeucleosis can present with sorethroat and c. Dementia
lymphadenothy like tonsillitis and if treated with d. Huntington’s chorea
ampicillin leads to eruption of patchy rashes]. e. Intracranial HTN
1156. A child with a hx of asthma is Ans. The key is B. Space occupying lesion. [This is
brought to ED with a cut on knee and complex partial seizure due to space occupying
sprained on her left wrist.cWhich is the lesion].
best analgesic for her? 1159. A 28yo female presented with
a. Paracetamol
complains of difficulties in swallowing
b. NSAIDs
liquids only. She also suffers from
c. Cocodemol
recurrent chest infection in the past few
d. Ibuprofen
months. What is the most probable dx?
Ans. The key is A. Paracetamol. [NSAIDs including a. Foreign body
ibuprofen can precipitate asthma and Cocodamol b. Plummer vinson syndrome
is also advised to avoid in asthma (due to its c. Achalasia cardia
codeine content). So paracetamol is the best d. Peptic stricture
option here]. e. Esophageal carcinoma
Ans. The key is C. Achalasia cardia. [Dysphagia to e. Warfarin
both solid and liquid or mostly to liquid are
common feature of achalasia. In achalasia there
may occur aspiration due to regurgitation during Ans. The key is B. DIC.
lying down to sleep which may result recurrent
chest infection and cough]. 1163. A study was done amongst 2 hosp
for the equal number of cancer pts. It was
1160. Mother having 2 children with CF.
noted that hosp A had the higher rate of
What is the risk of getting another baby?
mortality than hosp B for treated cancer
a. 1:2
b. 1:8
pts.
c. 1:4
What is the study done here classified as?
a. Retrospective
d. 1:16
b. Observational
e. 1:1
c. Cohort
d. Case study
Ans. The key is C. 1:4. [Cystic fibrosis is an Ans. The key is C. Cohort study. [A cohort is a
autosomal recessive disease. If both parents are group of people who share a common
carrier there is 1:4 chance of risk of occurrence of characteristic or experience within a defined
the disease for each child]. period (e.g., are born, are exposed to a drug or
vaccine or pollutant, or undergo a certain medical
1161. A 14yo boy has been dx with procedure].
nephrotic syndrome. 5d later he presents
with flank pain, hematuria and fluctuating 1164. A 17yo girl comes to see her GP
urea levels. A dx of renal vein thrombosis after having unprotected sex 2d ago. She
is made. What is the most likely cause for asks if her GP can explain to her how this
renal vein thrombosis? prescribed procedure would work by
a. Protein C deficiency helping her not to get pregnant.
b. Vasculitis a. It helps to prevent implantation
c. Loss of antithrombin III b. It helps in preventing or delaying
d. High estrogen levels ovulation
e. Stasis c. It causes an early miscarriage
d. It releases progesterone and stops
ovulation
Ans. The key is C. Loss of antithrombin III. e. It causes local enzymatic reaction
1166. A healthy 8yo boy had antibiotic tx 1169. A young child, 3yo, has presented
for meningitis. Initially he wasn’t with vomiting for 3d. Exam: mild-mod
resuscitated. What will be the outcome if dehydration. What is his ABG profile likely
he receives full tx? to show?
a. He will recover fully to his prv health a. pH low, PCO2 low
b. He will have hearing impairment b. pH low, PCO2 high
c. He will have brain abscess c. pH high, PCO2 low
d. He will have encephalitis d. pH high, PCO2 high
1214. A 46yo man who is a heavy drinker 1217. A prv healthy 23yo presented a
is brought to the ED in a drowsy state. He week hx of bloody diarrhea and
is responding vaguely to questions. Exam: abdominal pain with cramps and fever.
nystagmus and hyperreflexia. MCV=103fl. Exam: tenderness in lower abdomen.
What is the most likely cause for his What is the most appropriate dx?
cognitive impairment? a. Celiac disease
a. B1 deficiency b. Colorectal polyps
b. B12 deficiency c. UC
c. Folate deficiency d. Laxative abuse
d. B6 deficiency e. Gastroenteritis
e. Alcohol withdrawal
1219. A 26yo man strongly believes that 1222. A 45yo man has had impaired vision
every elderly man he meets is his father. and pain on eye movement in his left eye
Although they look different, he is sure it over the last 5d. He also notes loss of color
is father wearing different disguises. What vision in the same eye. In the left eye, the
kind of delusions is this man suffering visual acuity is up to counting fingers.
from? When the pupil is stimulated with light, it
a. Delusion of persecution dilates. His fundus is normal. What is the
b. Erotomania single most appropriate clinical dx?
c. Delusion of grandeur a. Acute dacryocystitis
d. Delusion of doubles b. Acute iritis
e. Delusion of reference c. Papillitis
d. Retrobulbar neuritis
Ans. The key is D. Delusion of doubles. [The
e. Scleritis
Fregoli delusion, or the delusion of doubles, is a
rare disorder in which a person holds a delusional Ans. The key is D. Retrobulbar neuritis.
belief that different people are in fact a single [Presentation can be described by either optic
person]. neuritis or retrobulbar neuritis. In optic neuritis
there is disc pallor. As in this case disc is normal it
1220. A 26yo passed a 4mm stone in his
is more likely to be a case of retrobulbar neuritis.
urine. On US a 3mm stone is found in the
renal pelvis. What is the single most 1223. A 56yo pt has been dx with MS. She
appropriate management? presents with a positive Romberg’s test.
a. ESWL She also has weakness and loss of
b. None sensations in all her 4 limbs. Which site is
c. Open Surgery most likely to be affected?
d. Conservative a. Cerebral cortex
b. Cerebellum
Ans. The key is D. Conservative. [Increased fluid
c. Cervical spinal cord
intake is advised].
d. Thoracic spinal cord
1221. A 35yo man has had acute pain and e. Brain stem
swelling below the mandible on the left
side for 2h. The swelling occurred after
eating a large meal. What is the single Ans. The key is C. Cervical spinal cord.
most likely dx? [Quadriplegia is the feature of cervical cord
a. Laryngocele lesion].
b. Ranula
c. Neck abscess 1224. A 58yo man suddenly becomes
d. Parotid calculus
shocked several days after suffering an
e. Submandibular calculus
acute ant MI. His CXR shows a large
Ans. The key is E. Submandibular calculus. [Pain globular-shaped heart and clear lung
and swelling below mandible is due to stone in fields. What is the single most likely
submandibular salivary gland duct. Pain and explanation for the abnormal inv?
a. Acute pericarditis Ans. The key is C. Closed reduction of fracture.
b. Cardiac tamponade This is a wrong key! Correct option seems to be B.
c. Atrial thrombus Below elbow split plaster of paris. [Older people
d. Left ventricular aneurysm are less likely to go under closed reduction even if
e. Dressler syndrome the dorsal angulation is not anatomic (like 20
degrees!). Less than 10 degree dorsal angulation
does not require even closed reduction but only
immobilization. [Medscape]. As plaster cast is
Ans. The key is B. Cardiac tamponade. [Shock in a
often advocated B. Below elbow split plaster of
post MI patient with globular heart on X-ray and
paris seems to be the option of choice].
clear lung field indicate Cardiac tamponade].
1227. A 16yo girl who is normally fit and
1225. A 56yo alcoholic man who has well attends her GP complaining of heavy
increased the amount of alcohol he is and painful periods. She is requesting tx
using wants to attend his daughter’s for these complaints. She denies being
wedding that is in 2wks. He is now coming sexually active. Select the most
to you for help. How would you help him? appropriate management for her
a. Acamprosate menorrhagia?
b. Refer to clinical psychologist a. Antifibrinolytics (tranexamic acid)
b. COCP
c. Refer to GP
d. Despiramine c. Endometrial ablation
d. IUS progestrogens (mirena)
e. Refer to community mental health
support group. e. NSAIDS (mefenamic acid)
Ans. A. Secure airways. [Breathing difficulty with 1238. A 36yo man has been dx with DI.
cyanosis may indicate saliva, blood, foreign body What electrolyte picture is expected to be
(as denture), fall back of tongue etc. for which 1st seen?
action is to secure airway]. a. High serum Na, low serum osmolarity,
high urine osmolarity
b. Low serum Na, low serum osmolarity,
1236. A 30yo man is becoming concerned high urine osmolarity
about the safety of his family. He has been c. Low serum Na, high serum osmolarity,
checking the locks of the door every hour high urine osmolarity
during the night. He becomes very anxious d. High serum Na, high serum osmolarity,
if his wife tries to stop him. What is the low urine osmolarity
e. Normal Na, normal serum osmolarity,
most likely dx?
a. Paranoid delusion normal urine osmolarity
b. PTSD
c. Social phobia
d. OCD Ans. The key is D. High serum Na, high serum
e. GAD osmolarity, low urine osmolarity.
and LOC. She is brought to the hosp where
she is found to be conscious and
1239. The artery that supplies the ant
completely alert. Exam: normal pulse and
right ventricular wall. What is the single
BP with no abnormal neurological sign.
most appropriate option?
a. Acute marginal branch What is the next step in her
b. Left ant descending artery management?
a. Admission for observation
c. Coronary sinus
b. CT brain
d. Circumflex artery
c. MRI head
e. Right coronary artery
d. Reassurance and discharge home
e. XR skull
1242. A 32yo woman suffers an episode of 1245. A 16yo boy came home from
severe occipital headache with vomiting boarding school with a cough. His CXR
showed bilateral consolidations. What is radiculopathy (pressure on spinal nerve root) can
the most likely organism which would cause this shooting pain which can be
have caused his symptoms? demonstrated by MRI spine].
a. Legionella pneumophilia
b. Mycoplasma pneumonia 1248. A young man returns to his hostel
c. Mycobacterium TB and gets headache and lethargy. Now
d. Pneumocystis jiroveci presents with fever. There are crepitations
e. Pseudomonas aeruginosa on the auscultation of lung. What is the
most likely organism which would have
caused his symptoms?
Ans. The key is B. Mycoplasma pneumonia. a. Legionella pneumonia
[Mycoplasma is common in military barrack, b. Mycoplasma
prison or boarding dwellers]. c. Staphylococcus
d. Streptococcus
1246. After an MI, a man presents with
Ans. The key is A. Legionella pneumonia.
pansystolic murmur which is radiating to [Legionella is common in hostel, hotel, hospital,
the axilla. What is the dx? nursing home where it spread through their
a. Tricuspid regurgitation water system, aircondition etc.].
b. Mitral regurgitation
c. Aortic stenosis 1249. A pt is about to undergo surgery.
d. Mitral stenosis Her Hgb=8.9g/dl and MCV=70. What is
the best option for her?
a. Inv and postpone the surgery
Ans. The key is B. Mitral regurgitation. [MI can b. Transfuse and proceed with surgery
lead to papillary muscle rupture causing mitral c. Transfuse and defer surgery
regurgitation]. d. Continue with surgery
1247. A 34yo laborer developed severe Ans. Key is A. Investigation and postpone the
pain in his lower back after lifting a sack surgery. [For elective operation only proceed if
of sand. He also complains of shooting Hb% > 10 g/dl. If Hb% < 10 g/dl then defer the
pain down his leg. The GP has prescribed operation and investigate first. If Hb% < 8 g/dl
him complete bed rest, with painkillers also must be transfused. Samson note].
and also scheduled an MRI for him. What
is the most likely dx? 1250. A 24yo male presents with
a. Peripheral vascular disease discomfort in the groin area and scrotal
b. Intervertebral disc prolapse swelling. Exam: scrotal skin is normal.
c. Hairline fx of the spine What would be the next best step?
d. Sprain of the back muscles a. Urgent US
e. Muscle injury b. Urgent surgery
c. OPD referral
d. Antibiotics
Ans. The key is B. Intervertebral disc prolapsed.
[Intervertebral disc prolapsed can cause severe
back pain following heavy lifting and
Ans. The key is C. OPD referral. [A case of inguinal to go home the following day. Which of
hernia. In both torsion or epididymo-orchitis the following does she require?
scrotal skin will be inflammed (erythematous and a. OPD referral to relationship counselor
oedematous). Should be referred to Surgery b. OPD referral to psychiatrist
OPD]. c. Inpatient referral to psychiatrist
d. Inpatient referral to psychologist
1251. A 22yo girl unhappy about her
weight with BMI=22. She likes to have her
dinner in an expensive restaurant. She Ans. The key is C. Inpatient referral to
does excessive shopping. K+=3.3. What is psychiatrist. [Suicidal ideation is abnormal
the dx? psychology and if discharged there is chance of
a. Anorexia nervosa repeated suicidal attempts. So inpatient referral
b. Bipolar to psychiatrist is needed for this patient].
c. OCD
d. Bulimia
1254. A 74yo man presents with sudden
onset of with right sided weakness and
Ans. The key is D. Bulimia. [BMI 22, even though slurred speech. He also has loss of
unhappy, hypokalemia, like to have dinner in an sensation over the right side of the body
expensive restaurant (probable binge eating)
and visual field defects. CT shows ischemic
suggest the diagnosis of bulimia].
stroke. What is the most appropriate
management?
1252. A 59yo pt has been dx with HTN. His a. Alteplase
BP has been >160/90mmHg on 3 separate b. Streptokinase
occasions. His biochemical profile is as c. Nimodipine
follows: Na+=145mmol/l, K+=6.2mmol/l, d. Aspirin
creatinine=112umol/l, urea=5.7mmol/l. e. Labetolol
What is the most appropriate anti-HTN
drug for him?
a. Amlodipine Ans. The key is A. Alteplase. [Patient has acute
b. Bendroflumethiazide onset of symptoms and time of onset is also
c. Ramipril known. So considering window period of 4.5
d. Lorsartan hours from onset of symptoms can be given
e. Propranolol alteplase].
1257. A pt, 50yo smoker and heavy 1260. A pregnant woman had hit her
drinker, presents with complaints of chest 3wks ago. Now she is 24wks
racing heart. A 24h EKG comes out pregnant and presents with left upper
normal. What is your next step in quadrant mass with dimpling. What is the
management? most probable dx?
a. Echo
a. Breast ca
b. Reassure
b. Carcinoma
c. Stress test
c. Fibroadenoma
d. Fibroadenosis
e. Fatty necrosis of breast
Ans. The key is B. Reassure. [Racing heart or
palpitation is a common phenomenon in
alcoholics which is not serious or harmful. So
Ans. The key is E. Fatty necrosis of the breast.
reassure the patient].
1294. An 84yo woman with drusen and 1297. A 25yo primigravida of 8wk GA
yellow spots in the center of retina. What presents with severe lower abdominal
is the single most likely dx? pain, vaginal bleeding and passage of
a. Macular degeneration clots. The internal os is open. What is the
b. HTN retinopathy most likely dx?
c. MS a. Appendicitis
d. DM background b. Placental abruption
e. Proliferative DM retinopathy c. Ectopic pregnancy
d. Abortion
1320. A 22yo man presents with a red, 1323. A house-bound 78yo man with
hot, swollen, metatarsal phalangeal joint, severe COPD has had a gradual
sarcoilitis and onycholysis. What is the deterioration over recent months and is
single most likely cause of his condition? now breathless at rest. He is on maximal
a. Gout inhaled medical therapy. Result: pH=7.36,
b. RA PaCO2=5.9kPa, PaO2=6.9kPa. What is the
c. Reiter’s syndrome single most appropriate additional tx?
d. Psoriatic arthropathy a. Aminophylline PO
b. ACEi PO
c. Antibiotic PO
Ans. The key is D. Psoriatic arthropathy. [Psoriatic d. Oxygen
arthritis initially can affects the toes very similar e. Steroid PO
c. Allergy
d. Filariasis
Ans. The key is D. Oxygen. [In patients with
chronic hypoxaemia, LTOT should usually be
prescribed after appropriate assessment, when
the PaO2 is consistently at or below 7.3 kPa (55 Ans. The key is A. Lymphedema. [As during
mm Hg) when breathing air during a period of mastectomy axillary clearance is done, lymphatics
clinical stability. Clinical stability is defined as the on that upper limb is compromised. So there
absence of exacerbation of chronic lung disease ocurs lymphedema].
for the previous five weeks. The level of
1326. A homeless person is found
PaCO2(which may be normal or elevated) does
not influence the need for LTOT prescription].
wandering on the street. He had ataxic
gait, nystagmus and opthalmoplegia. He
looked unkempt and his clothes had a
1324. A 79yo man has a swelling of the sweaty odour. He had a dry mucous
right groin which was clinically dx to be membrane with a BP=118/70mmHg and
indirect inguinal hernia. What is the single PR=90bpm. Blood sugar level=8. Alcohol
feature of the hearnia sac that would breath test= -ve. What would the most
confirm the dx? imp initial inv?
a. Comes through femoral ring a. IV insulin
b. Doesn’t pass through the deep inguinal b. Vit B complex
ring c. Bolus IV 0.9%NS
c. Lies below and lateral to the pubic d. IV dextrose
tubercle e. Antibiotics
d. Only passes through the superficial
inguinal ring
e. Passes through the deep inguinal ring Ans. The key is B. Vit B complex. [This is
wernicke’s encephalopathy due to alcohol
withdrawal, so Vit B complex should be checked.
This occurs mainly due to thiamine deficiency].
Ans. The key is E. Passes through the deep
inguinal ring. [Ingunal hernia lies mostly above
and medial (occasionally lateral particularly when
small) to pubic tubercle. It first enters the inguinal 1327. A 34yo man has supra-orbital pain
canal through deep inguinal ring and then enters and tenderness and developed tenderness
the scrotum through the superficial inguinal ring]. over the maxilla. He also has mild fever.
What is the single likely cause for these
symptoms?
1325. A 56yo woman with hx of breast a. Acute sinusitis
cancer 10y ago has undergone radical b. GCA
mastectomy and axillary LN removal, now c. Trigeminal neuralgia
complains of swollen upper limb 3wks d. Maxillary carcinoma
after an insect bite. The bite site is better
but gross edema is still present. What is
the cause? Ans. The key is A. Acute sinusitis. [In frontal
a. Lymphedema sinusitis there is supraorbital pain in frontal
b. Breast Ca sinusitis and tenderness over maxilla in maxillary
sinusitis. Also mild fever may present in acute 1330. A young child dx with chicken pox.
sinusitis]. Usually goes to day care. What is the most
appropriate advice?
a. Child should be admitted to hospital
1328. A 51yo woman presents with straight away
painful tongue and complains of tiredness. b. Isolate the child from parents and siblings
She is pale and has angular stomatitis and at home
c. Advice that he can go back to nursery
a smooth red tongue. There is no when the rash is crusted over
koilonychea. Choose the single cell type
you will find on the blood film.
a. Numerous blast cells Ans. The key is C. Advice that he can go back to
b. Oval macrocytes nursery when the rash is crusted over. [At this
stage patient is no more infectious].
c. Spherocytes
d. Microcytic hypochromic
e. Mexican hat cells 1331. A 7yo boy is brought by his mother.
f. Erythrocytes There are multiple perioral and forehead
vesicles. Some vesicles are crusted and
some are not. The face is hot. What is the
Ans. The key is B. oval macrocytes. [Macrocytes most likely dx?
are two types, round and oval. Oval macrocytes a. Varicella zoster
are seen in megaloblastic anemia which occurs b. Herpes zoster
due to Vit B12and/or folic acid deficiency. c. Fungal infection
Glossitis (painful tongue), angular stomatitis and d. Impetigo
smooth tongue are though characteristic feature e. Psoriasis
of iron deficiency but also known feature of
megaloblastic anemia. Absence of koilonychea
also favours megaloblastic anemia].
Ans. The key is B. Herpes zoster.
Ans. The key is A. ventricular septal defect. Ans. The key is B. Hepatitis B full vaccine and Ig.
Ans. The key is A. Adenocarcinoma. Ans. The key is D. Clean the wond. [“NEXT” best
step is clean the wound].
1341. A 24yo man after a head injury
presents with difficulty dressing himself, 1344. A 56yo male presents with
difficulty in writing and inability to persistent watery diarrhea. What is the
differentiate the fingers of his hand. most likely dx?
Which part of the brain is most likely to be
a. Treponema pallidum
affected?
a. Frontal lobe b. Nesseria meningitides
b. Parietal lobe c. Cryptosporidium
c. Temporal lobe d. Staph aureus
d. Occipital lobe e. Pseudomonas aeruginosa
e. Brainstem
Ans. The key is A. Cryptococcus neoformans. [The 1356. A boy with a hx of recurrent swollen
key is doubtful as Toxoplasmosis is most common tender joints on both knees and elbows
cns lesion in AIDS]. and not able to participate in sports. What
is the inv of choice to dx?
a. RF/ASO titre
1353. A child has developed rash after the b. Clotting factor
tx of penicillin. What will be the cause of c. ESR
rash?
a. Drug reaction
Ans. The key is B. Clotting factor. [Haemarthrosis
b. Kawasaki
causing disability].
c. Inf Mono
1357. A 26yo man is referred for
gastroscopy because of a hx of several
Ans. The key is A. Drug reaction. months of dyspepsia. He has routine
bloods checked and is found to have a
serum calcium level=3.2mmol/l with a
1354. A child comes with recurrent joint venous bicarbonate level of 33mmol/l.
pain, multiple bruises, swollen ankle and Renal and LFT are both mornal. CXR is
unable to move his legs. What is the inv of normal. What is the most likely cause of
choice? his hypercalcemia?
a. ESR a. Melanoma
b. Metastatic malignancy Ans. The key is E. Tracheal intubation. [If a burnt
c. Milk alkali syndrome patient gets soots in mouth and /or nose tracheal
d. Primary hyperparathyroidism intubation is done].
e. Sarcoidosis
1359. A pt after his house fire came with Ans. The key is A. Budd-Chiari syndrome. [In
gastrinoma there is no ascites! In hepatoma there
hematemesis with erosion/ulcer of
will be exudative ascites as in TB and pancreatitis.
esophagus and on examination there is
Budd-Chiary syndrome causes transudative
55% burn and on endoscopy there is a ascites and the likely option here].
stomach/gastric erosion and soot in the
mouth. What is the tx?
a. PO PPI 1362. A 7yo boy presents with his mother
b. IV PPI to GP surgery. His mother describes he
c. PPI and antibiotic had presented this since 3wks ago. He had
d. H. pylori test not experienced any trauma. No other
e. Tracheal intubation symptoms a/w the condition. Exam: non
tender swollen ankles bilaterally. There is
no rash or lesion. He is otherwise well. onset rotational vertigo which comes on
Which single test would be the best as an with sharp movements of the head and
initial assessment? neck. Which of the following would be
a. Plasma electrolytes most helpful?
b. Albumin a. Caloric testing
c. Total serum protein b. Hallpikes maneuver
d. Anti-streptolysin c. Gutenbergers test
d. Meniere’s test
e. Otoscopy
Ans. The key is B. Albumin. [Likely case of ankle
oedema from hypoalbuminema].
Ans. The key is C. Cobalamin. [Pancreatic 1383. A pt, a small child presented with
insufficiency can lead to malabsorption and B12 URTI and later developed fever, earache
(cobalamin) deficiency. Dementia and diminished and tympanic membrane is dull. What is
vibratory sensation and proprioception in the the likely dx?
lower extremities were the most common a. OM
objective findings of cobalamin deficiency]. b. OE
c. Glue ear
d. Perforation of the tympanic membrane
1381. A man after MI presented with
e. Referred ear ache
sudden breathlessness and dyspnea.
Exam: scattered pansystolic murmur all
over the precordium. What is the next inv
Ans. The key is A. OM.
that will lead to dx?
a. ECG
b. Echo
c. CT
1384. A 72yo male who is a regular
d. Blood culture smoker has come to the ED with
e. CXR complaints of loss of weight and loss of
appetite. He also complains of
odynophagia. Exam: actively bleeding started on flucloxacillin. What other meds
ulcer on right tonsil. What is the most will you add?
appropriate dx? a. Ciprofloxacin
a. Tonsillar ca b. Gentamicin
b. Vincent’s angina c. Metronidazole
c. Irritant ingestion d. Benzylpenicillin
d. Paracoccidiodmycosis e. Ceftriaxone
e. Herpes simplex infection
1386. A pt with celiac disease from birth, Ans. The key is A. Scopolamine. [Any secretory
now as an adult presented with some condition of lung is reduced and improved with
abdominal symptoms. The biopsy shows scopolamine].
infiltration of the gastric epithelium by 1389. A pt presented after eating a seafood dish
lymphocytes. What is the most likely dx? at a local restaurant. He complains of difficulty in
a. Lymphoma breathing. His speech is slurred and his
b. Diverticular disease BP=85/55mmHg. What would be the most
c. Lynch syndrome appropriate next step?
d. Gastric TB a. IV adrenaline
e. Peritoneal tumor b. IM adrenaline
c. SC adrenaline
d. PO chlorpheniramine
Ans. The key is A. Lymphoma. [Coeliac disease e. IV chlorpheniramin
leads to an increased risk of both
adenocarcinoma and lymphoma of gut]. Ans. The key is B. IM adrenaline. [Patient is
getting anaphylactic shock with difficulty of
breathing and slurred speech (probably from
1387. A 55yo man presented with hot,
tongue swelling) which are indications of IM
raised, tender area of skin on his right leg. adrenaline].
He is febrile with rigors. He has been
Ans. The key is C. Hypnagogic hallucinations.
1390. A 7yo boy presents with proptosis [Hallucinations during going for sleep is
and periorbital edema. What is the hypnagogic and hallucinations during wake up
immediate action that needs to be taken? from sleep is hypnapompic hallucinations].
a. IV morphine and immediate
ophthalmoscopy
b. IV morphine 1393. A 32yo man on psychiatric meds
c. Observation only presents with coarse tremors and
diarrhea. What is the most likely alt to the
drug causing the prb?
Ans. The key is A. IV morphine and immediate a. Lithium
ophthalmoscopy. b. Diazepam
c. Haloperidol
d. Valproate
1391. A schizophrenic man complains that e. Citalopram
he can hear voices talking about him and
telling him to end his life by cutting his
throat. He only hears them when he Ans. The key is D. Valproate. [Diarrhea and coarse
wakes up from sleep and not at other tremors are well known side effects of valproate].
times. What type of hallucinations is he
having?
a. Somatic 1394. A man is brought to the ED after he
b. Kinesthetic was stabbed in the best. Chest is clear
c. Hypnogogic bilaterally with muffled heart sounds.
d. Hypnopompic BP=60/0mmHg, pulse=120bpm, JVP is
e. Lilliputian raised. What is the most probable dx?
a. Pulmonary embolism
Ans. The key is D. Hypnapompic hallucination.
b. Cardiac tamponade
[Hallucinations during going for sleep is
c. Pericardial effusion
hypnagogic and hallucinations during wake up
from sleep is hypnapompic hallucinations]. d. Hemothorax
e. Pneumothorax
1407. A 45yo man has undergone detox Ans. The key is C. Metronidazole.
and now wants a drug to stop him from
craving alcohol. What med would be that
drug of choice? 1410. A 35yo man has been given a dx of
a. Disulfiram allergic rhinitis and asthma. Exam:
b. Acamprosate peripheral neuropathy with tingling and
c. Thiamine numbness in a ‘glove and stocking’
d. Naloxone distribution. Skin elsions are present in the
e. Diazepam form of tender subcutaneous nodules. The
pt is responding well to corticosteroids.
What is the single most appropriate dx?
Ans. The key is B. Acamprosate. [Disulfirum is a a. AS
deterrent which does not reduce craving. b. Churg-strauss syndrome
Acramposate by restoring deranged brain c. Crytogenic organizing
chemical reduces craving]. d. Extrinsic allergic alveolitis
e. Tropical pulmonary eosinophilia
1412. A 68yo man on tx for an irregular Ans. The key is C. HSP. [Henoch-Schönlein
heart beat comes to the ED. He has purpura (HSP) is an acute immunoglobulin A
palpitations for the last 3h. Exam: (IgA)–mediated disorder characterized by a
pulse=regular, 154bpm. Carotid sinus generalized vasculitis involving the small vessels
massage settled his pulse down to 80bpm. of the skin, the gastrointestinal (GI) tract, the
What is the most likely rhythm kidneys, the joints, and, rarely, the lungs and the
disturbance? central nervous system (CNS). Rash on trunk,
a. SVT abdominal pain, arthritis, proteinuria and
b. V-fib hematuria are well known features of HSP].
c. VT
d. V-ectopics
e. A-fib 1415. A pt is on loop diuretics. What effect
do loop diuretics produce?
a. Low Na+, low K+
b. Low Na+, normal K+
c. Normal Na+, normal K+ 1418. A 49yo man comes with hx of cough
d. High Na+, low K+ and SOB. His CD4 count is measured as
e. High Na+, high K+ 350. CXR shows lobar consolidation. What
is the single most appropriate option?
a. Mycobacterium avium intercellular
Ans. The key is A. Low Na+, Low K+. [Loop b. CMV
diuretics causes hyponatremia and hypokalemia]. c. Streptococcus
d. Toxoplasmosis
e. Pneumocystis jerovici
1416. A 6yo girl is being investigated for
renal failure. She is found to have a
congenital abnormality of the insertion of Ans. The key is C. Streptococcus. [Features are
the ureters into the urinary bladder. What consistent with lobar pneumonia].
is the single most likely cause for renal
failure in this pt? 1419. A 32yo woman with prv hx of PID
a. SLE
now presents with severe abdominal pain.
b. PKD
Her LMP was 8wks ago. What is the most
c. Wilm’s tumor
probable dx?
d. Acute tubular necrosis a. Ectopic pregnancy
e. Reflux nephropathy b. Ovarian torsion
c. Hematometrium
d. Chronic PID
Ans. The key is E. Reflux nephropathy. [Reflux e. Cholecystitis
nephropathy is kidney damage (nephropathy)
due to urine flowing backward (reflux) from the
bladder toward the kidneys; the latter is called Ans. The key is A. Ectopic pregnancy. [PID is a risk
vesicoureteral reflux (VUR)]. factor for ectopic pregnancy].
Ans. The key is A. Laryngoscopy. [Probable 1428. A 20yo student came to the OPD
diagnosis is recurrent laryngeal nerve palsy]. with complains of headache, malaise, dry
cough, joint pain and vomiting. Exam:
temp=39C. CXR: patchy consolidation.
1426. A 40yo pt came to OPD with What is the single most likely causative
complaint of fever, pleuritic chest pain, organism?
productive cough and painful vesicles a. Pneumococcal pneumonia
around the lips. Exam: temp=38C. He has b. Mycoplasma
a hx of splenectomy last yr. What is the c. Klebsiella
single most likely causative organism? d. Streptococcus
a. Pneumococcal pneumonia e. PCP
b. Staphylococcus
c. Klebsiella
d. Streptococcus Ans. The key is B. Mycoplasma.
e. Chlamydia psitacci
Ans. The key is C. Pleural effusion. This is a wrong 1435. A 34yo IVDA (intravenous drug
key. The correct key should be B. Pleurisy. [The addict) presents with a 4m hx of
likely diagnosis is mesothelioma where there is productive cough. He has lost 10kgs.
pleural thickening and dullness. As there is pleural What is the single most appropriate inv?
rub still pleurisy is present and once effusion will a. Sputum for AFB
occur the rub will be lost and absent]. b. Laryngoscopy
c. Bronchoscopy
d. CT neck
1433. A 67yo female presents with e. CXR
balance prbs. Exam: nystagmus on left
lateral gaze, a loss of the left corneal
Ans. The key is A. Sputum for AFB. [In IVDA cavitation. What is the single most likely
immunity becomes low and easily gets infected causative organism?
with TB]. a. Legionella
b. Mycoplasma
c. Staphylococcus
1436. A 25yo pt came to the OPD with d. Klebsiella
complaint of fever, malaise, e. Streptococcus
breathlessness, cough and anorexia. His gf
has got similar symptoms. He had hx of
sore throat and ear discharge a month Ans. The key is D. Klebsiella. [Upper lobe
ago. What is the single most likely cavitation favours Klebsiella pneumonia].
causative organism?
a. Legionella 1439. A 20yo man complains that all his
b. Mycoplasma movements are being watched.
c. Chlamydia pneumonia Sometimes he feels as though his actions
d. PCP are being controlled by his radio. At other
e. Chlamydia psitacci times he is aware of voices describing
what he is doing. What is the most
probable dx?
Ans. The key is C. Chlamydia pneumonia. a. Mania
b. Drug induced psychosis
c. Delusion of control
1437. A 72yo male presents with acute d. Schizophrenia
confusion. He has been in the hosp for e. Korsakoff psychosis
2wks having been treated for a DVT. The
nurses have noticed that he became
increasingly drowsy. Exam: small scalp Ans. The key is D. Schizophrenia.
laceration, a GCS of 8 and bilateral up-
going plantar response.
a. Infection toxicity 1440. A 35yo is agitated and euphoric. He
b. Delirium tremens claims to be helping the prime minister
c. Extradural hematoma with economic policy, although this is not
d. Subdural hematoma true when checked. What is the most
e. Electrolyte imbalance likely dx?
a. Mania
b. Schizophrenia
Ans. The key is D. Subdural hematoma. [Even c. Hypomania
trivial head trauma can lead to subdural d. Drug induced personality disorder
hematoma. Presence of small scalp laceration, e. Delusion of grandeur
confusion and becoming increasingly drowsy
Glasgow coma scale of 8 are suggestive of
subdural hematoma].
Ans. The key is E. Delusion of grandeur. Key is
1438. A 50yo DM pt came to the OPD wrong! Correct key should be mania. [Agitated,
euphoric and delusion of grandiosity makes the
with complaint of fever, muscle ache, dry
likely dx to be “Mania”. Agitation and euphoria
cough and anorexia. Inv: CXR=upper lobe
are not feature of delusion of grandiosity but d. Dermatitis herpetiformis
mania]. e. Hyperthyroidism
f. Primary biliary cirrhosis
Ans. The key is E. 100% oxygen. [Oxygen Ans. The key is B. Pharyngeal Ca.
saturation is low. So we have to give oxygen
initially].
1456. A 42yo female who is obese comes
with severe upper abdominal pain and
1453. A 25yo man presented with painless
right shoulder tip pain with a temp=37.8C.
cervical lymphadenopathy with lethargy,
She has 5 children. What is the most
night sweats and itching. What is the
probable dx?
single most likely causative factor? a. ERCP
a. Lymphoma
b. LFT
b. Polycythemia
c. Serum amylase
c. IDA
d. MRCP
d. Uremia
e. US abdomen
e. Drug induced
1470. A 30yo lady complaining of right ear Ans. No key is given. Correct key is B. Reactionary
deafness with decreased corneal reflex hemorrhage. [Hemorrhage within 1st 24 hours
and past pointing. Acoustic analysis shows which usually occurs due to dislodgement of clot
SNHL. What is the next most appropriate or slippage of a ligature].
inv to do?
a. CT brain
b. CT acoustic canal 1473. A 40yo woman has had varicose
c. MRI brain vein surgery, planned as a day pt. After
d. MRI acoustic canal the op, she is distressed by repeated
e. PET brain retching and vomiting. Her pain is
currently well controlled. What is the best
management strategy?
Ans. The key is MRI brain. This is wrong key! a. Tramadol
Correct key is D. MRI acoustic canal. b. Co-codamol
c. IM morphine
d. IV ondansetron
1471. A 29yo woman who returned from
e. PO ondansetron
Egypt 2wks ago now presents with
difficulty in breathing, chest pain, cough
and purulent sputum with an episode of
Ans. The key is D. IVondensatron. [As there is
blood staining. She is on COCPs. What is
repeated retching and vomiting pt. cannot keep
the most likely dx? oral medication down. So IV ondansatron].
a. Pulmonary embolism
b. Pneumonia
c. Lung abscess
1474. A pt with renal failure has serum
d. Pneumothorax
K+=7.5, raised creatinine and broad
e. Pulmonary edema
complex tachycardia. What is the most
appropriate management? Ans. The key is D. Barium meal. [Probable
a. Calcium gluconate diagnosis is Crohn’s disease. Can be
b. Sodium bicarbonate demonstrated by barium meal].
c. Dialysis
d. Furosemide
e. Sotalol 1477. An 83yo elderly woman presented
in the ED with cough, fever and sneezing.
Tx was given but she became confused
Ans. The key is A. Calcium gluconate. [Calcium and again presented with above said
gluconate don’t shift K+ to cell or reduce symptoms. What is the cause of her
potassium level but it prevents arrythmogenic condition?
action of raised K+ till definitive measure is a. Aspiration due to confusion
taken]. b. Alveolar damage due to drugs
c. Drug toxicity
d. Pneumothorax
1475. An 18yo lady in her 30th wk of
pregnancy is brought to the hospital in an
altered sensorium. She is taking slow, Ans. The key is A. Aspirin due to confusion.
shallow breaths and her breath has a
fruity smell. An ABG: pH=7.20, urine 1478. A 37yo man presents with some
ketones: +ve. What is the most probable raised lesions on the shin. He came with
dx? cough and also complains of arthralgia.
a. HONK
Exam: bilateral hilar lymphadenopathy
b. DKA
and erythema nodosum is present. What
c. HELLP syndrome
is the single most likely cause?
d. PIH
a. CD
e. GDM
b. UC
c. Sarcoidosis
d. Streptococcal infection
Ans. The key is B. DKA. [Shallow breath and fruity e. TB
smell with acidosis in ABG and positive ketone
body in urine suggests the diagnosis of DKA].
Ans. The key is B. UC. It is a wrong key! Correct
key is C. Sarcoidosis. [Cough, arthralgia, bilateral
1476. A 26yo man presented with hilar lymphadenopathy and erythema nodosum
abdomen distension and pain. His stools are suggestive of sarcoidosis].
have been mucoid and sometimes blood
stained. What is the most appropriate 1479. A young lady with cervical ectropion
inv? bleeds on touch. What is the most
a. Stool C&S
appropriate next inv?
b. Gastroscopy
c. IgG tissue transglutaminase a. Transvaginal US
d. Barium meal b. Cervical smear
e. Jejunal biopsy c. Punch biopsy
d. Serum estradiol need basis. In such case if investigation is needed
e. Colposcopy colposcopy can be done].
1538. A 2yo fell on outstretched hand on Ans. The key is A. Foreign body. [Sudden onset of
playground. He presents with pain on dysphagia to both liquid and solid and recent
base of the thumb. XR=no fx. What is the history of fitting denture suggests foreign body
single most likely dx? (denture) in oesophagus].
a. Colles fx
b. Head of radius
c. Mellet finger 1541. A 62yo man with chronic
d. Scaphoid fx schizophrenia presents with a mask like
e. No fx face and involuntary pill rolling movement
in both hands. He complains of chronic
cough and forgetfulness. He is on long
Ans. The key is D. Scaphoid fx. [Scaphoid fracture term antipsychotic meds. What is the
is often missed in initial x-ray].
single most likely dx?
a. Shy drager syndrome
1539. A pt was admitted with increased b. Parkinsonism
frequency of passing urine, increased c. Huntington’s chorea
thirst, weakness and muscle cramps. d. Tardive dyskinesia
What is the most probable dx? e. Akathisia
a. Conn’s syndrome
b. Cushing’s syndrome
c. Pheochromocytoma Ans. The key is B. Parkinsonism. [Antypsychotic
d. Hyperthyroidism drugs can cause parkinsonism].
e. Hypoparathyroidism
Ans. The key is B. Flial chest. [In flial chest 1571. A 70yo man presents with a
multiple adjacent ribs are broken in multiple fluctuant swelling of the scrotum which
places, separating a segment, so a part of the feels like worms when he is standing but
chest wall moves independently. Pain, difficulty
regresses when he lies down. What is the
breathing, tachypnea and tender area of chest
wall movement suggests flial chest]. most probable dx?
a. Varicocele
1569. A 37yo woman had an elective LSCS b. Hematocele
1d ago. You are called to see her as she c. Testicular ca
becomes SOB with left sided chest pain d. Epidydimal cyst
and a cough. She has had 3 children, 2 e. Saphena varix
born by LSCS. Exam: she has reduced air
entry at left lung base. Her observations
include sat=92% on air, BP=105/84mmHg, Ans. The key is A. Varicocele. [Fluctuant swelling
of the scrotum which feels like worms are
pulse=120bpm, temp=37.2C. Choose
suggestive of varicocele].
among the options which C-section
complications has she developed?
a. Aspiration pneumonia
b. Aspiration pneumonitis
1572. A 52yo woman has had a swelling in
c. Spontaneous pneumothorax
the neck, hoarseness and stridor-both
d. Pulmonary embolism inspiratory and expiratory for 2m. What is
e. DVT the most probable dx?
a. Ca larynx
b. Ca thyroid
c. Vocal chord nodules
Ans. The key is D. Pulmonary embolism.
d. Ca bronchus
e. Thyrotoxicosis
1570. A pt presents with increasing
retrosternal pain and dysphagia for both
Ans. The key is B. Ca thyroid. Bruit over 1 or both subclavian arteries or
the abdominal aorta
Arteriographic narrowing or occlusion of
the entire aorta, its primary branches, or
1573. A woman became acutely SOB in
large arteries in the upper or lower
the recovery bay and is coughing after GA. extremities that is not due to
Auscultation: reduced air entry at the arteriosclerosis, fibromuscular dysplasia,
right lung base and diffuse wheeze. or other causes].
Observation: HR=88bpm,
BP=112/76mmHg, temp=37.8C and 1575. A 35yo woman presents with mass
sat=91% in air. Choose among the options
in the groin. Exam: mass found just below
which C-section complication has she and lateral to the pubic tubercle. There is
developed?
no cough impulse and it is irreducible.
a. Aspiration pneumonitis
What is the most probable dx?
b. Spontaneous pneumothorax
a. Direct inguinal hernia
c. Endometritis
b. Strangulated hernia
d. Pulmonary embolism
c. Femoral hernia
e. Tension pneumothorax
d. Saphenavarix
e. Femoral aneurysm
Ans. The key is E. Femoral hernia. 1580. A 25yo man present with a mass in
the groin after heavy lifting. Exam: mass is
found just above and medial to the pubic
1578. A 32yo woman of 38wks GA attends tubercle. It is reducible. On applying
the antenatal day unit with pain in the pressure on the internal ring, cough
suprapubic area that radiates to the upper impulse is still present. What is the most
thighs and perineum. It is worse on likely dx?
a. Direct inguinal hernia
walking. Her urine dipstick showed a trace
b. Indirect inguinal hernia
of protein but no white cells, nitrates or
c. Femoral hernia
blood. What’s the most likely dx?
a. Braxton hicks contractions d. Strangulated hernia
b. Round ligament stretching e. Femoral aneurysm
c. Symphasis pubis dysfunction
d. Labor
e. Complicated femoral hernia Ans. The key is A. Direct inguinalhernia. [On
occlusion of deep inguinal ring if cough impulse
still palpable (actually more appropriate is visible)
on medial to occluded ring it is direct inguinal
Ans. The key is C. Symphysis pubis dysfunction.
hernia].
Ans. The key is C. Facial nerve palsy. [Due to 1591. A 6yo boy has been noticed to have
malignant infiltration]. problems with co-ordinating his voluntary
movements over the last two years. He
has a waddling gait and needs to support
1589. A 6yo boy presents with jaundice himself on his hands when rising from the
following treatment with sulphathiazole. floor. He has larger calves than other boys
Investigations suggest that the jaundice is but he runs more slowly. Which is the
due to haemolysis caused by G6DP most likely dx?
deficiency. Which is true regarding a. Myotonia
etiology of G6DP deficiency? b. Myasthenia gravis
a. Inherited as autosomal dominant c. Duchenne muscular dystrophy
condition d. Muscular atrophy
b. Inherited as sex-linked dominant
condition
c. Inherited as sex-linked recessive Ans. The key is C. Duchenne muscular dystrophy.
condition [The child having difficulty with walking, running,
d. Results from auto-antibodies to red cell jumping and climbing stairs. Walking may look
antigens different with a 'waddling' type of walk. The boy
may be late in starting to walk (although many
children without DMD also walk late).
Ans. The key is D. Measurement of varicella IgG 1600. A 31yo woman, G5P4, who has
level. [If previous infection is doubtful do varicella amenorrhoea for 12 weeks and a positive
IgG level]. pregnancy test presents to the ED with
vaginal bleeding. Symphysial-fundal
height measurement corresponds to 22
1598. A 24yo primigravida presents to the weeks gestation. Ultrasound examination
ED with a history of 8-week amenorrhoea reveals bilateral cystic masses. No fetal
followed by heavy vaginal bleeding and parts are seen during the examination.
severe, crampy abdominal pain. Exam: The cervix is closed. Which is the most
HR=110/min and BP=120/80mmHg. The likely dx?
uterus is bulky. The cervix is dilated and a. Tubal pregnancy
there is active bleeding from the cervical b. Endometriosis
os, but no tissue has been expelled. c. Hydatidiform mole
d. Threatened abortion
Risk factors:
Ans. The key is C. Hydatidiform mole. [In molar Fertility treatments and intrauterine
pregnancy uterus is more enlarged than contraceptive devices (IUCDs) are the
gestational age and on US no fetal part but cystic most important associated risk factors.
masses are seen]. Pelvic inflammatory disease may cause
complete tubal occlusion or delay the
transport of the embryo so that
implantation occurs in the tube.
The final set (1601-1700) is done by DR. ARIF
Adhesions from infection and
SIDDIQUI. Thanks for Dr. Siddiqui for his kind inflammation from endometriosis may
contribution! play a part.
Ectopic pregnancy has been reported in
tubes that have been divided in a
1601. A married 25yo woman presents sterilisation operation and where they
with 6h hx of abdominal pain located in have been reconstructed to reverse one.
Ectopic pregnancy can occur in the
the LIF. The pain is persistent, of
treatment of infertility.
increasing intensity and not radiating first Right-sided tubal pregnancy is more
experienced while she was lying down. common than left-sided. This is thought
She feels giddy when she tries to stand to be from spread of infection from
erect. The last menstrual period was 6 appendicitis.
weeks ago. The radial pulse=130/min and Presentation:
Symptoms and signs of ectopic pregnancy
BP=80/40 mmHg. Pelvic US shows free
can resemble those of other more
intra-peritoneal fluid. What is the most common conditions, including urinary
appropriate next step in management? tract infections and gastrointestinal
a. Immediate laparoscopy. conditions.
b. Immediate laparotomy. The most common symptoms are:
c. Pregnancy test (urine or serum).
* Abdominal pain.
d. Observation for 24 hours in the ICU * Pelvic pain.
* Amenorrhoea or missed period.
* Vaginal bleeding (with or without clots).
Key: Pregnancy test (urine or serum) (C) * Other symptoms may include:
* Dizziness, fainting or syncope.
Reason: This is a classic case of ruptured Ectopic * Breast tenderness.
pregnancy (though most rupture occurs a bit late * Shoulder tip pain.
that is 7/8 week) and the best step would * Urinary symptoms.
pregnancy test to establish the diagnosis. The * Passage of tissue.
immediate next step if the patient is * Rectal pain or pressure on defecation.
hemodynamically unstable would be an •There may be a history of a previous ectopic
immediate laparoscopy. Observation wouldn’t pregnancy. After one ectopic pregnancy the
help anyone and a laparotomy would be chance of another in the other tube is much
considered in third-world countries. increased.
Tests:
Key: Traumatic rupture of the aorta (E)
The most accurate method to detect a tubal
pregnancy is transvaginal ultrasound. (A traumatic aortic disruption is caused by a rapid
acceleration (or deceleration) causing a tear in
* This can identify the location of the pregnancy the aorta. Normally this is immediately fatal, but
and also whether there is a fetal pole and those who survive may show a widened
heartbeat. mediastinum on CXR. This can be confirmed with
* Human chorionic gonadotrophin (hCG) levels CT scan or angiography of the aorta and requires
are performed in women with pregnancy of prompt surgical correction. Stable the
unknown location who are clinically stable. haemodynamics and surgical correction. Note: pts
with diaphragmatic rupture usually presents days
* hCG levels are taken 48 hours apart. If there is a after trauma with vague symptoms, history is
change in concentration between 50% decline vital)
and 63% rise inclusive over 48 hours then the
woman should be referred for clinical review in
an early pregnancy assessment service within 24 Reason: The wide mediastinum and the radio-
hours femoral delay make Traumatic rupture of the
Management: Arch of the Aorta the most likely diagnosis in this
case. Tension Pneumothorax will not present this
* Admit as an emergency if the diagnosis of way and a rib fracture would be less severe than
ectopic pregnancy is considered a possibility. A the patient is currently. Diaphragmatic rupture
bedside pregnancy test should be performed on would have bowel sounds in the chest and
all women of childbearing age presenting with cardiac tamponade would have Beck’s triad –
lower abdominal pain where pregnancy is even
Muffled heart sounds, distended neck veins and
the remotest possibility.
decreased blood pressure.
* Anti-D rhesus prophylaxis should be given (at a
dose of 250 IU) to all rhesus negative women who
have a surgical procedure to manage an ectopic
pregnancy. 1603. A 36yo woman presents with
* Conservative management may be appropriate swelling in the groin. Exam: swelling is
if the levels of hCG are falling and the patient is diffuse and soft and lies below the
clinically well. Repeat hCG levels are performed in
inguinal ligament. It empties with minimal
these cases.
pressure and refills with release. There is a
cough impulse and it disappears on lying
1602. A 40yo man has fallen off a roof. He down. On the calf of the same leg there
is shocked and has chest pain. There is a are varicosities on the medial aspect.
delay between the radial and femoral What is the most likely dx?
pulse. His CXR=widening of the a. Varicose vein
mediastinum. What is the single most b. Varicocele
likely dx? c. Saphena varix
a. Cardiac tamponade d. Femoral hernia
b. Diaphragmatic rupture e. Inguinal hernia
c. Fx ribs f. Key: Saphena Varix (C)
d. Tension pneumothorax
Reason: The emptying on lying down and the ache with time. Bulge can often be seen in the
coexistence of varicose veins make a saphena lower abdomen with the patient erect and
varix the most likely answer to this question. It is straining. This can be reduced by pressure with a
a dilatation of the saphenofemoral junction due 'gurgling' noise and then the hernia orifice can
to incompetent valves when the saphenous vein often be felt. However, the defect may not be
drains into the femoral vein. seen as a swelling palpable or a bulge may be found distant from
the site. This needs prompt repair.
around 2-4cm inferio-lateral to the pubic
tubercle). This may be mistaken for a hernia, but
a Saphena Varix has a bluish tinge
1605. A 70yo man presents with acutely
Femoral and Inguinal hernia not possible because painful, pale paralysed and pulseless left
the swelling is diffuse and empties with minimal
pressure. Varicose veins are present but they are
leg. He is noted to have a-fib. What is the
not the choice because the saphena varix is most probable dx?
immediately below the inguinal ligament. a. Intermittent claudication
b. Cardiovascular syphilis
c. Buerger’s disease
1604. A man presents with a swelling d. Chronic limb ischemia
above the groin crease in the abdomen. e. Acute limb ischemia
He has not had any med problems of note.
What is the most probable dx?
a. Inguinal hernia
Key: Acute limb ischemia (E)
b. Spigelian hernia
c. Testicular tumor Reason: This is acute presentation of limb
d. Epidydimal cyst ischemia because the atrial fibrillation has thrown
e. Irreducible hernia a clot which has blocked arteries in the leg
leading to this acute picture. Chronic ischemia
would present with amputation or gangrene plus
lipodermatosclerosis of limbs, intermittent
Key: Spigelian Hernia (B) Reason: A Spigelian claudication would present with pain on walking
hernia (or lateral ventral hernia) is a hernia and no pain on rest, Buerger’s disease would
through the spigelian fascia, which is the need a smoking history and syphilis of the CVS
aponeurotic layer between the rectus abdominis would have aneurysms of the large arteries rather
muscle medially, and the semilunar line laterally. than small vessel involvement leading to limb
There is a common misconception that they symptoms.
protrude below the arcuate line owing to Acute limb ischaemia is most often due to either
deficiency of the posterior rectus sheath at that acute thrombotic occlusion of a previously
level, but in fact the defect is almost always partially occluded, thrombosed arterial segment,
above the arcuate line. These are generally or to embolus from a distant site. Without
interparietal hernias, meaning that they do not lie surgical revascularisation, complete acute
below the subcutaneous fat but penetrate ischaemia leads to extensive tissue necrosis
between the muscles of the abdominal wall; within six hours.
therefore, there is often no notable swelling. All Presentation: The affected part becomes pale,
the other options lie below the groin crease i.e. pulseless, painful, paralysed, paraesthetic and
the inguinal ligament. 'perishing with cold' ('the 6 Ps')
Discussion:
•If the occlusion is embolic, the options are 1606. A 50yo woman complains of several
surgical embolectomy (Fogarty balloon months hx of weakness and difficulty
embolectomy catheter) or local intra-arterial climbing stairs. Exam: fissuring of the skin
thrombolysis: of her hands. CXR: pulmonary fibrosis.
What is the single most likely positive
antibody?
•If embolectomy with a Fogarty catheter fails, an a. Anti Jo1
on-table angiogram is performed and bypass graft b. Anti Scl 70
or intraoperative thrombolysis considered.
c. Anti Ro
Routine intraoperative angiography for arterial
d. Anti dsDNA
thromboembolectomy has been shown to be
e. Anti centromere
beneficial.
Reason: These symptoms point towards Reason: The patient is high risk on the suicide risk
Ankylosing Spondylitis with the episodic back pain assessment scale and doesn’t warrant out-patient
that is relieved by exercise and NSAIDs for four follow up or discharge to be seen by a GP. He
years. The presentation is that of Uveitis which is should be admitted and treated in the Psychiatry
associated with autoimmune diseases, specifically ward as needed. Referral to the local alcohol
Iritis with acutely painful red eye. Chorioretinitis treatment team or clinical psychologist is never
would present in the same way, but 1/3 or AS the right answer.
Neutrophils=0.1, plt=14. No abnormal
white cells were seen on the blood film.
1627. A healthy baby boy is born at term
She was transfused and given IV
to a woman who was unwell with
antibiotics and her condition improved.
confirmed acute hep B during pregnancy.
3wks later her blood count has returned
The mother is very concerned that she
to a similar picture. What is the SINGLE
may have infected the baby with hep B.
most likely underlying dx?
What SINGLE preventative intervention
a. ALL
should be given to the baby?
b. AML
a. Full course of hepatitis B vaccine
c. Aplastic anemia
b. Hepatitis B immunoglobulin alone
d. CML
c. Hepatitis B vaccine and hepatitis B
e. Pernicious anemia
immunoglobulin
d. Hepatitis B vaccine as single dose
e. None until hepatitis B status confirmed
Key: Aplastic Anaemia (C)
Ans)C
Reason: The age of the patient and pancytopenic
Key)Babies born to mothers infected with picture give us a clinical diagnosis of Aplastic
hepatitis B have a high risk of acquiring infection, anemia. Normal WBC morphology rules out ALL,
which can be prevented by vaccination at birth.All AML and pernicious anaemia while the age rules
babies with seropositive mothers should have the out CML as a diagnosis.
full primary course of hepatitis B immunisation
and most should also have HBIG within 24 hours
of birth. 1629. An 83yo woman admitted with a
chest infection becomes confused with
impaired attention and poor
Reason: If the mother has active confirmed acute concentration. She is restless and
Hep B, give Immunoglobulins and vaccinate the
frightened. She is verbally abusive and has
baby at birth. Also perform serology of the baby
perceptual abnormalities. There is no
at 12-15 months (1 year). If Hep B Antibodies +ve
and HbSAg -ve, do nothing. Full course of the significant prv psychiatric hx. What is the
vaccine is unnecessary because the SINGLE most likely dx?
aforementioned course will provide sufficient a. Delirium
coverage, Ig alone and Vaccine alone will not be b. Drug induced psychosis
enough for the baby. Full course with c. Lewy body dementia
immunoglobulin will be given at BIRTH, 1 d. Multi-infarct dementia
MONTH, 2 e. Psychotic depression
Reason: There are 1750 cases a 5 year period in Key: Amoxicillin (A)
this scenario. The number of cases in 1 year in the Reason: This is the picture of Acute Otitis Media
0.5 million people in this town are 350 cases/0.5 which has led to tympanic membrane
million people. To calculate the incidence per perforation. PO Amoxicillin for 7 days is the
million of bladder cancer, we would simply treatment of choice with appropriate analgesics.
double the number to get an estimate which Amoxicillin +/- Clavulanate is the first line drug
would give us 350 x 2 = 700 cases/million. because it fits the organisms responsible which
are Strep pneumonia, Moraxella catarrhalis and
H. influenza.
1631. A 28yo woman who has had a prv
pulmonary embolism in pregnancy wishes
to discuss contraception. She has 1633. A 38yo man has disturbing thoughts
menorrhagia but is otherwise well. What about his house being infected by germs.
is the SINGLE most suitable contraceptive He is anxious about safety and checks the
method for this patient? locks of his doors repeatedly before going
a. COCP to bed. For the last 8wks he has been
b. Copper IUCD washing his hands every time he touches
c. Levonorgestrel intra-uterine system the lock, 20-30 times a day. What is the
d. Progestogen implant
SINGLE most appropriate management?
e. POP a. Antidepressant
b. Antipsychotic
c. Anxiolytic
Key: Levonorgestrel Intra-Uterine System (C) d. CBT
Reason: The woman has a history of e. Psychodynamic psychotherapy
thromboembolic disease, which essentially rules
out COCPs. POPs, Copper IUCD (Copper – T) and
Key: CBT (D) She has two young children. What is the
Reason: This scenario describes a case of OCD for SINGLE most likely dx?
which the best management is CBT followed by a. Psoriasis
SSRIs or TCAs. The first line treatment is always b. Reactive arthritis
CBT, not pharmacological therapy. Psychotherapy c. Rheumatoid arthritis
is indicated in depression, psychosomatic d. Sarcoidosis
disorders, dissociative or conversion disorders, e. SLE
personality disorders, relationship problems or
grief. key)B
Ans)
1643. A 48yo man with renal cancer had Patient is suffering from HFMD (Hand,Foot and
radiotherapy for metastatic spinal cord Mouth Disease)
compression at the 11th thoracic vertebra • HFMD is due to an infection that usually
4wks ago. He has retained sensation but is causes a typical illness, including a typical rash. It
unable to stand. He has pain in a band is most commonly caused by the Coxsackie A16
around his lower trunk controlled by virus
• HFMD most commonly affects children
regular oral morphine. He is distressed by
under 10 years of age
increasingly frequent episodes of painful • This might include a high temperature
muscle spasms in his right leg. What is the (fever). After this, a sore throat commonly occurs,
SINGLE most appropriate management of quickly followed by small spots that develop
his symptoms? inside the mouth. These soon progress into small
a. Amitriptyline mouth ulcers
b. Baclofen • In many cases, spots also develop on the
skin. This is typically a day or so after the mouth
c. Fentanyl patch
ulcers develop. The spots are small lumps that are
d. Gabapentin a few millimetres in diameter and usually appear
e. Increase morphine dose on the hands and feet, they are not usually itchy
but sometimes they can be a little bit sore.
Treatment:
Key is B: Baclofen FOR MUSCLE RELAX • There is no treatment that will take away
the virus
Spinal metastases, often Also cause neuropathic
pain with a radicular component (sensory deficit,
burning pain, painful shock-like sensations) which
1645. A 32yo woman has had 3 episodes
requires the additional administration of of slurred speech and 2 episodes of
anticonvulsants (e.g., gabapentin) and perhaps transient weakness of both legs in the
past 5yrs. Each episode has resolved in • Weakness or paralysis of some muscles.
3m. What is the SINGLE most likely dx? Mobility may be affected.
a. Meningioma • Problems with passing urine.
• Inability to have an erection in men.
b. Migraine
• Difficulty with speaking.
c. Multiple sclerosis Investigation and Treatment:
d. Stroke • A firm diagnosis of MS is often not made
e. Transient ischaemic attack until two or more relapses have occurred
• A magnetic resonance imaging (MRI) scan
of the brain is a useful test. This type of scan can
detect small areas of inflammation and scarring in
Key is C: Multiple sclerosis
the brain which occur in MS.
• At present, although there is no cure for
MS.
Points in favor: symptoms have a relapsing-
remitting form
• MS is thought to be an autoimmune
disease 1646. An 8yo girl is complying with her
• About 1 in 600 people in the UK develop asthma treatment of low-dose inhaled
MS. It can affect anyone at any age, although it is
corticosteroid prophylaxis and short-
rare in young children. It most commonly first
develops around the age of 30. MS is the most acting bronchodilators as required. Her
common disabling illness of young adults in the inhaler technique is good. She now has a
UK. It is twice as common in women as in men. frequent night cough and mild exercise-
symptoms include: induced wheeze. What would be the
• Visual problems SINGLE most appropriate change in her
The first symptom of MS for around one in four
treatment?
people with MS is a disturbance of vision.
a. Add leukotriene antagonist
Inflammation (swelling) of the optic nerve can
occur. This is called optic neuritis. This can cause b. Add oral theophylline
pain behind your eye and also some loss of your c. Add regular long-acting bronchodilator
vision. This usually only affects one eye. d. Increase dose of inhaled corticosteroid
• Muscle spasms and spasticity e. Short course of oral corticosteroid
Tremors or spasms of some of your muscles may
occur. This is usually due to damage to the nerves
that supply these muscles. Some muscles may
shorten (contract) tightly and can then become Key is D: Increase dose of inhaled corticosteroid
stiff and harder to use. This is called spasticity. A common treatment plan for a typical person
• Pain
with moderate asthma is:
There are two main types of pain that may occur
in people with MS: • A preventer inhaler (usually a steroid
Neuropathic pain inhaler), taken each morning and at bedtime. This
Musculoskeletal pain usually prevents symptoms throughout the day
• Fatigue and night.
Extreme tiredness (fatigue) is one of the most • A reliever inhaler (short acting
common symptoms of MS. bronchodilators) may be needed now and then if
Other symptoms which may occur include: breakthrough symptoms occur. For example, if
• Numbness or tingling in parts of the skin. symptoms flare up when you have a cough or
This is the most common symptom of a first cold.
relapse.
• If exercise or sport causes symptoms then a 1648. An 84yo woman with Alzheimer's
dose of a reliever inhaler just before the exercise dementia has recently become incontinent
usually prevents symptoms.
and more confused than usual. What is
• The dose of the preventer inhaler may
need to be increased for a while if you have a the SINGLE most likely dx?
cough or cold, or during the hay fever season a. Detrusor overactivity
b. Neuropathic bladder
c. Nocturnal enuresis
1647. A 38yo man with longstanding d. UTI
alcohol dependence has vertigo and a e. Uterine prolapse
tremor every morning. What is the SINGLE
most likely dx?
a. Anxiety
Key is D: UTI
b. Benign positional vertigo
c. Cerebellar degeneration
d. Optic neuritis
1649. A 4yo boy complains of pain around
e. Temporal lobe epilepsy
his right eye. He is unwell, febrile and also
suffers from pain on the right side of his
Key is C: Cerebellar degeneration face. What is the most probable dx?
a. Allergic reaction
• Cerebellar degeneration is a process in
b. Furuncle
which neurons in the cerebellum - the area of the
brain that controls coordination and balance - c. Folliculitis
deteriorate and die d. Foreign body
Associated diseases: e. Periorbital cellulitis
• ischemic or hemorrhagic stroke, when
there is lack of blood flow or oxygen to the
cerebellum
Key is E: Periorbital cellulitis (also called preseptal
• cerebellar cortical atrophy, multisystem
atrophy, and olivopontocerebellar degeneration, cellulitis)
progressive degenerative disorders in which • preseptal cellulitis occurs at younger ages
cerebellar degeneration is a key feature (80% of patients are under 10 years of age and
• Friedreich’s ataxia, and other most are younger than 5 with a mean age of 21
spinocerebellar ataxias, which are caused by months)
inherited genetic mutations that result in ongoing Preseptal cellulitis:
loss of neurons in the cerebellum, brain stem, • Acute onset of swelling, redness, warmth
and spinal cord and tenderness of the eyelid.
• transmissible spongiform encephalopathies • Fever, malaise, irritability in children.
(such as Creutzfeldt-Jakob disease) in which • Ptosis
abnormal proteins cause inflammation in the • Diagnosis: is usually made based on the
brain, including the cerebellum clinical findings
• multiple sclerosis, in which damage to the Management:
insulating membrane (myelin) that wraps around • Most children are initially admitted to
and protects nerve cells can involve the hospital (even for preseptal cellulitis) unless there
cerebellum is good reason not to. This may be just for 24
Other diseases that can cause cerebellar hours. Children should be considered to have
degeneration include: orbital cellulitis until proven otherwise (ie
• chronic alcohol abuse that leads to repeated examinations normal, good response to
temporary or permanent cerebellar damage antibiotics in first 24 hours and normal CT scan).
• Oral co-amoxiclav may be used for both movements of both legs. Based on the
adults and children as long as there is no allergy likely clinical dx, which one of the
to penicillin. Clinical improvement should occur
following genetic patterns is most likely?
over 24-48 hours.
a. AD inheritance with anticipation
b. AD with variable penetrance
1650. A pt presents with irregularly c. AR
irregular pulse of 162bpm. What drug is d. X-linked
most useful initially? e. Mitochondrial disorder
a. Amiodarone
b. Digoxin Key is A: AD inheritance with anticipation
c. Bisoprolol o Patient is suffering from Huntingtons disease
d. Warfarin and that is autosomal dominant with
e. Heparin anticipation which means a genetic disorder
is passed on to the next generation, the
Key is C: Bisoprolol (most likely its acute AF and
symptoms of the genetic disorder become
1st line is verapamil/bisoprolol, 2nd line
digoxin/amiodarone) apparent at an earlier age with each
generation.
Bisoprolol is Beta blocker used as
antihypertensive and to control the heart rate Huntingtons Disease:
aswell.
• It is an inherited (genetic) condition that
Bisoprolol slows down the activity of your heart affects the brain and nervous system. It can
by stopping messages sent by some nerves to interfere with movements of your body, can
your heart. It does this by blocking tiny areas affect your reasoning, awareness, thinking and
(called beta-adrenergic receptors) where the judgement (cognition) and can lead to a change in
your behavior
messages are received by your heart. As a result,
• This faulty gene is carried on chromosome
your heart beats more slowly and with less force.
4.
• HD affects between 5-10 people per
100,000 in the UK.
1651. A 59yo man has shown a change in Presentation
his mood and personality over a 9m The symptoms of HD can be grouped into three
period. He has subsequently developed main areas:
difficulty with memory and conc, and then • Problems with movement
progressive fidgety movements of his • Problems with cognition
limbs and facial musculature. By the time • Mood and behavioural problems
Treatment:
of medical assessment he has frank
choreiform movements and a mini-mental • At present there is no cure for HD. Also,
state exam of 21/30. Other exam is there is no treatment that has been found to
normal. He was adopted and therefore no delay the onset of symptoms or to delay the
progression of symptoms
information on his famhx is available. He
has 3 adult children (27, 30, 33) of whom
the 2 youngest are asymptomatic. 1652. A 35yo pt has been dx with
However, the oldest son has recently been schizophrenia. He mimics the doctors and
inv by the neurology dept for slightly attendants – doing the same physical
erratic behavior and fidgety restless
actions as them. What symptom does this • Schizophrenia develops in about 1 in 100
pt have? people. It can occur in men and women. The most
a. Echopraxia common ages for it first to develop are 15-25 in
men and 25-35 in women.
b. Echolalia
Presentation:
c. Perseveration • Delusions
d. Apraxia • Hallucinations.
e. Anosognosia • Disordered thoughts.
Excluded Points:
1671. A pt being sedated with fentanyl Serum ferrtitin Level reduced, normal 12-15
develops severe respiratory depression. mcg/L, ( serum ferritin is falsely raised during
This is best reversed using? infections)
a. Ethanol Anisocytosis and poikilocytosis
b. Naloxone
c. Phyostigmine Total iron binding capacity is increased
d. Atropine
e. Methylene blue
f. Diphenhydramine Treatment: Iron supplememtation with B12 and
g. Calcium disodium ethylene
folic acid
h. diamine tetra-acetic acid
i. Deferoxamine mesylate
j. Flumazenil
k. Folic acid 1673. A 20yo prv healthy woman presents
with general malaise, severe cough and
Ans: Naloxone breathlessness which has not improved
with a seven day course of amoxycillin.
Opioid Antagonist, reverses the effects of There is nothing significant to find on
fentanyl, though it has to be administeres for a examination. The x-ray shows patchy
longer period of time due long half life of
shadowing throughout the lung fields. The
fentanyl.
blood film shows clumping of red cells
with suggestion of cold agglutinins.
a. Mycobacterium avium complex
1672. A pt presented with the following b. Coxiella burnetii
blood work, MCV: Decreased c. Escherichia coli (Gram -ve)
Serum ferritin: Decreased Total ,iron d. Haemophilus influenza
binding capacity: Increased ,Serum iron: e. Legionella pneumophila
Decreased f. Strep pneumococcus
g. TB
Marrow iron: Absent. What is your dx? h. Mycoplasma pneumonia
a. Thalassemia trait i. PCP
b. Hypoparathyroidism j. Staph aureus
c. Hereditary sideroblastic anemia
d. Protein energy malnutrition
e. Chronic renal failure Ans: Mycoplasma pneumonia
f. Anemia of chronic disease
g. Acute blood loss Reason : inablity to respond to a seven day course
h. IDA of amoxicillin suggests atypical pneumonia,
patchy shadows throughout lung fields and cold symptoms. After a few wks, each of the 7
agglutination points towards mycoplasma. people completely recovers and they
replace their caterer. What is a likely dx?
a. Pancreatic ca b. Hemochromatosis
M. pneumoniae:[9] b. Laennec’s cirrhosis
c. Hep A
Vague and slow-onset history over a few days or
d. HCC
weeks of constitutional upset, fever, headache, e. Rotor’s syndrome
dry cough with tracheitic ± pleuritic pain, myalgia, f. Primary biliary cirrhosis
malaise and sore throat. g. Gilbert’s syndrome
h. Hep B
This is like many of the common viral illnesses but
i. Hemolysis
the persistence and progression of symptoms is
what helps to mark it out.
Ans: Hepatitis A
In otherwise healthy individuals, it usually
resolves spontaneously over a few weeks.
The hacking, dry cough can be very persistent. Symptoms of Hepatitis A range from mild nauseas
to liver failure (very rare).Spread is normally by
Extra-respiratory features include rashes such as
the faecal-oral route although there are
erythema multiforme, erythema nodosum and
occasional outbreaks through food sources.Hand
urticaria; neurological complications like Guillain-
washing and good hygiene around food and drink
Barré syndrome, transverse myelitis, cerebellar
prevent spread of infection.Increasing age is a
ataxia and aseptic meningitis; haematological
direct determinant of disease severity.
complications such as cold agglutinin disease and
haemolytic anaemia; joint symptoms like
arthralgia and arthritis; cardiac complications
S/S The incubation period is 2-6 weeks with a
such as pericarditis and myocarditis; rarely, may
mean of 4 weeks.
cause pancreatitis
There is a prodrome of mild flu-like symptoms
TReatment: Macrolides, Fluroquinolones
(anorexia, nausea, fatigue, malaise and joint pain)
preceding the jaundice. Smokers often lose their
taste for tobacco. Diarrhoea can occur,
particularly in children.
Ans:6 months
1675. A 35yo 1st time donor suddenly social smile 6 weeks, becomes increasingly
socially responsive.
passes out as she is donating blood.
Which of the following steps would be palmer and rooting reflexes are primitive reflexes
least useful in managing this adverse
event?
1677. A mother is concerned because her
a. Ensure donor is adequately hydrated and
1m boy has a swelling in his scrotum. He
has not skipped a meal
was born prematurely. On examination
b. Elevating the donor's legs as this is
usually due to a vasovagal syncope
the swelling is seen to transilluminate. The
c. Haemoglobin of the donor meets the
likely cause is?
a. Lymphogranuloma Venereum
minimum requirement for donation
b. Testicular Torsion
d. The donation is usually continued along c. Hydrocele
with simultaneous normal saline infusion d. Epididymitis
e. The donor should be encouraged to e. Seminoma
mobilise after they have recovered f. Mature teratoma
g. Varicocele
h. Lymphoma
i. Orchitis
Ans: c j. Spermatocele
Option c has no role after such an event has
Ans: Hydrocele
occured.
Reason: 1-2% neonates present with congenital
hydrocele which disappears by 1-2 years.
1676. An infant is being examined as part
of a routine examination. The child can
hold its head up and lifts its chest off a Spermatoceles:Smooth, extratesticular, spherical
table. He has a palmer and rooting reflex cysts in the head of the epididymis are not
as well as a social smile. He is not afraid of uncommon in adult men. They are benign and do
not usually require treatment.Epididymal cysts
strangers. What is the most likely age of
usually develop in adults around the age of 40.
this child?
Epididymal cysts are rare in children and when
a. neonate
they occur, usually present around puberty.
b. 2 months
c. 6 months
d. one year
1678. A 2m girl has an ante-natal through the first year of life, and their parents
diagnosis of right hydronephrosis. counseled regarding the risk of urinary tract
infections. The management of patients with
Postnatal serial US exams revealed
pelviureteric junction or vesicoureteric junction
increasing dilatation of the right obstruction depends on clinical features and
pelvicalyceal system. No reflux was results of sequential ultrasonography and
demonstrated on a MUCG. Appropriate radionuclide renography. Surgery is considered in
management should include? patients with increasing renal pelvic APD and/or
a. Surgical repair an obstructed renogram with differential renal
b. Intermittent catheterization function <35-40% or its subsequent decline.
c. Diuresis renography Further studies are necessary to clarify the role of
d. Anticholinergic agents prenatal intervention, frequency of follow up
e. Phenylpropanolamine investigations and indications for surgery in these
f. Gellhorn pessary patients.
g. Biofeedback-assisted behavioral 1679. Jean is a 72yo woman with
h. treatment recurrent bowel cancer following a hemi-
i. Oral Estrogen therapy colectomy 2y ago. She is known to have
j. Vaginal Estrogen therapy
both local recurrence and liver mets and
k. Ring pessary
her pain has been under control on MST
90mg bd. She has had quite severe pain in
Ans : c. Diuresis renography the RUQ for the past hour despite having
taken her normal dose of MST. You find
that she has an enlarged liver which is
hard and irregular. There is marked
Understanding the pathophysiology of prenatally localised tenderness over the right lobe of
diagnosed RPD allows different interpretations of her liver. Her abdomen is otherwise soft
the diuretic renogram and may affect the and non-tender and the bowel sounds are
treatment of these children. Bju 2001 normal. She is apyrexial. The tx of choice
Revised Guidelines on Management of Antenatal would be?
Hydronephrosis a. Oral NSAIDs
b. TENS
Widespread antenatal screening has resulted in c. radio therapy to the liver
increased detection of anomalies of the kidneys d. IM diamorphine
and urinary tract. The present guidelines update e. Paracetamol
the recommendations published in 2000. f. Prednisolone
Antenatal hydronephrosis (ANH) is transient and g. Physiotherapy
resolves by the third trimester in almost one-half h. epidural anaesthetic
cases. The presence of oligohydramnios and i. Pitocin
additional renal or extrarenal anomalies suggests j. Aspirin
significant pathology. All patients with ANH
should undergo postnatal ultrasonography; the
intensity of subsequent evaluation depends on Ans: I/M diamorphine . Pain ladder
anteroposterior diameter (APD) of the renal
pelvis and/or Society for Fetal Urology (SFU) 1680. Titubation is a feature of disease
grading. Patients with postnatal APD exceeding involving the?
10 mm and/or SFU grade 3-4 should be screened a. Cerebellum
for upper or lower urinary tract obstruction and b. Basal ganglia
vesicoureteric reflux. Infants with vesicoureteric c. Corpus callosum
reflux should receive antibiotic prophylaxis d. Pons
e. Temporal lobe Investigations: Hepatitis B surface antigen is
f. Occipital lobe positive in 30%.
g. Optic chiasma
h. 3rd ventricle • The p-ANCA test is usually negative in
i. Hypothalamus PAN.
• There is a prominent acute phase
response but this is nonspecific.
Ans: Cerebellum • FBC shows leukocytosis with raised
neutrophils.
titubations (head/body nodding) mostly occur • Hypergammaglobulinemia occurs in 30%.
due to cerebellar lesions. • Biopsy
Arteriography shows aneurysms
1681. A 50yo farmer complains of pain in
his left arm. Exam: he appears to have a
neuropathy affecting isolated nerves in TREATMENT: Corticosteroids..RELAPSE add
multiple, random areas of his left arm. He Cyclophosphamide.=>
also has a palpable purpura and tender
Azathioprine useful in maintenance therapy.
nodules on both of his upper and lower
limbs. A likely diagnosis is? (IV-Ig) and aspirin are effective in childhood PAN.
a. Carpal tunnel syndrome
b. Polyarteritis nodosa
c. Angina Pectoris 1682. A patient with chronic neutropenia
d. Gout
e. Cellulitis develops a chronic cough. A CXR reveals a
f. Rheumatoid arthritis cavitating intrapulmonary lesion
g. Erysipelas containing a movable rounded ball lesion.
h. Fascitis A likely dx is?
i. Reiter's Syndrome a. Tuberculosis
b. Bronchiectasis
Ans: PAN c. Cystic fibrosis
d. Pulmonary hemosiderosis
e. Mitral stenosis
PAN is necrotising arteritis of medium or small f. Aspergillosis
arteries without glomerulonephritis or vasculitis g. Wegener’s granulomatosis
in arterioles, capillaries, or venules, and not h. Goodpasture’s syndrome
i. Pulmonary embolism
associated with antineutrophil cytoplasmic
j. Non-SCLC
antibodies (ANCAs).It can affect any organ but,
for unknown reasons, it spares the pulmonary
and glomerular arteries. Ans: Aspergillosis.
Presentaion:Peripheral nerves and skin are the mostly affects people with reduced immunity,
most frequently affected tissues. reduced neutrophil count is also predilection for
PURPURA,LIVEDOID,SUBCUTANEOUS NODULES aspergillosis. five clinical types of Aspergillosis
and NECROTIC ULCERS. Neurologically,
APBA
MONONEURITIS MULTIPLEX>...involvemnet of
CNS,Git,kidneys and heart means higher Severe Asthma with fungal sensitization
mortality.RENAL
INVOLVEMENT:hypertension,AKI, Aspergilloma : (fungal ball in already caivitated
GIT:necrosis,perforation.Myalgia space due to TB, Sarcoidosis)
Invasive aspergillosis A rough barking cough with croup
labored breathing
Instillation of amphotericin
1684. INR:Normal, APTT:Elevated,
hemoptysis needs to be treated with bronchial Thrombin time:Elevated, Plt
artery embolization. count:Normal, Bleeding time: Normal. a
likely aetiology is?
a. Waldenström's
1683. A mother brings her 1yo infant to b. macroglobulinaemia
her pediatrician. She describes that c. Heparin
following a common cold her child's voice d. Sézary cell leukaemia
e. Pelger-Hüet anomaly
has become hoarse and has developed a
f. von Willebrand's disease
cough that sounds harsh and brassy and g. Haemophilia
was worse at night. Exam: the child was h. HIV infection
noted to have trouble drawing air into its i. DIC
lungs between coughs and had trouble j. Acanthocytosis
drawing air into its lungs. There was k. Vit K deficiency
visible stridor on inhalation. The cause is
most likely to be? Ans: Heparin
a. EBV
b. Rhinovirus
c. Parainfluenza PT-test for extrinsic system
d. Flavivirus INR- 0.9-1.2 (PT Control), Increased INR in
e. HIV warfarin , Vitamin K and liver disease
f. Rotavirus
g. CMV APTT- intrinsic system, increased PTT (Heparin,
h. Kemerovo Haemophillia (Factor 8 affected))
i. Creutzfeld-Jacob
j. Rubella Thrombin Time- 10-15 seconds, increased in
heparin, increased in DIC
a. Osteoporosis
b. Thiazide diuretics 1693. A young girl with a psychiatric hx on
c. Skeletal metastases med tx is brought to the dermatologist by
d. Primary hyperparathyroidism her mother because of recurrent patchy
e. Hypoparathyroidism hair loss. Exam: the hair shafts revealed
f. Osteomalacia twisting and fractures. This suggests the
g. Multiple myeloma
h. Paget's disease of bone
following pathology:
a. Infection with Trichophyton tonsurans
i. Sarcoidosis
b. Infection with Microsporum canis
j. Hyperthyroidism
c. Alopecia areata
d. Telogen Effluvium
Ans : i. Sarcoidosis e. Androgenetic Alopecia
f. Lichen planus
g. Traction Alopecia
h. Alopecia totalis 1696. A 7yo boy with frequent episodic
i. Trichorrhexis nodosa asthma is on tx with sodium
j. Trichotillomania
cromoglycate. His physician wants to add
a non-steroid preventer. The mother of
Ans : j. Trichotillomania the boy, a teacher, has just read about a
1694. Syphilis typically causes nonsteroidal medication which acts on the
a. Lymphogranuloma Venereum mast cells, stopping them from releasing
b. Testicular Torsion harmful chemicals. Her physician agrees
c. Hydrocele to add this medication to the boy's drug
d. Epididymitis regimen. Which medication is the
e. Seminoma physician most likely to add to the boy's
f. Mature teratoma
g. Varicocele
treatment?
h. Lymphoma a. Inhaled short acting
i. Orchitis b. bronchodilator
j. Spermatocele c. SC adrenaline
d. Nedocromil Sodium
e. Inhaled long acting
Ans : i.Orchitis f. bronchodilator
g. Inhaled sodium cromoglycate
Chlamydia and gonorrhea are implicated in a h. Inhaled steroids
number of above conditions but syphilis is kind of i. Inhaled SABA
eccentric disease j. Oral steroids
k. Nebulised bronchodilators
1695. A middle aged woman has severe l. Oral theophylline
collapse of the right femoral head
requiring replacement. The removed
Key : C
femoral head is sent for pathology and is
found to contain enlarged fat cells. The Investigations for asthma
pathologist explains that this is the likely Spirometry:
cause of the patient's femoral head
collapse. A likely aetiology is? FEV1:FVC ratio, A low value indicates that you
a. Septic emboli have narrowed airways which are typical in
b. Impaired venous drainage asthma Therefore, spirometry may be repeated
c. Hgb SS disease after treatment. An improvement in the value
d. Steroid use after treatment with a bronchodilator to open up
e. Alcoholism the airways is typical of asthma.
f. Gaucher's disease
g. missed fracture PEFR: morning readings are usually lower than
h. Cushing's disease evening readings in asthmatics
i. Radiation
j. Vasculitis
Gaucher's disease fits here in the scenario 1697. A 3yo boy is playing with his brother
although common causes are Steroid use and
when he falls. He cries immediately and
trauma.
refuses to walk. His mother carries him to
hospital. He had a full term NVD with no
neonatal complications. His
immunisations are up to date. Exam: looks vomiting. He was connected to vital
well and well-nourished, no dysmorphic monitors which were not reassuring.
features. He has slight swelling, warmth What is the management?
and discomfort on the lower 1/3 of the left a. Oral antacids
tibia, and refuses to weight bear. AP and b. IV PPI
lateral x rays of the tibia are normal. c. Oral PPI
What is the most likely dx? d. Endoscopy
a. Ankle fx e. Analgesia
b. Ankle sprain
c. Fibular fx
key)D
d. Knee dislocation
e. Tibial fx Ans) long term use of an nsaid predisposes to
peptic ulcers. the symptoms described are of a
Key)E possibly perforated peptic ulcer or acute upper Gi
bleed caused by the ulcer. The guidelines suggest
Ans)Toddler's fracture
that
• Undisplaced spiral fractures of the tibial
shaft in children under 7 years old often follow Endoscopy is the primary diagnostic investigation
minimal trauma and may not be visible on initial in patients with acute UGIB
X-ray.[8] • Endoscopy should be undertaken
• Can be difficult to diagnose but should be
immediately after resuscitation for unstable
suspected whenever a child presents with a limp
patients with severe acute UGIB.
or fails to bear weight on the leg.
• Treatment consists of immobilisation for • Endoscopy should be undertaken within
a few weeks to protect the limb and to relieve 24 hours of admission for all other patients with
pain. UGIB.
• Subperiosteal bone formation is usually
apparent on X-rays by two weeks.
1700. A 68yo man presents with bruising
and hx of falls. He is found to have a
1698. Which one of the following
mask-like face, pillrolling tremor and
electrocardiographic changes is found in
shuffling gait. EEG=normal. Which of the
hypercalcaemia?
a. Increased QRS interval following conditions is he most likely
b. Prolonged Q-T interval being treated for?
a. HTN
c. Short P-R interval
b. DM
d. Short Q-T interval
c. Psychosis
d. TIA
e. Complex partial seizure
key)D-- Short Q-T interval secondary to a
shortened ST segment
key) C
Ans)Fact
Ans) features are classic for parkinson’s disease.
1699. An elderly male pt with prior hx of and people with PD usualy go on to develop
hematemesis is having hx of long term use parkinson related dementia or psychosis.
Symptoms of Parkinsons:
of aspirin and other drugs, now presents
with severe epigastric pain, dysphagia and
• Slowness of movement (bradykinesia). banging his head on the steering wheel,
For example, it may become more of an effort to which revealed no abnormality. 6m
walk or to get up out of a chair. This is a 'shuffling'
following this episode his symptoms have
walk with some difficulty in starting, stopping,
and turning easily. resolved. What did his original symptoms
• Stiffness of muscles (rigidity), and muscles likely represent?
may feel more tense. Also, your arms do not tend a. Conversion disorder
to swing as much when you walk. b. PTSD
• Shaking (tremor) is common, but does c. Somatization disorder
not always occur. It typically affects the fingers, d. GAD
thumbs, hands, and arms, but can affect other
e. Post-concussion syndrome
parts of the body. It is most noticeable when you
are resting. It may become worse when you are
anxious or emotional. It tends to become less
when you use your hand to do something such as Ans. The key is E. Post concussion syndrome.
picking up an object. [Post concussion syndrome is a set of symptoms
that may continue for weeks, months, or a year or
it is diagnosed clinically and treatment includes more after a concussion – a minor form of
traumatic brain injury].
1. levodopa plus a dopa decarboxylase
inhibitor 1703. A 34yo man had a 4mm ureteric
2. dopamine agonists stone which he passed in urine. This time
3. anticholinergics (tremor) he presents withh 3cm stone in the right
4. MAO inhibitors kidney. Single most appropriate
treatment?
1701. A 45yo woman presents with easy a. No treatment
fatigability, even on no exertion, chronic b. ESWL
headaches and body aches and severe c. Laparotomy
physical and mental exhaustion. She has d. Observe
no underlying conditions and all inv are e. Operative stone removal
non-conclusive. What is the most likely Ans. The key is E. Operative stone removal.
dx?
a. Somatization [Stones < 5mm: pass spontaneously, Inc fluid
b. Chronic fatigure syndrome intake
c. Polymyalgia rheumatic Stones 5mm-1cm /pain not resolving: medical
d. GCA e. Depression expulsive therapy---> Nifedipine or
Tamsulosin(and/or prednisolone)
Stones 1cm-2cm: ESWL or Ureteroscopy using
dormia basket
Ans. The key is B. Chronic fatigue syndrome.
Stones > 2cm/large/multiple/complex:
[Chronic fatigue syndrome (CFS) causes persistent
Percutaneous nephrolithotomy].
fatigue (exhaustion) that affects everyday life and
doesn't go away with sleep or rest].
1704. An 18m girl who has had single UTI
1702. A 23yo male presents to his GP is seen in the OPD. She has fever and
2wks after a RTA concerned about vomiting but these improved with course
increasing anxiety lethargy and headache. of trimethoprim. Subsequently, MCUG
At the time he had a CT brain after
showed bilateral vesicoureteric reflux.
Single most appropriate mgmt?
a. Prophylactic antibiotics
b. Reassure
c. No treatment
d. Ureteric surgery